Download as pdf or txt
Download as pdf or txt
You are on page 1of 142

[CAREER AID FOR BANK AND BCS] Written Math

Formulas
Number System
msL¨v c×wZ

Number

Imaginary Real

Complex Rational Irrational

Fraction Integers

Negative Zero Positive

Whole Natural
Numbers Numbers

Even Odd Prime Composite


numbers Numbers Numbers Numbers

Co Prime Numbers/ Twin Mersennes


Reactively Prime Prime
prime numbers Numbers Numbers

Real numbers (ev¯Íe msL¨v)


Rational and irrational numbers ক এক সে Real numbers ejv nq|
For example: 0, 1, 2/3, 6/11. 2, 3.... etc
Complex number (RwUj msL¨v):
A complex number is a number that can be expressed in the form a + bi, where a and b are
real numbers, and i is a symbol called the imaginary unit, and satisfying the equation i2 = −1.
Because no "real" number satisfies this equation, i is called an imaginary number.
Rational Numbers (g~j` msL¨v)
p
য সকল numbers ক form এ লখা যায়, তােক Rational numbers e‡j|
q
But two condition আেছ first p and q েটাই integer হেব and 2nd condition হল q not equal zero হেত
হেব।
For example : 0, 1, 2, 1/4, 2/7, 0.6, -2/7,...,etc.
Rational number ক ‘Q’ িদেয় denote করা হয়।
[CAREER AID FOR BANK AND BCS] Written Math

p
Decimal numbers ও rational numbers Gi g‡a¨ Av‡m, KviY decimal number ‡K Avgvi q from এ কাশ
3
করেত পাির। যমন: 0.3 = 10 0.7 = 7/10,...etc.
p
0 = 0/1 = p/q, A_©vr Avgiv '0' ‡K from এ িলখেত পাির, so, zero is a rational number.
q
22
'π' rational number নয় but 'π' এর value 3.1416 and rational number because 3.1416 π এর
7
actual value না nearest values.
Irrational Numbers (Ag~j` msL¨v)
p
‡h mKj number †K from G cÖKvk করা যায়না, যখােন p,q integer and q not equal to zero, তােক
q
irrational number বা অ লদ সং া বেল।
For example: 2, 5,..., 0.10110111011110.......
Even Numbers (hyM¥ msL¨v ev †Rvo msL¨v)
যসম numbers ক 2 িদেয় divide করা যায়, Zv‡`i‡K even numbers বলা হয়।
Example: 2,4,6,8,10,12,14,16,18,20,........etc.
Odd Numbers (AhyM¥ msL¨v ev we†Rvo msL¨v)
য সম numbers ক 2 িদেয় divide করা যায় না, Zv‡`i‡K odd numbers বলা হয়।
Example: 3,5,7,9,11,13,15,17,...........,etc.
Integers (c~b© msL¨v)
Whole numbers and negative numbers (-1,-2,-3 ...) ক একসে integer বেল।
Integer’s এ positive and negative `yB ai‡Yi numbers থােক।
Integer two type এর হয় positive integer and negative integer.
Zero †_‡K eo right hand side Gi numbers ক positive integer and zero †_‡K †QvU left hand side এর
number ক negative integer বলা হয়।
For example: ...,-2,-1,0.+1,+2,+3,.....etc.
Note:
Integer ক 'I' or 'Z' িদেয় denote করা হয়।
'0' (zero) এক integer but positive or negative integer নয়।
Natural Numbers (¯^vfvweK msL¨v)
' 1 ' ‡_‡K ïiæ K‡i cici' 1 ' add K‡i ‡h mKj numbers cvIqv hvq, Zv‡`i‡K ¯^vfvweK msL¨v ev natural number
ejv হয়। natural numbers MYbv Kiv †_‡KB m„wó n‡q‡Q, ZvB natural numbers is called counting
number also.
For example: 1,2,3,4,5,6,7,8,9,10,............etc.
Note:
Natural or counting numbers ক' N ' িদেয় denote Kiv nq|
Smallest natural number হল '1'
1. Even Natural Numbers:
[CAREER AID FOR BANK AND BCS] Written Math

A system of natural numbers, which are divisible by 2 or are multiples of 2, is called a set of even
numbers.
E= (2, 4, 6, 8, 10, 12…….)
2. Odd Natural Numbers:
O = (1, 3, 5, 7, 9………)
Whole Number (অখ সং া ) :
'0' †_‡K ïiæ K‡i cici '1' add কের য সকল numbers cvIqv hvq, Zv‡`i‡K ALÛ msL¨v ev whole number ejv
nq|
Natural numbers এর সে '0' add K‡i whole numbers এর দল পাওয়া যায়। তাই আমরা বলেত পাির '0' and
natural number িমেল whole numbers তির হয়।
For example: 0,1,2,3,4,5,6,7,8,9......etc.
Note:
Smallest whole number is '0'
Prime Number (‡gŠwjK msL¨v)
†h mg¯Z number ক 1 Ges †mB msL¨v Qvov Ab¨ †Kv‡bv msL¨v w`‡q divide Kiv hvqbv Zv‡`i‡K prime number
e‡j|
Ex : 2,3,5,7,11,13,17,19,23,......etc.
Notes:
Smallest prime number is '2'.
একমা even prime number হল '2'.
পরপর prime number হল 2 and 3
1-50 এর মে 15 prime numbers আেছ।
50-100 এর মে 10 prime numbers আেছ।
1-100 এর মে 25 prime numbers আেছ।
GK †_‡K GKk (1 - 100) chšÍ© †gŠwjK msL¨v ¸‡jv nj :
2,3,5,7,11,13,17,19,23,29,31,37,41,43,47,53,5 9,61,67,71,73,79,83,89,97= 25 ।
†gŠwjK msL¨v wbY©q
mvaviYZ ‡Kvb msL¨vi ‡kl wWwRU 0, 2, 4, 5, 6, 8 n‡j msL¨vwU ‡gŠwjK nqbv|
91 wK ‡gŠwjK msL¨v ?
1৷ †h msL¨vwU †gŠwjK msL¨v wKbv Rvb‡Z Pv‡”Qb Zvi wbKUeZ©x (wb‡P) GKwU c~b© eM© msL¨v wbY©q Kiæb৷ Ex: 100> 91 >
81
2| Gevi H c~b©eM© msL¨vi eM©g~j wbb©q Kiæb ৷
81 = 9
3| H eM©g~j msL¨vwUi wb‡P hZ¸jv †gŠwjK msL¨v Av‡Q †mB msL¨v¸‡jv w`‡q †h msL¨vwU ‡gŠwjK (91) wKbv Rvb‡Z Pv‡”Qb
Zv‡K fvM K‡i †`Lyb wbt‡k‡l wefvR¨ nq wKbv| wbt‡k‡l wefvR¨ n‡j Zv †gŠwjK msL¨v bq, wbt‡k‡l wefvR¨ bv n‡j
eyS‡eb †gŠwjK msL¨v| 9 msL¨vwUi wb‡P hZ¸‡jv †gŠwjK msL¨v = 7, 5, 3, 2
91, 3 w`‡q wbt‡k‡l wefvR¨ | [‡gŠwjK msL¨v bq]
Twin prime number:
যসম prime numbers এরমে 2 এর difference আেছ , H pair of prime numbers ক twin prime number বলা
হয়।
Ex: 3 and 5, 5 and 7,... etc.
[CAREER AID FOR BANK AND BCS] Written Math

Prime Triplet:
Prime triplet ওই 3 prime numbers †K ej‡ev hv‡`i g‡a¨ 2-এর difference আেছ। কবল মা এক set prime
triplet number Av‡Q, Zvn‡jv {3, 5, 7}.
Co-prime number/Relative Prime, Mutually Prime (ci¯ci ‡gŠwjK msL¨v)
hw` `ywU natural number Gi g‡a¨ '1' Qvov Ab¨ †Kv‡bv common factor bv _v‡K, ZLb H `ywU natural number
†K Co-prime number ev ci¯ci †gŠwjK msL¨v ejv nq। However, two composite numbers can also be
co prime or relatively prime.
Example: 7 and 9, 5 and 8 ... etc
Condition
Numbers have no common factors other than 1
So the greatest common factor of coprimes is 1
Multiple
 All natural numbers are whole numbers.
 All whole numbers are not natural numbers.
 0 is a whole number which is not a natural number.
 The smallest prime number is 2.
 The only even prime number is 2.
 The first odd prime number is 3.
 1 is a unique number – neither prime nor composite.
 The least composite number is 4.
 The least odd composite number is 9.
Even number + Even number = Even number
Odd number + Odd number = Even number
Even number + Odd number = Odd number
Even number – Even number = Even number
Odd number – Odd number = Even number
Even number – Odd number = Odd number
Odd number – Even number = Odd number
Even number × Even number = Even number
Odd number × Odd number = Odd number
Even number × Odd number = Even number
Divisibility Test
Divisibility by 2
If a number is even or a number whose last digit is an even number i.e. 2,4,6,8 including 0, it is
always completely divisible by 2.
2 Øviv wefvR¨Zvi wbqg: hw` †Kvb msL¨vi †kl ev GKK ¯’vbxq A¼ïb¨ ev †Rvo msL¨v nq,
Z‡e msL¨vwU 2 w`‡q fvM Kiv hv‡e|
Divisibility Rules for 3
Divisibility rule for 3 states that a number is completely divisible by 3 if the sum of its digits is
divisible by 3, it is a multiple of 3
3 Øviv wefvR¨Zvi wbqg: hw` †Kvb msL¨vi mgwó 3 w`‡q fvM Kiv hvq, Z‡e msL¨vwU 3 w`‡q fvM Kiv hv‡e
[CAREER AID FOR BANK AND BCS] Written Math

Consider a number, 308. To check whether 308 is divisible by 3 or not, take sum of the digits (i.e.
3+0+8= 11). Now check whether the sum is divisible by 3 or not. If the sum is a multiple of 3
then the original number is also divisible by 3. Here, since 11 is not divisible by 3, 308 is also not
divisible by 3.
Similarly, 516 is divisible by 3 completely as the sum of its digits i.e. 5+1+6=12, is a multiple of 3.
Divisibility by 4
If the last two digits of a number are divisible by 4, then that number is a multiple of 4 and is
divisible by 4 completely.
4 Øviv wefvR¨Zvi wbqg: †Kvb msL¨vi †kl `ywU A¼ ïb¨ A_ev 4 Øviv wefvR¨ nq Z‡e msL¨vwU 4 Øviv wefvR¨|
Example: Take the number 2308. Consider the last two digits i.e. 08. As 08 is divisible by 4, the
original number 2308 is also divisible by 4.
Divisibility by 5
Numbers which last with digits, 0 or 5 are always divisible by 5.
5 ারা িবভাজ তার িনয়ম: কানসং ার একক ানীয় অ অথবা 5 থােক তেব, সং া 5 ারা িবভাজ ।
Example: 10, 10000, 10000005, 595, 396524850 etc.
Divisibility by 6
Numbers which are divisible by both 2 and 3 are divisible by 6. That is, if the last digit of the
given number is even and the sum of its digits is a multiple of 3, then the given number is also a
multiple of 6.
6 Øviv wefvR¨Zvi wbqg: †KvbmsL¨v 2 I 3 Øviv wefvR¨ nq Z‡e msL¨vwU 6 w`‡q fvM Kiv hv‡e|
Example: 630, the number is divisible by 2 as the last digit is 0.
The sum of digits is 6 + 3 + 0 = 9, which is also divisible by 3.
Hence, 630 is divisible by 6.
Divisibility Rules for 7
The rule for divisibility by 7 is a bit complicated which can be understood by the steps given
below:
Example: Is 1073 divisible by 7?
 From the rule stated remove 3 from the number and double it, which become 6.
 Remaining number becomes 107, so 107-6 = 101.
 Repeating the process one more time, we have 1 × 2 = 2.
 Remaining number 10 – 2 = 8.
 As 8 is not divisible by 7, hence the number 1073 is not divisible by 7.
Another way
7 Øviv wefvR¨Zvi wbqg: †Kvb msL¨vi Wvbw`K †_‡K ïiæ K‡i wZbwU K‡i A¼ wb‡q MwVZ msL¨v¸wji hyM¥ ¯’vbxq m•Lv¸wji
†hvMdj I AhyM¥ ¯’vbxq msL¨v¸wji †hvMd‡ji we‡qvMdj '0' A_ev '7' w`‡q fvM Kiv hv‡e , Zvn‡j msL¨vwU 7 w`‡q wefvR¨|
†hgb- 629 079 976: (629 + 976) - 079 = 152,hv 7 w`‡q fvM Kiv hv‡e| ZvB msL¨v¸wj 7 Øviv fvM Kiv hv‡e
Divisibility by 8
If the last three digits of a number are divisible by 8, then the number is completely divisible by
8.
[CAREER AID FOR BANK AND BCS] Written Math

8 Øviv wefvR¨Zvi wbqg: hw` †Kvb msL¨vi †k‡li wZbwU A¼ 0 A_ev 8 w`‡q fvM Kiv hvq, Z‡e †mB msL¨vwU 8 w`‡q fvM
Kiv hv‡e|
Example: Take number 24344. Consider the last two digits i.e. 344. As 344 is divisible by 8, the
original number 24344 is also divisible by 8.
Divisibility by 9
If the sum of digits of the number is divisible by 9, then the number itself is divisible by 9.
9 Øviv wefvR¨Zvi wbqg: hw` †Kvb msL¨vi mgwó‡K 9 w`‡q fvM Kiv hvq Z‡e †mB msL¨vwU 9 w`‡q fvM Kiv hv‡e|
Example: Consider 78532, as the sum of its digits (7+8+5+3+2) is 25, which is not divisible by 9,
hence 78532 is not divisible by 9
Divisibility by 10
Divisibility rule for 10 states that any number whose last digit is 0, is divisible by 10.
Example: 10, 20, 30, 1000, 5000, 60000, etc.
Divisibility Rules for 11
If the sum of all digits of a number is divisible by 11, then that number is divisible by 11
completely.
In order to check whether a number like 2143, is divisible by 11, below is the following
procedure.
 Sum of the digits = 2+1+4+3=10
 10 is not divisible by 11
 Therefore, 2143 is not divisible by 11.
Another way
11 Øviv wefvR¨Zvi wbqg: †Kvb msL¨vi AhyM¥ ¯’vbxq A‡¼i mgwó I hyM¥ ¯’vbxq A‡¼i mgwói AšZi dj 0 nq A_ev 11 Øviv
fvM Kiv hvq Z‡e †mB msL¨vwU 11 w`‡q fvM Kiv hv‡e| †hgbÑ
36602984 = (3 + 6 + 2 + 8) - (6 + 0 +9 + 4) = 19 - 19 = 0.
12 Øviv wefvR¨Zvi wbqg: †Kvb msL¨v‡K hw` c„_K c„_K fv‡e 3 I 4 w`‡q fvM Kiv hvq Z‡e msL¨vwU 12 w`‡q fvM Kiv
hv‡e|
Local Value/Place value and Face value
Place value or local value of a digit in a numeral is the value of digit, depending on its place. We
can relate place with 'place value'.
Face value of the digit in numeral is value of digit itself, irrespective of its place in numeral.
For example, we have 435676
Here place value of 5 is 5000, while its face value is 5.
Similarly, place value of 3 is 30000, while its face value is 3.
GCF/ HCF and LCM
Formula: Product of numbers = (HCF of the numbers) x (LCM of the numbers)
`ywU msL¨vi ¸Ydj = `ywU msL¨vi jmv¸  `ywU msL¨vi Mmv¸
Least number exactly divisible
LCM = Least Common Factor
Step 01: Find LCM
Multiple of LCM is required number
[CAREER AID FOR BANK AND BCS] Written Math

Anotherway
Step 01: Find LCM
Step 02: Divide the least “n” digit number by LCM
Step 03: P =LCM – remainder of the division
Required number = “n” digit number + p
Average and Statictics
Mo I cwimsL¨vb
Sum of quantities cÖ`Ë ivwki mgwó
1. m~Î : Average = A_ev, Mo = cÖ`Ë ivwki msL¨v
Number of quantities  
2. The average of first n odd number is n
3. The average of first n even number is n + 1
n+1
4. The average of first n natural number is
2
nth number + (n + x)th number
5. Find the average of odd / even number from n to n + x is
2
Median (ga¨K)
The median is the middle value of the observations
nth observation + n + 1th observation
2 2 
   
For even number =
2
n+1 th
For odd number = observation
2
Mode (cÖPziK)
The mode is the value that appears most often in a set of data values.
Range: The Range is the difference between the lowest and highest values. [†iÄ nj me©wbgœ Ges
m‡e©v”P gv‡bi g‡a¨ cv_©K¨|]
Standard deviation: Standard deviation is a number used to tell how measurements for a group
are spread out from the average [cwimsL¨v‡bi †¶‡Î, gvb weP¨ywZ nj gvb¸wji GKwU †m‡Ui Qwo‡q covi cwigvc|
GKwU wbg&œ gvb weP¨ywZ wb‡`©k K‡i †h gvb¸wj †m‡Ui M‡oi KvQvKvwQ _v‡K, hLb GKwU D”P gvb weP¨ywZ wb‡`©k K‡i †h
gvb¸wj we¯Í…Z cwim‡i Qwo‡q c‡o]
∑(x – mean)2
σ=
n
x = set of the numbers
σ = standard deviation
n = number of observation
Variance: The variance is the average of the squared differences from the mean
∑(xi  
x)2
s2 =
n1
2
s = sample variance
[CAREER AID FOR BANK AND BCS] Written Math

xi = value of the observation



x = mean
n = the number of observations
Some other formulas
sum of the observation x1 + x2 + x3 + ...... + xn
Arithmetic mean (MvwYwZK Mo), AM = 
x= =
number of observation n
n
Geometric mean (R¨vwgwZK Mo), GM = x1  x2  x3  .......  xn
n
Harmonic mean (nvigwbK Mo), HM =
1 1 1 1
+ + + ........ +
x1 x2 x3 xn

w1x1 + w2x1+w3x3 + ..... wnxn


Weighted mean/average =
w1 + w2 + ...... + wn
Relation among AM, GM and HM: AM × HM = GM2
Descending order relationship among AM, GM and HM: AM>GM>HM
Weighted Average
If we have “n” groups with weights W1, W2, W3, W4, ……., Wn and having X1, X2, X3, X4, ..., Xn elements
then the weighted average is given by the formula
x1  w1 + x2  w2 + x3  w3 + ......+ xn  wn
AW =
w1 + w2 + ........ wn
Use of mean
Geometric mean: Population growth rate, portfolio returns, stock indexes, water quality
standard etc
Harmonic mean: average speed of car, Insurance premium rate calculation, average purchased
cost of Bankers share over time etc
Weighted mean: Cumulative grade point average
Average Speed
Total distance
Average speed =
Total time
Same distance at two different speeds
If a person travels two equal distances at a speed of “a” km/hr and “b” km/hr, then average
speed ” A ” km/hr [যিদ একজন ি "a" km/hr এবং "b" km/hr এর গিতেত সমান র মণ কের, তাহেল গ
2ab
গিত "A" km/hr] A= a + b
Same distance at three different speeds
If a person travels three equal distances at a speed of “a” km/hr, “b” km/hr & “c” km/hr, then average
speed” A “km/hr[যিদ একজন ি "a" km/hr, "b" km/hr এবং "c" km/hr এর গিতেত িতন সমান র মণ কের, তাহেল গ
3ab
গিত "A" km/hr], A =
ab + bc + ca
[CAREER AID FOR BANK AND BCS] Written Math

Age related problems


eqm msµvšÍ mgm¨vejx
1. If the current age is x, then n times the age is nx.
2. If the current age is x, then age n years later/hence = x + n. [ যিদ বতমান ব স x হ , তাহেল ব স n বছর পের ব স = x +
n]
3. If the current age is x, then age n years ago = x - n. [যিদ বতমান ব স x হ , তাহেল ব স n বছর আেগ ব স = x – n]
4. The ages in a ratio a : b will be ax and bx.
1 x
5. If the current age is x, then th of the age is
n n
Algebra
exRMwYZ
exRMvwYwZK m~Îvejx :
1. (a+ b)² = a²+2ab+b² 13. (a+ b) ³ = a³+ b³+ 3ab× (a+ b)
2. (a+ b)² = (a-b)²+4ab 14. (a- b)³= a³- 3a²b+ 3ab²- b³
3. (a- b)² = a²-2ab+b² 15. (a- b) ³= a³-b³-3ab(a-b)
4. (a - b)² = (a+ b)²- 4ab 16. a³+ b³= (a+ b) (a²-ab+b²)
5. a² + b² = (a+ b)² - 2ab 17. a³+ b³= (a+ b) ³-3ab(a+ b)
6. a² + b² = (a- b)² +2ab. 18. a³- b³ = (a-b) (a²+ab+b²)
7. a²- b² = (a + b) (a -b) 19. a³- b³ = (a-b) ³+3ab(a- b)
8. 2(a²+b²) = (a+ b)²+ (a- b) ² 20. (a² + b² + c²) = (a + b + c)² – 2(ab + bc + ca)
9. 4ab = (a+ b) ²-(a-b)² 21. 2 (ab + bc + ca) = (a + b + c) ² – (a² + b² + c²)
a+b 2 ab 2 22. (a + b + c) ³ = a³ + b³ + c³ + 3 (a + b) (b + c) (c + a)
10. ab = ( ) -( )
2 2 23. a³ + b³ + c³ – 3abc = (a+ b+ c) (a² + b² + c²–ab–bc– ca)
11. (a+ b+ c)² = a²+ b²+ c² + 2× (ab+ bc+ ca)
12. (a+ b)³ = a³+ 3a²b+ 3ab²+ b³
Equation
mgxKiY
mgxKiY (Equation) n‡jv msL¨v I cÖZxK e¨envi K‡i †jLv GK ai‡bi MvwYwZK wee„wZ, hv‡Z `yBwU wRwbm‡K
MvwYwZKfv‡e mgvb ev mgZzj¨ †`Lv‡bv nq| mgvb wPý (=) e¨envi K‡i mgxKiY †jLv nq, †hgbÑ
cÖwµqv wPý cÖwµqv wPý

Pj ‡Ki NvZ mgvb wPý aªæeK

Wvbcÿ 5x2  3 = 2x + 3 evgcÿ

mnM
PjK
Number of solution of an equation
[CAREER AID FOR BANK AND BCS] Written Math

If two equation are given as:


ax + by = c
dx + ey = f
a b
 Unique solution(GKwU mgvavb)
d e
a b c
= = Infinite solution (AmsL¨ mgvavb)
d e f
a b c
d = e  f No solution (†Kvb mgvavb †bB)
Quadratic equation
ax²+ bx + c = 0 wØNvZ mgxKiY
GLv‡b, (b² - 4ac) Gi gvb ch©v‡jvPbv Ki‡jB wØNvZ mgxKi‡Yi g~j؇qi cÖK…wZ Rvbv hvq |
GRb¨ (b²-4ac) †K wØNvZ mgxKi‡Yi wbðvqK ev wbiƒcK (Discriminant) ejv nq |
b b
i. hw` b² - 4ac = 0 Or, b² = 4ac nq Z‡e g~j `yBwU n‡e – Ges A_©vr g~j `yBwU ev¯Ze, g~j` I mgvb n‡e|
2a 2a
ii. b²- 4ac > 0 Or, b² > 4ac n‡j g~jØq ev¯Ze I Amgvb n‡e |
iii. b²- 4ac < 0 Or, b² < 4ac n‡j g~jØq AbyeÜx RwUj msL¨v n‡e |
iv. (b²- 4ac) c~Y©eM© n‡j g~jØq ev¯Ze, g~j` I Amgvb n‡e |
v. c = 0 n‡j GKwU g~j 0 n‡e |
–c –c
vi. b = 0 n‡j g~j `yBwU n‡e + Ges – A_©vr g~j `yBwUi gvb mgvb wKš‘ wecixZ wPýwewkó n‡e| j¶Yxq,
a a
G‡¶‡Î a I c GKB wPýhy³ n‡j g~jØq RwUj Ges wecixZ wPýhy³ n‡j g~jØq ev¯Ze n‡e ।

Exponent
m~PK
Rule Formula
a b a+b
Product x x =x
xa ab
Quotient =x
xb
Power of power (xa) b = xab
Power of product (xy)a = xaya
Power of one x1 = x
Power of zero x0 = 1
1
Power of negative one/inverse x1 =
x
1
Change sign of exponents xa = a
x
m
Fractional exponents n n m
xn= xm = ( x)
[CAREER AID FOR BANK AND BCS] Written Math

Logarithm
Formla 1
6. loga x =
1. loga (xy) = loga x + loga y logx a
x logb x log x
2. loga   = loga x  loga y 7. loga x = =
y logb a log a
3. logx x = 1 8. alogax = x
4. loga 1 = 0 9. xlogay = ylogax
5. loga (xp) = p (loga x) p
10. logaq xp = lognx
q
11. logxa = y n‡j xy = a

Pencentage
kZKiv
1. Percent implies “for every hundred”. % is read as percentage and x % is read as x per cent.
p p×y
2. To calculate p % of y = ( )×y= , Hence, p % of y = y % of p
100 100
y
3. To find what percentage of x is y: × 100
x
change
4. To calculate percentage change in value, Percentage change =  100%
initial value
5. Percentage point change = Difference of two percentage figures
S
6. Increase N by S % = N( 1+ )
100
S
7. Decrease N by S % = N( 1 – )
100

Increase Decrease Decrease Increase


25% 20% 20% 25%
20% 16.67% 25% 33.33%
50% 33.33% 33.33% 50%
r
Reduce consumption after price increase = ×100%
100 + r
r
Increase consumption after price decrease = ×100%
100  r
False weight formula
Error
Gain% = × 100%
True value  Error
[CAREER AID FOR BANK AND BCS] Written Math

true weight  false weight


For false weight, profit percentage will be P% =  100%
false weight
Change
Gain/loss% =  100%
Original
Successive increase/decrease
o If there are successive percentage increases/decrease of a % and b%, the effective percentage
ab
increase/decrease is: ± a ± b ± %
100
o If there are successive percentage increases/decrease of a %, b% and c%, the effective percentage
ab bc ca abc
increase/decrease is ± a ± b ± c ± ± ± ±
100 100 100 10000
o

Profit and Loss


jvf I ÿwZ
(Cost Price)µqg~j¨ †hB `v‡g GKwU wRwbm µq Kiv nq, Zv‡K µqg~j¨ e‡j|
(Selling Price) weµqg~j¨: †hB `v‡g GKwU wRwbm wewµ Kiv nq, Zv‡K weµqg~j¨ e‡j|
(Profit) jvf: hw` weµqg~j¨, µqg~‡j¨i †_‡K †ewk nq, Zvn‡j we‡µZvi jvf nq|
(Loss)ÿwZ: hw` weµqg~j¨, µqg~j¨ †_‡K Kg nq, Zvn‡j we‡µZvi ÿwZ nq|
 Profit, P = Selling Price – Cost Price; Selling Price > Cost Price
 Loss, L = Cost Price – Selling Price; Cost Price > Selling Price
 Discount = Markup Price– Selling Price
 Selling Price = Markup Price – Discount
Profit Selling price  Cost price SP  CP
 Profit% = × 100% = ×100% = ×100%
Cost price Cost price CP
Loss Cost price  Selling price CP  SP
 Loss% = ×100% = ×100% = × 100%
Cost price Cost price CP
Selling price  Cost price
 jvf ev jm weµq g~‡j¨i Dci n‡j, Profit% = × 100%
Selling price
Discounted price  Cost price
 Profit = ×100%
Cost price
Simple Interest
mij my`
i. Avmj (Principal): GKwU wbw`©ó mg‡qi Rb¨ †hB A_© avi †bIqv ev †`Iqv nq, †mB A_©‡K Avmj e‡j|
ii. my` (Interest) : wKQz cwigvY A_© avi wb‡j, ‡h AwZwi³ A_© cÖ`vb Ki‡Z nq, †mwU n‡jv my`|
iii. mij my` (Simple Interest) ev S.I.) : GKB wbw`©ó Rb¨ aviKiv A‡_©i my` Awfbœ nv‡i Av`vq Kiv n‡j, †mwU‡K mij my`
e‡j|
awi, Avmj = P, my‡`i nvi = R% eQi (p. a.) Ges mgq = n eQi| Zvn‡j
P  R  n
i. mij my` =  
 100 
100  S.I. 100  S.I. 100  S.I.
ii. P =   ;R= Ges n =  
 Rn  Pn  PR 
Easy Formula
[CAREER AID FOR BANK AND BCS] Written Math

I = Pnr
Where, I = Interest (my`), P = Principal (Avmj)
r = Rate of Interest (1 eQiv‡šÍ my‡`i nvi), n = time (mgq)
Future value = I + P = Pnr + P = P(1 + nr)
Compund Interest
Pµe„w× my`
Pµe„w× my` : wKQz †ÿ‡Î FY`vZv Ges FYMÖnxZv FY †kva Kivi Rb¨ evwl©K, lvb¥vwmK ev ˆÎgvwmK nv‡i GKwU wbw`©ó mg‡qi
GKK w¯’i K‡i| GB‡ÿ‡Î, cÖ_g GKK mg‡qi c‡ii my`-Avmj nq wØZxq GKK GKK mg‡qi Avmj| wØZxq GKK mg‡qi
c‡ii my`-Avmj nq Z…Zxq GKK mg‡qi Avmj Ges µgk| wbw`©ó mgqKvj ci, Avmj Ges my`-Avm‡ji cv_©K¨‡K †mB
mg‡qi Pµe„w× my` e‡j|
awi, Avmj = P, my‡`invi = evwl©K R%, mgq = n eQi|
 R n
I. my` evwl©K nv‡i (Yearly/Annually) n‡j: my`-Avmj = P1 + 
 100
II. my` lvb¥vwmK (Half yearly/Bi annually) nv‡i n‡j : my`-Avmj = P1 +
 (R/2)2n
 100 
 (R/4)4n
III. my` ˆÎgvwmK (Quartarly) nv‡i n‡j : my`-Avmj = P1 +
 100 
Easy Formula
nm
Future value, F.V. = P(1 + r%)n hence, F.V. = P 1 +
 r% 

 m
Interest, I = F.V. – P = P(1 + r%) – P = P {(1 + r%)n – 1} – P
n

Where, I = Interest (my`), P = Principal (Avmj)


r = Rate of Interest (1 eQiv‡šÍ my‡`i nvi), n = time (mgq), m = compounding time (Pµe„w×i mgq)
Installment
x x
Sum = +
(1 + 10%)1 (1 + 10%)2

Time Work and Unitary


mgq KvR I HwKK wbqg
1
I. hw` A GKwU KvR n w`‡b Ki‡Z cv‡i, Zvn‡j A Gi GKw`‡bi KvR = |
n
1
II. hw` A Gi GKw`‡bi KvR = nq, Zvn‡j A KvRwU n w`‡b †kl Ki‡Z cv‡i|
n
III. hw` A, B Gi Zzjbvq wZb¸Y KvR K‡i, Zvn‡j A Ges B Gi Kv‡Ri AbycvZ = 3: 1 KvRwU †kl Kivi Rb¨ A Ges B
Gi cÖ‡qvRbxqZv mg‡qi AbycvZ = 1: 3|
 If a person can do W1 work in D1 days working T1 hours in a day and M2 Persons can do W2 work
in D2 days working T2 hours in a day, then the relationship between them is
[CAREER AID FOR BANK AND BCS] Written Math

M1  D1  T1 M2  D2  T2
=
W1 W2
 If A can do a piece of work in X days and B can do the same work in Y days, then (A + B)’s one-
1 1 x+y
day work: + =
x y xy
xy
 Time taken by (x + y) to complete the work :
x+y
 If A &B can do a piece of work in x and y days, B & C can the same work in Y and Z days and A
xyz
& C can do it in X and Z days, then working together A, B & C can do that work in: days
xy + yz + zx
100  x
 If A can do a work in days and B can do faster than A, then B will complete the work in : days
100 + y
 Unit of work = day × number of work

Pipe and Cistern


bj I ‡PŠev”Pv
INLET: An inlet is a pipe which is connected to the tank and with the help of this pipe, the tank is
filled.
OUTLET/LEAK: An outlet is a pipe which is connected to the tank. This pipe drains out water
from the tank and the tank gets emptied if this pipe is opened.
Formulae
1
If a pipe can fill a tank in a hours, then the part filled in 1 hour =
a
1
If a pipe can empty a tank in b hours, then the part of the full tank emptied in 1 hour =
b
If a pipe can fill a tank in a hours and the another pipe can empty the full tank in b hours, then
1 1
the net part filled in 1 hour, when both the pipes are opened =     Time taken to fill the
a b
ab
tank, when both the pipes are opened =
ba
If a pipe can fill a tank in a hours and another can fill the same tank in b hours, then the net part
filled in 1 hour, when both pipes are opened =  + 
1 1
a b
ab
 Time taken to fill the tank =
a+b
If a pipe fills a tank in ahours and another fills the same tank in b hours, but a third one empties
the full tank in c hours, and all of them are opened together, the net part filled in 1 hour=
1 + 1  1
 a b c
 
abc
 Time taken to fill the tank = hours.
bc + ac  ab
[CAREER AID FOR BANK AND BCS] Written Math

Speed and Distance


‡eM I `~iZ¡
Formula Conversion
 Distance(`~iZ¡) = speed (‡eM) × time 5
 x kmph = x × m/s
(mgq) 18
Or, D = vt 18
Total distance 2xy  x m/s = x × kmph
5
 Average speed = =
Total Time x+y
Total distance
 Relative speed =
Total time
Train
†Uªb
 5
I. a wK.wg./N›Uv = a 
18
wgUvi/†m‡KÛ

II.
 18
a wgUvi/†m‡KÛ = a   wK.wg./NÈv|
 5
III. g‡b Ki, `ywU †Uªb A_ev `ywU mPj e¯‘ GKB w`‡K u wgUvi/†m‡KÛ Ges v wgUvi/†m‡KÛ MwZ‡e‡M hv‡”Q, Zvn‡j
Zv‡`i Av‡cwÿK MwZ‡eM = (u + v) wgUvi/†m‡KÛ, †hLv‡b u > v|
IV. g‡b Ki, `ywU †Uªb A_ev `ywU mPj e¯‘ wecixZ w`‡K u wgUvi/†m‡KÛ Ges v wgUvi/†m‡KÛ MwZ‡e‡M hv‡”Q, Zvn‡j
Zv‡`i Av‡cwÿK MwZ‡eM = (u + v) wgUvi/†m‡KÛ|
Formula Consider (+) sign for opposite
L1 + L2 direction and (-) sign for same
T= direction.
V1 ± V2
Time, t sec Conversion
Length of first train/object, L1m 5
x kmph = x × m/s
Length of first train/object, L2m 18
Speed of first train, V1 km/hr 18
Speed of second train, V2 km/hr x m/s = 5 kmph

Relative speed = V1 ± V2
Boat and Stream
‡bŠKv I †¯ªvZ
I. R‡j, †¯ªv‡Zi w`‡K hvÎv‡K AbyK‚‡j Ges †¯ªv‡Zi wecix‡Z hvÎv‡K cÖwZK‚‡j hvÎv e‡j|
II. hw` w¯’i cvwb‡Z GKwU †bŠKvi †eM u wK.wg./NÈv Ges †¯ªv‡Zi MwZ‡eM v wK.wg./NÈv nq, Zvn‡j
AbyK‚‡j †eM = (u + v) wK.wg./NÈv
cÖwZK‚‡j †eM = (u  v) wK.wg./NÈv
III. hw` AbyK‚‡j †eMa wK.wg./NÈv Ges cÖwZK‚j †eM b wK.wg./NÈv nq, Zvn‡j;
[CAREER AID FOR BANK AND BCS] Written Math

1
w¯’i cvwb‡Z MwZ‡eM = 2(a + b) wK.wg./NÈv
1
†¯ªv‡Zi †eM = (a  b) wK.wg./N›Uv
2
Stream – The moving water in a river is called stream.
Upstream – If the boat is flowing in the opposite direction to the stream, it is called upstream. In
this case, the net speed of the boat is called the upstream speed
Downstream – If the boat is flowing along the direction of the stream, it is called downstream.
In this case, the net speed of the boat is called downstream speed
Still Water – Under this circumstance the water is considered to be stationary and the speed of the water
is zero
Upstream = (u−v) km/hr, where “u” is the speed of the boat in still water and “v” is the speed of the
stream
Downstream = (u+v)Km/hr, where “u” is the speed of the boat in still water and “v” is the speed of the
stream
1
Speed of Boat in Still Water = (Downstream Speed + Upstream Speed)
2
1
Speed of Stream = (Downstream Speed – Upstream Speed)
2
Average Speed of Boat = {(Upstream Speed × Downstream Speed) / Boat’s Speed in Still Water}
Distance = Up stream speed × time
Distance = Downstream speed × time

Ratio proportion and mixture


AbycvZ, mgvbycvZ I wgkÖY
I. wgkÖY : GB c×wZ‡Z cÖ`Ë g~‡j¨i `yB ev Z‡ZvwaK Dcv`vb †hB Abycv‡Z wgwk‡q bZzb g~‡j¨i wgkÖY †ewk g~‡j¨i GK
GKK cwigv‡Yi ˆZwi Kiv nq, †mwU wbY©q Kiv nq|
II. ga¨g~j¨ : wgkÖ‡Yi GK GKK cwigv‡Yi µqg~j¨‡K ga¨g~j¨ (Mean Price) e‡j|
Ratio: Comparison of two quantities
Proportion: Educational expression of two ratios
a b c
Proportional equation: = =
b c d
If the gradients are mixed in a ratio, then
Quantity of the cheaper C.P. of the dearer  Mean price
=
Quantity of the dearer Mean price  C.P. of the chaper
We represent it as under:
[CAREER AID FOR BANK AND BCS] Written Math

C.P. of unit quantity of cheaper (c) C. P. of unit quantity of


dearer

Mean Price

(dm) (mc)
 quantity removed each timen
Final Solution = Initial Solution 1  
 initial total solution 

Partnership
Askx`vwiZ¡
I. Askx`vwiZ¡ : hLb `yB ev Zvi †ewk †jvK GKmv‡_ GKwU e¨emv Pvjvq, Zv‡`i Askx`vi e‡j Ges Zv‡`i e¨emv‡K
Askx`vwiZ¡ e‡j|
II. jv‡fi fv‡Mi AbycvZ :
a. mvaviY Askx`vwiZ¡ : hLb Askx`vi mgvb mg‡qi Rb¨ A_© wewb‡qvM K‡i, Zv‡K mvaviY Askx`vwiZ¡ e‡j| GB
Askx`vwi‡Z¡, jvf ev ÿwZ, Zv‡`i wewb‡qv‡Mi AbycvZ Abymv‡i Askx`vi‡`i g‡a¨ fvM Kiv nq|
g‡b Ki, A Ges B †Kv‡bv e¨emvq GK eQ‡ii Rb¨ h_vµ‡g x UvKv Ges y UvKv wewb‡qvM Kij, Zvn‡j eQ‡ii
†k‡l| (A Gi jv‡fi fvM) : (B Gi jv‡fi fvM) = x : y
b. wgkÖ Askx`vwiZ¡ : hLb Askx`viei wewfbœ mg‡qi Rb¨ A_© wewb‡qvM K‡i, Zv‡K wgkÖ Askx`vwiZ¡ e‡j|
GB Askx`vwi‡Z¡, GKwU mg‡qi GK‡Ki wnmv‡e mgZzj¨ A_© wbY©q Kiv nq (g~jab x mg‡qi GK‡Ki msL¨v) a‡i|
GLb, jvf ev ÿwZ GB g~ja‡bi Abycv‡Z fvM Kiv nq|
g‡b Ki A, x UvKv p gv‡mi Rb¨ Ges B, y UvKv q gv‡mi Rb¨ wewb‡qvM Kij, Zvn‡j (A Gi jv‡fi fvM) : (B Gi
jv‡fi fvM) = xp : yq
III. mwµq Ges wbw®Œq Askx`vi : †hB Askx`vi e¨emv cwiPvjbv K‡i Zv‡K mwµq Askx`vi Ges †h ïaygvÎ g~jab wewb‡qvM
K‡i Zv‡K wbw®Œq Askx`vi e‡j|
Master formula
P1: P2 = C1 × T1: C2 × T2
P1 = Partner 1’s Profit.
C1 = Partner 1’s Capital.
T1 = Time period for which Partner 1 contributed his capital.
P2 = Partner 2’s Profit.
C2 = Partner 2’s Capital.
T2 = Time period for which Partner 2 contributed his capital.
[CAREER AID FOR BANK AND BCS] Written Math

Mensuration
cwiwgwZ
েকাণ (Acute angle) : GK mg‡KvY (900) A‡c¶v †QvU †KvY‡K m~¶‡KvY e‡j|

<90
a
সমেকাণ (Right angle): GKwU mij †iLvi Dci Ab¨ GKwU j¤^ Uvb‡j Ges j‡¤^i `ycv‡k Aew¯’Z f‚wg msjMœ †KvY `ywU mgvb n‡j,
cÖwZwU †KvY‡K mg‡KvY e‡j| GK mg‡KvY = 900

90

¯’~j‡KvY (Obtuse angle): GK mg‡KvY A‡c¶v eo weš‘ `yB mg‡KvY A‡c¶v †QvU †KvY‡K m_~j‡KvY e‡j|

>90
b
েকাণ (Reflex angle): `yB mg‡KvY A‡c¶v eo wKš‘ Pvi mg‡KvY A‡c¶v †QvU †KvY‡K cÖe× †KvY e‡j| A_©vr 360°>K>180°
n‡j K GKwU cÖe„ׇKvY|
>180
<360

সরলেকাণ (Straight angle): `ywU mij †iLv ci¯ci m¤ú~Y© wecixZ w`†K Mgb Ki†j †iLvwUi `yÕcv†k †h †KvY Drcbœ nq Zv†K mij†KvY
e†j| mij†KvY`yB mg†Kv†Yi mgvb ev 180°

c
180
িব তীপ কাণ (Vertically Opposite angle): `ywU mij †iLv ci¯ci †Q` Ki‡j †h PviwU †KvY Drcbœ nq G‡`i †h †Kvb GKwU‡K
Zvi wecixZ †Kv‡Yi wecÖZxc †KvY e‡j|
স রক কাণ (Supplementary angle): কােণর সমি 1800 বা ই সমেকাণ হেল এক েক অপর র স রক কাণ বেল।
রক কাণ (Complementary angle): কােণর সমি এক সমেকাণ বা 900 হেল এক েক অপর র রক কাণ বেল।
একা র কাণ(Alternate angles): সমা রাল রখােক অপর এক রখা িতযকভােব ছদ করেল ছদক রখার িবপরীত পােশ সমা রাল
রখা য কাণ উৎপ কের তােক একা র কাণ বেল। একা র কাণ েলা পর র সমান হয়।
Abyiƒc †KvY Similar/ Corresponding angles): `ywU mgvšÍivj mij †iLv‡K Aci GKwU mij †iLv †Q` Ki‡j †Q`‡Ki GKB cv‡k
†h †KvY Drcbœ nq Z‡K Abyiƒc †KvY e‡j| Abyiƒc †KvY¸‡jv ci¯ci mgvb nq|
সি িহত কাণ(Adjacent angle): যিদ কােণর এক সাধারণ বা থােক তেব এক কােণর অপর কােণর সি িহত কাণ বেল।
wÎfzR (Triangle): wZbwU mij‡iLv Øviv mxgve× †¶Î‡K wÎfzR e‡j|
m~²‡KvYx wÎfzR (Acute angle triangle): ‡h wÎfz‡Ri wZbwU †KvYB GK mg‡KvY Gi †QvU Zv‡K m~²‡KvYx wÎfzR e‡j|
¯’‚j‡KvYx wÎfzR (Obtuse angled triangle): ‡h wÎfz‡Ri GKwU †KvY ¯’‚j‡KvY ev GK mg‡KvY A‡c¶v eo Zv‡K ¯’‚j‡KvYx wÎfzR e‡j|
†Kvb wÎfz‡Ri G‡Ki AwaK ¯’‚j‡KvY _vK‡Z cv‡i bv|
[CAREER AID FOR BANK AND BCS] Written Math

mg‡KvYx wÎfzR (Right angled triangle) : ‡h wÎfz‡Ri GKwU †KvY mg‡KvY Zv‡K mg‡KvYx wÎfzR e‡j| †Kvb wÎfz‡R GKwUi AwaK
mg‡KvY _vK‡Z cv‡i bv| mg‡KvYx wÎfz‡Ri mg‡Kv‡Yi wecixZ evû‡K AwZf‚R Ges mg‡KvY msjMœ evû؇qi GKwU‡K f‚wg Ges AciwU‡K
j¤^ ejv nq|
Also read
Equilateral triangle: mgevû wÎfzR
Isosceles angle: mgwØevû wÎfzR
Scalene angle: welgevû wÎfzR
Equiangular triangles: স শেকাণী ি জ
ল িব বা ল েক (Orthocenter): ি েজর শীষ হেত িবপরীত বা র উপর অি ত ল িতন য িব েত িমিলত হ সই িব েক
ি েজর ল িব বলা হ । অ ভােব ি েজর উ তা রখা েলার ছদিব ই হল ল িব ।
পিরেক (Circumcenter): পির ে র ক ( য িব ি েজর শীষ থেক সম রে অবি ত)।
অ ঃেক (In center): ি েজর কাণ ে র সমি খ ক রখা িতন য িব েত িমিলত হ সই িব ই ি জ র অ ঃেক । ি েজর
অভ ের অি ত েক অ ঃ বলা হ । অ ঃ ে র ক ই উে িখত ি েজর অ ঃেক ।
ভরেক (Centroid) : ি েজর কাb এক শীষিব এবং তার িবপরীত বা র ম িব র সংেযাজক সরলেরখােক ম মা বেল। ি েজর
ম মা য় সমিব । Gই িব wU ি েজর ভরেক ।
পির (Circumscribed Circle): পির হেলা এমন এক যা ি েজর িতন কৗিণক িব িদে গমন কের। এ ে ি জ
ে র মে অ িলিখত অব া থােক।
চ জ (Quadrilateral ): চার রখাংশ িদে সীমাব সরলৈরিখক ে র সীমােরখােক চ জ বেল।
িবক সং া: চার রখাংশ িদে আব িচ েক চ জ বেল।
কণ (Diagonal): চ েজর িবপরীত শীষ িব েলার িদেয় তির রখাংশেক কণ বেল। চ েজর কণ েয়র সমি তার পিরসীমার চেয় কম।
চ েজর বিশ : চার বা , চার কান, অ বত চার কােনর সমি ৩৬০0।
সামা িরক (Parallelogram): য চ েজর িবপরীত বা েলা সমান ও সমা রাল এবং িবপরীত কাণ েলা সমান তােক সামা িরক বেল।
আ তে (Rectangle): য চ েজর িবপরীত বা েলা সমান ও সমা রাল এবং িত কাণ সমেকাণ, তােক আ তে বেল।
বগে (Square): বগে বলেত ৪ সমান বা বা জ িবিশ ব জ, তথা চ জেক বাঝা , যার েত ক অ ঃ কাণ এক সমেকাণ
বা ন ই িড ীর সমান । আয়তে ে র সি িহত বা সমান হেল তােক বগ বেল ।
র স (Rhombus): র স এক ধরেনর সামা িরক যার সব িল বা সমান িক কাণ েলা সমেকান নয়।
ািপিজয়াম (Trapezium): য চ জ এর ই বা সমা রাল িক অসমান তােক UªvwcwRqvg e‡j|
ছদক (Intersecting line/ Transversal): য সরলেরখা ই বা তেতািধক সরলেরখােক ছদ কের, তােক ছদক বেল ।
সবসম (Congruent): ই সবসম হেব যিদ এক অ র সােথ সবেতাভােব িমেল যায় । সবসম বলেত আকার ও আ িত সমান
ঝায় ।
শক (Tangent): এক ও এক সরলেরখার যিদ এক ও কবল ছদিব থােক তেবেরখা েক র এক শক বলা হয় ।
আয়তাকার ঘনব (Rectangularcuboids): য সকল ব বা পদাথ িক টা ান দখল কের থােক এবং যার দঘ , এবং উ তা আেছ
তােক ঘনব বেল।
য ব র দঘ, , উ তা আেছ এবং ৬ তল িবিশ তােক ঘন ব বেল ।
ি মাি ক ব ই ঘনব । আর ি মাি ক ব হেলা ি মাি ক জগত -এর ব । ি মাি ক ব বলেত ঝা য ব র অব ান িনণ করার জ
িতন মা া অথাৎ দঘ , ও উ তার ে াজন হ ।
তাই বলা হ , দঘ , ও উ তার সাহাে য ব র অব ান িনণ করা হ , তােক ঘনব বেল।
ঘনক(Cube) : আয়তাকার ঘনব র দঘ , ও উ তা সমান হেল, তােক ঘনক বেল । ঘনেকর বা = ৮
সম িমক কাণক (Right circular cone) : কান সমেকাণী ি েজ সমেকাণ সংল য কান এক বা েক ি র রেখ ঐ বা র চ িদেক
ি জ েক রােল য ঘনব উৎপ হয় তােক সম িমক কাণক বেল ।
িসিল ার বা ব ন (Cylinder): এক আয়তে ে র য কান এক বা েক ি র রেখ ঐ বা র চ িদেক আয়তে েক রােল য ঘনব
উৎপ হয় তােক িসিল ার বা ব ন বেল ।
Answer in one word
১/ এক প েজর সমি  ৬ সমেকাণ
[CAREER AID FOR BANK AND BCS] Written Math

২/ এক ষম ষ েজর অ : কাণ েলার সমি  ৭২০ িডি


৩/ ে র াস িতন ন ি পেল ফল ি পা  ৯ ন
৪/ কান ি েজর বা েলার ল ি খ য িব েত ছদ কের তােক বেল  অ : ক ।
৫/ শিব গামী াসাধ এবং শেকর অ কাণ  ৯০ িড ী।
৬/ িতন কাণ দও া থাকেল য সকল ি জ কা যা তােদর বেল  স শ ি জ
৭/ ি েজর য কােনা বা েক উভ িদেক বিধত করেল উৎপ বিহঃ কাণ ে র সমি  ই সমেকাণ অেপ া হ ম
৮/ কান ি েজর এক বা উভ িদেক বিধত করা উৎপ বিহঃ কাণ িল সমান হেল, ি জ  সমি বা
৯/ ২৫৩ িডি কাণেক কী কাণ বেল  কাণ
১০/ এক সরলেরখার সােথ আর এক রখাংশ িমিলত হে য , সি িহত কাণ উৎপ হ তােদর সমি  ১৮০ িডি ।
১১/ জ া' শে র A_©  িম ।
১২/ সি িহত কােণর সমি ই সমেকাণ হেল এক েক অপর র  স রক কাণ বেল ।
১৩/ এক সরলেরখার সােথ অপর এক রখাংশ িমিলত য সি িহত কাণ উৎপ হ , তােদর সমি হেব - ই সমেকাণ|
১৪/ রক কােণর সমি : ৯০°
১5/ স রক কােণর মান: ১৮০°
১6/ কান ি েজর িতন বা েক বিধত করেল উৎপ বিহঃ কাণ িতন র সমি : ৩৬০°
১7/ সাম িরেকর িবপরীত কােণর অ ি খ ক  পর র সমা রাল|
১8/ এক বগে ে র এক বা অপর এক বগে ে র পিরসীমার সমান হেল, বগে র কেণর অ পাত  ৪:১|
19/ কান এক সমেকাণী ি েজর অিত েজর উপর অি ত বগে ে র ফল ঐ ি েজর অপর ই বা র উপর অি ত বগে ে র
ফেলর সমি র সমান। (cx_v‡Mviv‡mi m~Î)
Angle and triangle
1
Area for triangle = × Base × Height sq units
2
Perimeter of a Triangle = Sum of three sides
3 2
Area for equilateral triangle = a sq units [equal sides = a]
4
3
Height of the equilateral triangle = a unit
2
Perimeter of an Equilateral Triangle = 3a units
b
Area for isosceles triangle= 4a2  b2 sq units [equal sides =a]
4
Perimeter of an isosceles Triangle = 2a+b units
1
Area of a isosceles triangle =  a × b sinθ sq units
2
Area for scales triangle = S(S  a) (S  b) (S  c) [Semi Perimeter = S, and a, b, c are the sides of the
triangle]
Perimeter of scales triangle = a + b + c units
a+b+c
The semi-perimeter = units
2
1
Area for right angle triangle = × Base ×Height sq units
2
Perimeter of a right angle Triangle = Sum of three sides = a + b + c units
[CAREER AID FOR BANK AND BCS] Written Math

Rectangle
The perimeter of a Rectangle, P = 2(l + b) units
Area of a Rectangle, A = l × b sq units
Diagonal of a Rectangle = a2 + b2 units
Bricks/Cuboids
Total Surface Area of cuboids = 2(lb + bh+lh) sq units
Lateral surface area of cuboids: 2h (l +b) sq units
Volume of the cuboids = (l × b × h) cubic units
Perimeter of a cuboids = 4 (l + b + h) units
Diagonal of the cuboids =√ (l2 + b2 +h2) units
Area to be four wall = 2(l+b) ×h sq units
Square
One side of a square = a units Area of a square = a2 sq units
Diagonal of a square = a 2 units Large diagonal of a square = a 3 units
Rhombus
1
Area of rhombus = × D1 ×D2 [Here, D1, D2= Diagonal]
2
Area of rhombus = base × height sq units
The perimeter of a rhombus = 4a units
The opposite angles of a rhombus are equal to each other. Also, the diagonals of a rhombus
bisect these angles.
The basic properties of the rhombus are:
1. The opposite angles are congruent. (me©mg)
2. The diagonals intersect (wef³ Kiv) each other at 90 degrees.
3. The diagonals bisect (w`¦LwÛZKiv) the opposite interior angles.
4. The adjacent angles are supplementary (m¤ú~iK)
Kite
1
Area of Kite = x D1 xD2 [Here, D1, D2= Diagonal]
2
Circle
Radius of a circle = r units Diameter of a circle, D = 2r units
Circumference of a circle = 2πr units Area of a circle = πr2 sq units
Largest cord of a circle is its diameter
সং া িক ত
(Circle): এক িব েক ক কের সমান র বজায় রেখ অপর এক িব তার চারিদেক একবার ের এেল য তরী
হয় তােক বেল।
পিরিধ (Circumference): ণ ব েরখার দঘ েক বলা হয় পিরিধ। ে র পিরিধ = 2πr
এখােন পাই হেলা পিরিধ এবং ােসর অ পাত। এর মান ২২/৭
চাপ (Arc): পিরিধর য কান অংশেক বলা হয় চাপ।
জ া (Chord): পিরিধর য কান ই িব র সংেযাজক সরলেরখােক জ া বেল। উে ে র াস হে হ ম জ া।
[CAREER AID FOR BANK AND BCS] Written Math

াস (Diameter): ে র কান জ া যিদ ক িদেয় যায় তেব জ া েক ে র াস বেল। অথাৎ, ক গামী জ াই হেলা াস।
াসাধ (Radius): ে র ক থেক পিরিধর উপর কান িব র র েক ঐ ে র াসাধ বেল।
িবেশষ : ে র হ ম জ া ই হে ঐ ে র াস এবং এ াসােধর ি ণ। অথাৎ, াসাধ ােসর অেধক।
e„‡Ëi e¨vm 3 ¸Y e„w× Ki‡j Gi †¶Îdj 9 ¸Y e„w× cvq|
e„‡Ëi GKB Pv‡ci Dci `Ûvqgvb †K›`ª¯’ †KvY e„˯’ †Kv‡Yi wظY A_ev e„˯’ †KvY †K›`ª¯’ †Kv‡Yi A‡a©K|
e„‡Ëi †K›`ª n‡Z mg `~ieZ©x mKj R¨v ci¯ci mgvb|
e„‡Ëi e¨vm wfbœ Ab¨ †Kvb R¨vi ga¨we›`y Ges †K‡›`ªi ms‡hvRK †iLvsk H R¨vGi Dci j¤^|
†K›`ª †_‡K †h‡Kvb mij‡iLv e„‡Ëi gvÎ GKwU we›`y‡Z †Q` K‡i|
¯ck©we›`y †Z ¯ck©‡Ki Ici Aw¼Z j¤^ †K›`ª Mvgx
e„‡Ëi †h‡Kvb we›`y‡Z Aw¼Z ¯ck©K ¯ck© we›`yMvgx e¨vmv‡a©i Dci j¤^|
`ywU e„Ë ci¯ci‡K ewnt¯ck© Ki‡j, †K›`ªØ‡qi `~iZ¡ e„Ë؇qi e¨vmv‡a©i mgwói mgvb n‡e
`ywU e„Ë ci¯ci‡K AšZ¯ck© Ki‡j, †K›`ªØ‡qi `yiZ¡ e„Ë؇qi e¨vmv‡a©i AšZ‡ii mgvb n‡e|
Sector( কলা)

Area of a sector = πr2 sq units
360

The area of a circle = πr2 sq units Length of an arc =  2πr units
360
Circumference of a circle = 2πr units
Cylinder ( বলন)
Lateral Surface Area or curved surface area (eµZ‡jii †ÿÎdj) = 2πrh sq units
Total Surface Area = Curved surface (eµZ‡jii †ÿÎdj) + Area of Circular base (f~wgi †ÿÎdj) = 2πrh
+ 2πr2 sq units
Volume (AvqZb) = πr2h cu units
Prism

h h
l
b
a
b
Rectangular Prism Triangular Prism

The surface (c„ôZj) area of a


prism = (2×basearea) + lateral
surface area = 2h (l +b) sq units
h h The volume of a prism = base
area × height sq units
a
TSA (mgMÖZj) of a prism = (2
a ×base area + perimeter of the
b
b base ×height) sq units
Pentagonal Prism Hexagonal Prism
[CAREER AID FOR BANK AND BCS] Written Math

Rectangular Prism
A Rectangular Prism has 2 parallel rectangular bases and 4 rectangular faces.
Base Area = b × l sq units
Surface area = 2 × (bl + lh + hb) sq units
Volume = l × b × h cubic units
Triangular Prism
A triangular prism has 3 rectangular faces and 2 parallel triangular bases.
1
Base area =2 ab sq units
Surface area = ab+3bh sq units
1
Volume = abh cubic units
2
Cone
Slant Height (‡njv‡bv Zj/D”PZv), l = √ (r2+h2) units
1
Volume = πr2h cubic units
3
Curved surface area (eµZ‡jii †ÿÎdj) = πrl sq units
The total surface area of the cone (mgMÖ Z‡jii †ÿÎdj, TSA) = πrl + πr2 sq units
Or, Area = πr(l + r) sq unit
r = refers to the radius of the circular base
l = refers to the slant height of the cone
h = is the height of the cone
Polygon (ব জ)
 Sum of interior angle = (n – 2) × 1800 [n, evûi msL¨v]
(n  2)
 Each Interior(অ ঃ ) angle of a polygon = × 1800 [n , বা র সং া]
n
 Sum of exterior angle = 3600
360
 Each exterior(বিহঃ ) angle of a polygon = [n , বা র সং া]
n
n  (n  3)
 Number of diagonals(কণ)© in any polygon = [n, বা র সং া ≥ 4]
2
n 180
 Area of a polygon (প জ)(given the length) = s2 cot  
4  n 
Where, n = no of sides
s = length of the sides
Perimeter of a polygon = number
360
r2nsin
2
 Area of r polygon (given the radius) =
2
Where, r is the radius (circum-radius)
n is the number of sides
[CAREER AID FOR BANK AND BCS] Written Math

SET
‡mU
Formulas for two sets
Group 1 + Group 2 – Both + Neither = Total
Formulas for three sets
Let’s call our sets A, B, and C.
If n = intersection and u = union. Here are the need-to-know formulas:

For 3 sets A, B, and C P(A  B  C) = P(A) + P(B) + P(C) – {P(A  B) + P(A  C) + P(B  C)} +
P(A  B  C) +Neither
xactly one set P(A) + P(B) + P(C) – 2P(A  B) – 2P(A  C) – 2P(B  C) + 3P(A  B  C)
xactly two of the sets P (A  B) + P (A  C) + P (B  C) – 3P (A  B  C)
= P (A) + P(B) + P(C) + P(A  B  C) – 3P(A  B  C) - P(A  B  C)
= P(A) + P(B) + P(C) - 2P(A  B  C) - P(A  B  C)
= all single – 2×all three - total= all single – 2×all three - total
xactly three of the sets P(A  B  C) = P(A  B  C) - P(A) - P(B) + P(C) + {P(A  B) + P(A 
C) + P(B  C)}
wo or more sets (at P(A  B) + P(A  C) + P(B  C) – 2P(A  n B  C)
least 2 sets)

WATCH
Nwo
GKwU Nwoi AvK„wZ e„ËvKvi hvi cwiwa 60 wU mgvb As‡k wef³ n‡q‡Q, G‡K wgwbU e‡j| GKwU Nwoi `ywU KvUv, †Qv‡UvwU‡K
ejv nq NÈvi KvUv Ges eo KvUvwU‡K ejv nq wgwb‡Ui KvUv|
i. cÖ‡Z¨K NÈvq `y‡Uv KvUv ci¯úi ci¯ú‡ii mv‡_ GKevi wgwjZ nq|
ii. hLb `ywU KvUv mg‡Kv‡Y _v‡K Zv‡`i `~iZ¡wU nq 15 wgwb‡Ui|
iii. hLb `ywU KvUv wecixZ w`‡K _v‡K ZLb Zviv 30 wgwbU `~‡i _v‡K|
iv. 12 NÈvq NÈvq KvUv Øviv Drcbœ †KvY = 360
v. 60 wgwb‡U wgwb‡Ui KvUv Øviv Drcbœ †KvY = 360
A Clock is a circular device provided with three hands viz. an hour hand, minute and second
hand. The study of the clock is known as “horology”.
 20 minutes past 10 gv‡b n‡jv 10Uv †e‡R 20wgwbU ev 10:20 Uv|
 20 minutes to 10 gv‡b n‡jv 10 Uv evR‡Z 20 wgwbU evwK, ev 9:40 Uv|
360
 Speed of a minute hand =
60 minutes
 Speed of a minute hand = 6° per minute.
1
 Angle between hour and minute hand of a clock = │11M – 60H │
2
[CAREER AID FOR BANK AND BCS] Written Math

Question: A clock seen through a mirror, shows quarter past three what is the correct time
shown by the clock?
Solution: Quarter past three A_© n‡jv Nwo‡Z 3 Uv †e‡R 15 wgwbU| Avqbvq NwowU 3:15 wgwbU ‡`Lv‡j cÖK…Z mgq
n‡e 11:60 ‒ 3:15 = 8:45 wgwbU
Awael © (Leap Year) :
i. cÖwZwU eQi hv 4 Øviv wefvR¨ †mwU n‡jv Awael©, Z‡e †mwU kZvãx n‡j GB wbqg cÖ‡hvR¨ bq|
ii. cÖwZwU PZz_© kZvãx n‡jv Awael©|
[we. `ª.: cÖwZwU Awael© 366 w`‡bi|]
D`vniY:
i. 1948, 2004, 1676 BZ¨vw` Awael©|
ii. 400, 800, 1200, 1600, 2000 cÖf…wZ Awael©
iii. 2001, 2002, 2003, 2005, 1800, 2100 cÖf…wZ Awael© bq|
g‡b ivLybt
How many times in a day, the hands of a clock are straight? 44 times
How many times are the hands of a clock at right angle in a day? 44 times
How many times do the hands of a clock coincide in a day? 22 times
How many times in a day, are the hands of a clock in straight line but opposite in direction? 22
times
SERIES
aviv
Arithmetic Sequences
A sequence in which every term is created by adding or subtracting a definite number to the
preceding number is an arithmetic sequence.
Geometric Sequences
A sequence in which every term is obtained by multiplying or dividing a definite number with
the preceding number is known as a geometric sequence.
Harmonic Sequences
A series of numbers is said to be in harmonic sequence if the reciprocals of all the elements of
the sequence form an arithmetic sequence.
Fibonacci Numbers
Fibonacci numbers form an interesting sequence of numbers in which each element is obtained
by adding two preceding elements and the sequence starts with 0 and 1. Sequence is defined as,
F0 = 0 and F1 = 1 and Fn = Fn-1 + Fn-2
Arithmetic Progression Geometric Progression
Sequence a, a+d, a+2d,……,a+(n-1)d,…. a, ar, ar2,….,ar(n-1),…
Common Difference or Common difference = d = second Successive term/Preceding
Ratio term – first term term
Common ratio = r = ar(n-1)/ar(n-2)
General Term (nth an = a + (n-1)d an = ar(n-1)
[CAREER AID FOR BANK AND BCS] Written Math

Term)
nth term from the last an = l – (n-1)d an = 1/r(n-1)
term
Sum of first n terms n a (1  r n )
sn = [2a + (n-1)d] sn = if r < 1
2
1 r
*Here, a = first term, d =
common difference, r = common a (r n  1)
sn = if r > 1
ratio, n = position of term, l = last r 1
term

n(n + 1)
Sum of nth term =
2
n(n + 1)(2n + 1)
Sum of nth square term =
6
n(n + 1)2
Sum of nth cube term =  
 2 
2
Sum of nth odd number = n

COORDINATE GEOMETRY
¯’vbv¼ R¨vwgwZ
All Formulas of Coordinate Geometry
Tile of the formula Formula
General Form of a Line Ax + By + C = 0
Slope (ঢাল) Intercept Form of a Line y = mx + c [ m is the slope of the line]
Point-Slope Form y – y1= m(x − x1)
The slope of a Line using y y2 -y2
m= = [y = †KvwU ؇qi AšÍi, x = f~R ؇qi AšÍi]
coordinates x x2 -x2
y – y 1 x – x1
General equation of a straight Line =
y 1 – y 2 x1 – x2
Intercept-Intercept Form x y
[x I yAÿ †Q`Kvix †iLvi mgxKiY] + =1
a b
|P1P2| =
Distance Formula(`ywU we›`yi `~iZ¡)
(x2 - x1)2 + (y2 - y1)2
For Parallel (mgvšÍivj) Lines, m1 = m2
[CAREER AID FOR BANK AND BCS] Written Math

For Perpendicular(jg¦) Lines, m1×m2 = -1


Equation: ax+by+c = 0 b
Or, by = - ax –c Perpendicular equation: bx – ay +k = 0, m2 =
a
a c Proof: m1×m2 = -1
Or, y = -  -
b b a b
  =1
a b a
m1 = - b

Midpoint (ga¨we›`y) Formula M (x, y) = [½(x1 + x2), ½(y1 + y2)]


m1 -m2
Angle Formula tan θ =
1 + m1m2

Area of a Triangle Formula ½ ×|x1(y2 − y3) +x2 (y3 – y1)+x3(y1–y2)|

Ax0 + By0 + C
Distance from a Point to a Line d=| |
A 2 + B2

Trigonometry
w·KvbwgwZ
A

B C
ABC mg‡KvYx wÎfy‡R Formula
Height Sinθ
j¤^ (Height/Perpendicular) = AB tan  = =
Base Cosθ
AwZfzR (Hypotenuse) = AC Base 1
cos  = =
f‚wg (Base/Adjacent) = BC Hypotenuse Secθ
mg‡KvYx wÎfy‡R, †Kv‡Yi wecixZ evûwU j¤^| Height 1
sin  = Hypotenuse = Cosecθ
mg‡KvYx wÎfy‡R, mg‡Kv‡Yi wecixZ evûwU AwZfzR|
Sin2  + Cos2  = 1
Sec2   tan2  = 1
Cosec2   Cot2  = 1

Relation between side and angel of right angle triangle


[CAREER AID FOR BANK AND BCS] Written Math

Base Height Hypotenuse


Side 1 1 2
Angle 45 45 90

Base Height Hypotenuse


Side 1 3 2
Angle 30 60 90

Base Height Hypotenuse


Side 5 12 13
Angle 22.62 67.38 90
sin (90 - ) = cos , i.e. sin 40 = cos 50
cos (90- ) = sin , i.e. cos 15 = sin 75
tan (90 - ) = cot , t.e tan 30 = cot 60
sin ( - ) = -sin 
cos ( - ) = cos 
tan ( - ) = -tan 
Pythagorean triplet Coordinate position
All students take care
3, 4, 5
5, 12, 13 First = All positive
7, 24, 25 Second = Sin & Cosec (+ve)
8, 15, 17
1, 1, √2 Third = tan & cot (+ve)
1, √3, 2 Fourth = Cos & Sec (+ve)

Trigonometric values
Degree Radians Sinθ Cosθ tanθ cotθ secθ cosecθ
0 0 0 1 0 unefined 1 undefined
30  1 3 1 3 2 2
6 2 2 3 3
45  1 1 1 1 2 2
4 2 2
60  3 1 3 1 2 2
3 2 2 3 3
90  1 0 unndefined 0 undefined 1
2
180 π 0 -1 0 undefined -1 undefined
360 2π 0 1 0 undefined 1 undefined

Permutation and Combination


[CAREER AID FOR BANK AND BCS] Written Math

web¨vm I mgv‡ek
†MŠwbK (Factrotial) n: awi n, n‡jv GKwU abvZ¥K c~Y©msL¨v †MŠwYK n †K A_ev n Øviv cÖKvk Kiv nq, n A_ev n!
wn‡m‡e e¨vL¨v Kiv nq|
n = n(n  1) (n  2) (n  3) .... 4.3.2.1.
D`vniY:
i. 4 = (4  3  2  1) = 24
ii. 5 = (5 = (5  4  3  2  1) = 120
web¨vm (Permutation): GKwU cÖ`Ë msL¨vi wKQz A_ev m¤ú~Y© Ask GKmv‡_ wb‡q wfbœfv‡e mvRv‡bv‡K web¨vm e‡j|
D`vniY: 1, a, b, c Aÿi¸wji g‡a¨ GKB mv‡_ `ywU wb‡q ˆZwi web¨vm n‡jv (ab, ba, ac, ca, bc, cb) |
D`vniY: 2, a, b, c Aÿi¸wji g‡a¨ GKBmv‡_ wZbwU wb‡q ˆZwi web¨vm n‡jv (abc, acb, bac, bca, cab, cba) |
n!
ii. n msL¨K e¯‘ †_‡K r msL¨K e¯‘ †_‡K Avgiv cvB, nPr = n(n  1) (n  2)....(n  r + 1) =
(n  r)!
D`vniY:
i. 8P2 = (8  7) = 56.
ii. 7P3 = (7P3 = (7  6  5) = 210.
mgv‡ek (Combination): KZ¸‡jv wRwbm n‡Z K‡qKwU ev, meKqwU hZ cÖKv‡i evQvB Kiv hvq, Zv‡`i cÖ‡Z¨KwU‡K
GK GKwU mgv‡ek e‡j|
n n! 1 n! 1
Cr = = × = × nPr
r!(n  r)! rl (n  r)! r!
Number of combinations = n = GLv‡b n n‡”Q KZRb gvbyl ev KZwU `j Ask wb‡”Q|
Taken way (Fixed) = taken time = r = K‡Zv Dcv‡q †e‡Q wb‡Z n‡e|
PROBABILITY
m¤¢vebv
AbyK‚j djvdj (Favourable outcomes)
m¤¢ve¨Zv ev m¤¢vebv(Probability) =
†gvU djvdj (Total outcomes)
m¤¢ve¨Zvi ms‡hvMm~Î:
যিদ A I B `ywU AeR©bkxj/¯^vaxb NUbv nq Z‡e, P (AB) = P (A) +P (B) -P (AB) [Mutually Inclusive Events]
যিদ A I B `ywU eR©bkxj NUbv nq Z‡e, P(AB) = P(A) + P(B) [Mutually Exclusive Events]
m¤¢ve¨Zvi ¸Ybm~Î
যিদ A I B `ywU AeR©bkxj/¯^vaxb NUbv nq Z‡e, P (AB) = P (A)  P (B) [Mutually Inclusive Events]
hw` A I B Aধীন/eR©bkxj ঘটনা হ তেব, P (AB) = 0 [Mutually Exclusive Events]
kZ©vaxb m¤¢ve¨Zv:
hw` A I B ci¯ci eR©bkxj `ywU NUbv nq Ges P(B)>0 হ তেব B ঘটার শতাধীেন A ঘটার স া তা,
P(AB)
P(A/B) =
P(B)
P(AB)
Abyiƒcfv‡e, hw` nq P(A) > 0 Z‡e A NUvi kZ©vax‡b B NUvi m¤¢ve¨Zv, P(B/A) =
P(A)
Some meaning and condition
[CAREER AID FOR BANK AND BCS] Written Math

 At best = m‡e©v”P
 At most = m‡e©v”P
 At least = Kgc‡¶
 Probability of possible events = 1
 Probability of Impossible events = 0
Deck of cards (52)
Red(26) Black(26)
Heart(13) Diamond(1 Club(13) Spade(13) Face cards (12) Honorcards (20)
3)
King King King King King Face cards
Queen Queen Queen Queen Queen Ace cards
Jack Jack Jack Jack Jack 10
Ace Ace Ace Ace
10 10 10 10
9 9 9 9
8 8 8 8
7 7 7 7
6 6 6 6
5 5 5 5 Numeral cards = 9 × 4 = 36
4 4 4 4
3 3 3 3
2 2 2 2
Dice
GK bR‡i Q°v m¤cwK©Z Z_¨ I mgm¨vejxmg~n
1. GKwU evi Q°vwb‡¶c Kivi ci †gvU djvdj = 61= 6
2. `yevi Q°v wb‡¶c Kivi ci ‡gvU djvdj = 62 = 36
3. wZbevi Q°v wb‡¶c Kivi ci †gvU djvdj = 63 = 216
[CAREER AID FOR BANK AND BCS] Writtten math

Algebra

Find the Value


x²-7x+12
1) Solve the equation: = 3, calculate the value of X.
x²+4
Solution
x²-7x+12
=3 Or, x² ‒ 7x + 12 = 3x² + 12
x²+4
Or, x² ‒ 7x + 12 – 3x² ‒ 12 = 0 Or, ‒ 2x² ‒ 7x = 0
Or, 2x² + 7x = 0 Or, x(2x+7) = 0
So, x = 0 or, 2x + 7 = 0
7
⸫ x = 0 or ‒ [Answer]
2
1
2) If x  3 2 the find the value of x 2 
x2
Solution
Given that, x  3 2
1 1
ev, 
x 3 2
1 3 2
ev, 
x  
3 2 3 2 
1 3 2
ev, 
x 3 2
1
  3 2
x
2
1  1 1
Now, 2
x  2  x    2x  =
x  x x
 3 2  3 2 2 
  2
= 2 3  2 = 4.3 2 = 10 Answer: 10
2 2 1 3 2
3) Given: + = and + = 0 Find ‘a’ for which y = ax – 4
x 3y 6 x y
2 2 1 3 2
Solution Here, + = ‒‒‒‒‒‒ (1) + = 0 ‒‒‒‒‒ (2)
x 3y 6 x y
1 3 1 2 1 1 1 2
Now multiplying (2) by we get × + × = 0 × Or, + = 0 ‒‒‒‒‒ (3)
3 x 3 y 3 3 x 3y
2 1 2 2 1
Now, subtracting equation (3) from equation (2) − + − = −0
x x 3y 3y 6
Or, x = 6

31
[CAREER AID FOR BANK AND BCS] Writtten math

2 1
Put the value of x = 6v in equation (3) we get, = − Or, y = − 4
3y 6
1
Now, y = ax – 4 Or, − 4= a×6 – 4 Or, a = ×(4 − 4) = 0
6
Solution a = 0
4) + + = + + = + ?
Solution
Here given, a + a b + b = 3
Or, (a ) + 2a b + (b ) − a b = 3
Or, (a + b ) − (ab) = 3
Or, (a + ab + b )(a − ab + b ) = 3
Or, (a − ab + b ) = = 1 … … … … … . (i)
And Given, a + ab + b = 3 … … … … … . . (ii)
By adding (ii) and (i)
a + ab + b + a − ab + b = 3 + 1 = 4
Or, 2(a + b ) = 4
Or, a + b = = 2 Answer: 2
. Find the value of + ; if = √ − √ [Dhaka Bank‒MTO‒2017]
Give the, x = √5 − √4 –––––––––––– (I)
Now, Reversing Equation (I)
1 1
=
x √5 − √4
√ √
Or =
√ √ √ √
√ √
Or, =
√ √
√ √
Or, =
Or, = √5 + √4 –––––––––––– (II)
{(I) + (II)} We have
x + = √5 − √4 + √5 + √4
Or, x + = √5 − √4 +√5 + √4
.`. x + = 2√5 –––––––––––– (III)
Now,x + = (x ) + = x + − 2x

= x + −2 = (x) + −2

32
[CAREER AID FOR BANK AND BCS] Writtten math

= x+ − 2x −2 = x+ −2 −2

= 2√5 − 2 − 2 = (20 − 2) − 2
= (18) − 2 = 324 − 2 = 322
Answer: 322
4. If + = 1 then find the value of + + ++ + + +5 [Janata
Bank Ltd, AEO Teller – 2020]
Solution
Given that,
+ =1 Or, (x+ ) 2 – 2 × x× = 1
Or, (x+ ) 2 = 3 Or, (x+ ) = √3
Or, (x+ ) 3 =(√3) 3 Or, + + 3× x× (x+ ) = 3√3
Or, + + 3√3 = 3√3
Or, + = 3√3 − 3√3 = 0

Now, + + + + + +5
= ( + )+ ( + )+ ( + ) +5
= ×0+ ×0+ ×0 + 5 = 5 Answer: 5
5. If x = 2 + + , then the value of x3  6x2 + 6x is
Solution
2 1 2 1
Given that, x = 2 + 23 + 23  (x  2) = 23 + 23
 2 13
 (x  2) = 23 + 23 [taking cube]
3 
 23  13 2 1 2 1
 x  3x .2 + 3x(2)  (2) = 23 + 23 + 3. 23.23 23 + 23
3 2 2 3     
2 1  2 1
+
 x  6x + 12x  8 = 2 + 2 + 3. 23 3 (x  2)  (x  2) = 23 + 23
3 2 2 
 x3  6x2 + 12x  8 = 4 + 2 + 3.21 (x  2)
 x3  6x2 + 12x  8 = 6 + 6x  12
 x3  6x2 + 12x  6x =  6 + 8
 x3  6x2 + 6x = 2 Answer: 2
6. Given a = 3 + 2√ , find the value of √ − [Janata Bank (AEO‒RC)‒2017]

Solution
Given that, Now,

33
[CAREER AID FOR BANK AND BCS] Writtten math

1
a = 3 2 2 a
a
Or, a = 3  2 2 +1 = 2 +1   2 1 
Or, a =  2   2.
2
 
2. 1  1
2
= 2 +1  2 +1
a =  2  1
2
Or, = 2(Answer.)
 a  2 1
1
Therefore,  2 1
a
2 x2  x  2
7. If x   1 then find the value of 2 [Rupali So 2019]
x x (1  x)
Solution
2 x2  2
Given that, x  1 ev, 1
x x
ev, x2+2 = x ev, x2x = ‒ 2
ev, x ‒ x2 = 2

x2  x  2 x x
Now, = [ x2+2 = x]
x 1  x  x.x 1  x 
2

2x 2x
= = =1 [Answer]
x x  x  x. 2
2

1 x6 1
2
8. If x  2  7 , the find the value of 3
[40th BCS]
x x
Solution:
Given that, Now,
1 x6 1
x2  2  7
x x3
2
 1 1 1
ev,  x    2 .x.  7 = x3  3
 x x x
2 3
 1  1 1 1
ev, x  9 =  x    3 x.  x  
 x  x x x
1
ev, x 3 = (3)3 3 .3
x
= 27.  9 = 18
Answer: 18

34
[CAREER AID FOR BANK AND BCS] Writtten math

1 1
9. x - = 3 then find the value of x 6  6 [38th BCS]
x x
1
Given that, x - = 3
x
2 2
1  1   1  1
Now, x  6 = x 3
6

x
 2
  3  =  x 3  3   2. x 3 . 3
x  x
x  
3 2
 1 1 1 
=  x    3 x  x     2 =
 x x x  
 3   3 3  2
3 2

 
2
  2
= 3 3  3 3  2 = 6 3  2 = 36.3 + 2 = 108 + 2 = 110
Answer: 110
1 1
10. If x   3 the find the value x 6  6
x x
Solution
2
1  1
Given that, x   3 Or,  x    32
x  x
3
1 1  1 
Or, x 2  2
 2. x.  9 Or,  x 2  2   7 3
x x  x 
1 1 1 
Or, x 6  6  3.x.  x 2  2   743
x x x 
1 1
Or, x 6  6  743  21  x 6  6  322
x x
Answer. 322
1 1
11. If x+ = 3 then find the value x 5  5
x x
Solution
1
Given that, x   3
x
2
1  1 1
We know, x  2   x    2 x = (3)2 2 = 7
2

x  x x
1
 x2  2 7
x
3
1  1  1
Again, x  3   x    3  x   = 333.3 = 279 =18
3

x  x  x

35
[CAREER AID FOR BANK AND BCS] Writtten math

 1  1 
Now,  x 2  2   x 3  3  = 718
 x  x 
1 1 1 1
ev, x 2 .x 3  x 2 3  2  x 3  2 . 2 126
x x x x
1  1
ev, x 5  5   x   126
x  x
1
ev, x 5  5  126  3
x
1
 x 5  5 123 (Answer)
x
Proof
1. − = + = .
Solution
Here, x− =4

Or, x − = 4 [By Squaring both side ]


Or, x + − 2. x. = 16
Or, x + = 18
1
Or, x + = (18)
x
1
Or, x + + 2 = 324
x
1
Or, x + = 324 − 2 = 322[Proved]
x
1 1
2. If a   3 the prove that a 4  4  119 [Rupali SO 2020]
a a
Solution
1
Given that, a   3
a
2
 1 1 1
ev,  a    9 ev, a22. a  2  9
 a a a
1  1 
ev, a2 + 2 11 ev,  a 2  2  = (11)2
a  a 
1 1 1
ev, a4 +2a2 2  4 121 ev, a4 + 4  121  2
a a a

36
[CAREER AID FOR BANK AND BCS] Writtten math

1
ev, a4 + = 119 (Proved)
a4
1 1
If a4 + 4
= 119 then prove that a  3 [Do yourself]
a a
1
3. x² + = 10 n‡j †`LvI †h x = √3 + √2

Solution
1 1 1
Given that, x² + = 10 Or, (x + )2 ‒ 2 ×x× = 10
x² x x
1 1
Or, (x + )2= 10+2 Or, x + = 12 = 2 3 ‒‒‒‒‒‒‒‒ (i)
x x
1 1 1
Again, x² + = 10 Or, (x ‒ )2 + 2 ×x× = 10
x² x x
1 1
Or, (x ‒ )2 = 10 ‒ 2= 8 Or, x ‒ = 8 = 2 2 ‒‒‒‒‒‒‒ (ii)
x x
(i) + (ii) We get

2x = 2√3 + 2√2
1
Or, x = ×2 (√3 + √2)
2
So, x = √3 + √2 [Showed]
Simplification
x 1 4 x
1. Simplify: 2
 2
x  x  20 x  4 x  5
Solution
x 1 4 x
=
x  1x  1 4  x x  4 
 2
2
x  x  20 x  4 x  5 x  5x  1x  4 
x 1 4 x x 2  1 4 2  x 2
= 2  2 =
x  5 x  4 x  20 x  5 x  x  5 x  5x  1x  4
x 1 4 x
=  16  1
x  x  5   4  x  5 x  x  5   1 x  5  =
x  5x  1x  4
x 1 4 x
=  15
x  5x  4  x  5x  1 =
x  5x  4x  1
(Ans)

37
[CAREER AID FOR BANK AND BCS] Writtten math

5x  2 2x 1
2. Simplify 2
 2 [Agrani SO Auditor]
x  x  20 x  4 x  5
Solution
5x  2 2x 1
=
5 x  2 x  1  2 x  1x  4 
 2
2
x  x  20 x  4 x  5 x  5x  4x  1
5x  2 2x 1 5x 2  5 x  2 x  2  2 x 2  x  4  8 x
= 2  2 =
x  5 x  4 x  20 x  5 x  x  5 x  5x  4x  1
5x  2 2x 1
=  7 x 2  14 x  2
x  5x  4  x  5x  1 = (Answer)
x  5x  4x  1

3. HCF : x316x, 2x3+9x2+4x, 2x3+x228x


Solution
First Case Second Case Third Case
x316x 2x3+9x2+4x 2x3+x228x
= x(x216) = x(2x2+9x+4) = x (2x2+x28)
= x (x+4) = x(2x2+8x+x+4) = x (2x2+8x7x28)
(x4) = x(x+4) (2x+1) = x{2x(x+4) 7(x+4)}
= x (x+4) (2x7)
Therefore the HCF is x(x+4) (Answer)
Factorize
1. Factorize x321x+20
Solution
Let, p(x) = x321x+20
Or, p (1) = (1)3 211+20 = 21  21 = 0
So, x = 1 ev, x1 = 0 is one of the factor of p(x)
Now, x321x+20
= x3x2 + x2x 20x+20
= x2. (x1) +x (x1) 20 (x1)
= (x1) (x2+x20)
= (x) {x(x+5) 4 (x+5)}
= (x1) (x4) (x+5) (Answer)
1 2
2. Factorize: a 2  2  2  2a 
a a
Solution
1 2 2 1  1
a 2  2  2  2a  = a  2  2  2 a  
a a a  a

38
[CAREER AID FOR BANK AND BCS] Writtten math

2
 1 1   1
=  a    2a   2  2  a  
 a a   a
2
 1  1
= a    2  2  2a  
 a  a
 1  1 
=  a    a   2.1
 a  a 
 1  1 
=  a    a   2  (Answer.)
 a  a 
x 2  2 xy  y 2
3. If f (x, y) = then find f (x, y) [BEZA AM 2020]
x2  y2
Solution
Given that,

F (x, y) =
x 2  2 xy  y 2

x  y 2

2
x y 2
x  y x  y 
xy
 f (x, y) =
x y
 x   y   x  y  x  y
Now, f (x, y) =  =
 x   y   x  y  x  y 
xy
 f (x,  y) = Answer.
x y

39
[CAREER AID FOR BANK AND BCS] Writtten math

Age
1) The average age of a committee of eight members is 40 years. A member aged
55 years retired and his place was taken by another member aged 39 years. The
average age of the present committee is?
Solution
Difference in age of last person = 55 ‒ 39 = 16
This change will be distributed among all 8 members, hence the average of group
16
reduced by = 2 years
8
So average age of present committee = 40 ‒ 2 = 38 years Answer 38 years
2) Two students appeared at an examination. One of them secured 9 marks more
than the other and his marks were 56% of the sum of their marks. What are the
marks obtained by them?
Solution
Let, One gets x & other One gets (x + 9).
According to the question,
14 14
X + 9 = (x + x + 9)× [ = 0.56]
25 25
Or, 25x + 225 = 28x + 126
Or, 3x = 99
Or, X = 33
Other one gets (x + 9) = 33 + 9 = 42 Answer 42
3) A father told his son, “I was as old as you are at present at the time of your
birth”. If the father’s age is 38 years now, the son’s age five years back was?
Solution
Let the son's present age be x years.
According to the question,
Difference of father's and son's present age = Son's present age
(38 − x) = x
Or, 2x = 38
Or, x = 19
Five years back = (19 − 5) = 14 Answer 14 years
4) A woman says, "If you reverse my own age, the figures represent my
husband's age. He is, of course, senior to me and the difference between our ages
is one‒eleventh of their sum." The woman's age is
Solution
Since the age of wife and husband is a two digit
number
Let, the unit digit of wife age = x
The tenth digit of wife age = y
So the age of wife = 10y + x

40
[CAREER AID FOR BANK AND BCS] Writtten math

The age of Husband = 10x + y


According to the question
11x + 11y
10x + y − 10y ‒ x =
11
Or, 9x − 9y = x + y
Or, 8x = 10y
Or, X: Y= 5:4
So wife’s age = 10×4 + 5 = 45
Husbands age = 10×5 + 4 = 54 Answer 45
5) A person's present age is two‒fifth of the age of his mother. After 8 years, he
will be one‒half of the age of his mother. How old is the mother at present?
Solution
2
Person’s present age = ×Mothers age
5
2
Or, P = M
5
According to the question,
(M + 8) 2
P+8 = (replacing P with M)
2 5
2M
Or, 2× +16 = M+8
5
(4M+80)
Or, = M+8
5
Or, 4M + 80 = 5M + 40
Or, M = 40
So, Mothers age = 40 years. Answer 40 years
6) 3 years ago father age was twice the age of his four sons. In 3 years time s
father will be equal to the sum of ages of four sons. Find the present age of
Father? Abyev`t 3 eQi c~‡e© evevi eqm 4 Rb cy‡Îi eq‡mi wظY wQj| 3 eQ‡i evevi eqm 4 Rb cy‡Îi
eq‡mi mgvb n‡j evevi eZ©gvb eqm KZ?
Solution
Let, 3 years ago father’s age = 2x and Son’s Age = x
In three years time father’s age becomes = 2x + 3 + 3 = 2x + 6
In three years time son’s age becomes = x + 4×6 = x + 24
According to the question,
2x + 6 = x + 24
Or, x = 18
So, the present Age of Father = 2×18+3 = 39 Answer 39
7) Sum of present age of A and B is 60; after 5 years the ratio of their age 3:4,
present age of A is? Abyev`t A Ges B Gi eZ©gvb eq‡mi mgwó 60 eQi, 5 eQi ci Zv‡`i eq‡mi
AbycvZ 3t4 n‡j A Gi eqm KZ?

41
[CAREER AID FOR BANK AND BCS] Writtten math

Solution
Let, after 5 years age of A = 3x and age of B = 4x
Their present age respectively = 3x ‒ 5 and 4x ‒ 5
According to the question,
3x ‒ 5 + 4x ‒ 5 = 60
Or, 7x ‒ 10 = 60
70
Or, x = = 10
7
So, Present age of A = 3×10 ‒ 5 = 30 ‒ 5 = 25 years
Another way
Let, present age of A = x years and present age of B = 60 ‒ x
According to the question,
x+5 3
=
60‒x+5 4
Or, 180 ‒ 3x + 15 = 4x + 20
Or, 7x = 175
175
Or, x = = 25 Answer 25years
7
8) The sum of ages of the Father and Son is 50 years. When the ages of the son
will be equal to the present age of the father, the sum of their ages will be 102
years. Find the age of Father and Son?
Solution
Suppose, the current age of the son is S years and the current age of the father is F
years.
So, S +F = 50 ‒‒‒‒‒‒ (i)
Let, after P years, the age of the son will be equal to the current age of thefather.
So, S+ P = F
And, S+P+F+P = 102
Or, S+F+2P = 102
Or, 2P = 102‒50 = 52
Or, P = 26
So, S+26 = F
Or, F ‒ S = 26 ‒‒‒‒‒‒‒ (ii)
(i)+ (ii) => 2F = 76
Or, F = 38.
So, S+38 = 50
Or, S = 12.
So, the current age of the son is 12 years, and the current age of the father is 38 years.
Another way
Let, Father's age = F
So, Son's age = (50 ‒ F)

42
[CAREER AID FOR BANK AND BCS] Writtten math

Difference of their ages = F ‒ (50 ‒ F) = 2F – 50


According to question,
Son’s age + Father’s age = 102
Or, (F + 2F ‒ 50) + F = 102
Or, 4F = 152
Or, F = 38
So, Age of son (50 ‒ 38) = 12 and Fathers age is 38 years old.
Answer: Age of father’s and son’s are 38 and 12 years old

Practise
1) 6 years ago, Rahim is three times older than his son Robin. Currently the ratio of
their age is 5:2. What is the sum of their age? [DPDC AM 2020]
Answer: 84 years
2) A Mother said to her daughter, "I was as old as you are at the present at the time of
your birth". If the Mother’s age is 38 years now, the daughter’s age five years back
was:
Answer: 14 years
3) The sum of the present ages of a father and his son is 60 years. Six years ago,
father's age was five times the age of the son. After 6 years, son's age will be [Dhaka
Bank MTQ 2021]
Answer: After 6 years, son's age will be 20 years
4) Ayisha's age is 1/6th of her father's age. Ayisha 's father's age will be twice the age
of Shankar's age after 10 years. If Shankar's eight birthdays was celebrated two years
before, then what is Ayisha 's present age.
Answer: 5 years

43
[CAREER AID FOR BANK AND BCS] Writtten math

Average
1) Of the three numbers, second is twice the first and is also thrice the third. If the
average of the three numbers is 44. Then what will be the largest number? [BB OFF 15]
Solution
Let, the fist number is 3x
Second number is 6x
Third number is 2x
According To the Question, = 44
Or, 11x = 44×3
Or, x = 12
So, 6x = 72 Answer: 72
2) The average weight of A, B and C is 40kg. Weight of C is 24 kg more than A's weight
and 3 kg less than B's weight. What will be the average weight of A, B, C and D, if D
weights 15 kg less than C?
Solution
A+ B+C Now, A+B+C = 120
Here, = 40
3 Or, A+A+27+A+24 = 120
Or, A+B+C = 120 Or, 3A+51 = 120
Given, C – A = 24 69
Or, A = = 23
Or, C = A+34 ‒‒‒‒‒‒‒ (1) 3
Again, B – C = 3 From eq (1), C = 23+24 = 47
Or, B = 3 + C From eq (2), B = 47 +3 = 50
Or, B = A+24+3 Given C – D = 15
Or, B = A+27 ‒‒‒‒‒‒‒ (2) Or, D = C – 15 = 47 −15 = 32
A + B + C +D 23 + 50 + 47 +32
Hence, average age of A, B, C & D is =
4 4
152
= = 38 kg [Answer]
4
3) The average weight of A,B,C is 50 kg. C is greater than A by 34 kg is shorter than B
by 3 kg, find out the average weight of A,B,C and D where C is greater than D by 25 kg.
Solution
A+ B+C Now, A+B+C = 150
Here, = 50
3 Or, A+A+37+A+34 = 150
Or, A+B+C = 150 Or, 3A+71 = 150
Given, C –A = 34 79
Or, A =
Or, C = A+34 ‒‒‒‒‒‒‒ (1) 3
Again, B – C = 3 79 181
Or, B = 3+C From eq (1), C = +34 =
3 3
Or, B = A+34+3 181 190
Or, B = A+37 ‒‒‒‒‒‒‒ (2) From eq (2), B = +3=
3 3

Given C – D = 25
181 181 - 25 106
Or, D = C – 25 = ‒ 25 = =
3 3 3

44
[CAREER AID FOR BANK AND BCS] Writtten math

A + B + C +D 150 +D
Hence, average age of A, B, C & D is = =
4 4
106
150 +
3
4
450 +106
= = 46.33 kg
12
So average weight will be 46.33 kg
Answer: 46.33 kg
4) The average weight of three men A, B and C is 84 kg. Another man D joins the
group and the average now becomes 80 kg. If another man E, whose weight is 3
kg more than that of D, replaces A, then the average weight of B, C D and E
becomes 79 kg. What is the weight of A?
Solution:
The average weight of A, B and C = 84kg
The total weight of A, B and C = 84 x 3 = 252Kg.
The average weight of A, B, C and D = 80 kg
The total weight of A, B, C and D = 80 x 4 = 320 kg
The weight of D = 320 ‒ 252 = 68 kg
The weight of E = 68 + 3 = 71 kg
The average weight of B, C, D and E = 79 kg
The total weight of B, C, D and E = 79 x 4 = 316 kg
Weight of A‒ E is = the total weight of A, B, C and D ‒ the total weight of B, C, D
and E = 320 ‒ 316 = 4 kg
Or, A‒E=4
Or, A = 4+E
Or, A = 4 + 71
⸫A = 75 kg
Answer: Weight of A is 75 kg
5) Three math classes: X, Y, and Z, take an algebra test. The average score in class X is
83. The average score in class Y is 76. The average score in class Z is 85. The average
score of all students in classes X and Y together is 79. The average score of all students in
classes Y and Z together is 81.What is the average score for all the three classes, taken
together?
Solution
Here, the average score of class X = 83
The average score of class Y = 76
The average score of class Z = 85
The average score of all students in X and Y together = 79
The average score of all students in Y and Z together = 81
Let, no of students in classes X, Y and Z = x, y and z respectively

45
[CAREER AID FOR BANK AND BCS] Writtten math

83x + 76y 4x
Now, for class X and Y, = 79 Or, 79x + 79y = 83x + 76y Or, 3y = 4x Or, y =
x+y 3
76y + 85z 5y 5 4x 5x
For class Y and Z, = 81 Or, 76y + 85z = 81y + 81z Or, 4z = 5y Or, z = = × =
y+z 4 4 3 3
4x 5x
83x+76× +85×
83x + 76y + 85z 3 3 83×3x + 76×4x + 85×5x
We have to find, = =
x+y+z 4x 5x 3
x+ +
3 3
3 249 + 304 + 425 3 978
× = × = = 81.5
3x+4x+5x 3 12 12
Answer81.5
6) Total distance of 800 meter is to be travelled 3/4 part is travelled at 135 meter per
minute and remaining distance 145 meter per minute. What is the average speed of
whole journey, express approximate value at whole number? [Jibon Bim, AM – 2020, FSIBL,
PO ‒ 2023]
Solution
3
Distance of first part = 800× = 600 m
4
1
Distance of second part = 800× = 200 m
4
600 40
Time to cover first part = = min
135 9
200 40
Time to cover second part = min
145 29
40 40 1 1 38 1520
Total time = + = 40( + ) = 40× = min
9 29 9 29 261 261
Total distance
Average speed =
Total time
800 261
Or, Average speed = = 800× ~ 137 m/min (Answer)
Total time 1520
Another way

Weight 1 = 3
Weight 2 = 1
Variable 1 = 135
Variable 2 = 145
∑(Total variable×weights) 135×3+145×1 550
Average speed = = ≈ 137 m/min
Total weights 3+1 4

46
[CAREER AID FOR BANK AND BCS] Writtten math

Percentage
Basic
1) With reference to a number greater than one, the difference between itself and its
reciprocal is 25% of the sum of itself and its reciprocal. By how much percentage is the
fourth power of the number greater than its square? [Janata Bank PLC, RC – 2024]
Solution
1
Let, the number is x and its reciprocal is x2 ‒ 1
x
1
x‒
x 1 x2 ‒ 1 1
According to the question, = Or, 2 = Or, 4x2 ‒ 4 = x2 + 1
1 4 x +1 4
x+
x
5
Or, 3x2 = 5 Or, x2 =
3
25
Fourth power of the number is, x4 = .
9
25 5

9 3 25 ‒ 15 3 2
Now, ×100 = × ×100 = ×100 = 66.67%
5 9 5 3
3

Answer 66.7%
2) The population of a town is 176400. It increases annually at the rate of 5% p.a. What
will be its population after 2 years? What was it 2 years ago?
Abyev`t GKwU kn‡i 176400 Rb RbmsL¨v i‡q‡Q| cÖwZ eQi 5% nv‡i RbmsL¨v e„w× †c‡j 2 eQi ci I c~‡e© RbmsL¨v
KZ wQj?
Solution
105 105
The population after 2 years = 176400× × = 194,481
100 100
100 100
The population before 2 years = 176400 × × = 160,000
105 105
Answer 194,481 and 160,000
3) Every year Mr Kalam saves Tk. 5400 which is 15% of her annual income. Mr
Rahim spends Tk. 2800 a month which is 87.5% of her monthly income. Who has
more income?
Solution
Let, Mr Kalam’s annual income = Tk x
15% ×x = 5400
5400
Or, x = ×100 = 36000
15
36000
His monthly income = Tk = Tk 3000
12
Let, Mr Rahim’s income = Tk y

47
[CAREER AID FOR BANK AND BCS] Writtten math

87.5%×y = 2800
2800
Or, y = ×100 = 3200
87.5
Mr Rahim has monthly income [Answer]
4) A person has two stamps; one is US Stamps another is European Stamps. US
stamps is sold at 30% profit and European Stamps is sold at 20% loss. The
original purchase price of US Stamp and European Stamps are tk 18000 and tk.
20000. What is the total revenue of selling the stamps?
Solution
Here, purchases price of US stamp = Tk 18,000
Purchase price of European stamps = TK 20,000
Revenue from selling US stamps = Tk 18,000×130% = Tk 23,400
Revenue from selling European stamps = Tk 20,000×80% = Tk 16,000
The total revenue of selling the stamps = Tk (23400 + 16000) = Tk 39,400
Answer Tk 39,400
5) Babu purchased 6 identical maple wood furniture and 7 identical birch wood
furniture for his office. The regular price of each maple wood furniture was
thrice the regular price of each birch wood furniture. However, Babu obtained a
20% discount off the regular price of each maple wood furniture. If the regular
price of each birch wood furniture was Tk. 500, what was the total price of all 13
furniture? [DBBL, TO – 2024]
Solution
Here, the price of maple wood furniture = Tk. 3×500 = Tk. 1500
After 20% discount the price of maple wood furniture = Tk. 1500×80% = Tk. 1200
Now, the price of 13 piece of furniture = 6×maple wood furniture + 7×birch wood
furniture = Tk. (6×1200 + 7×500) = Tk. (7200 + 3500) = Tk. 10700
Answer Tk. 10700
6) The population of a town increased from 175000 to 262500 in a decade. Find
the average percent increase of population per year? [Pubali Bank
(Cash)‒2017(Written)]
Solution:
Increase in 10 years = (262500‒175000) = 87500
Increase in 1 year = 87500 ÷ 10 = 8750
Percentage of increase in each year = × 100) % = 5%
Answer: 5%
7) A teacher has 3 hours to grade all the papers submitted by the 35 students in
her class. She gets through the first 5 papers in 30 minutes. How much faster does
she have to work to grade the remaining papers in the allotted time? [Sonali Bank
(SO)‒2018]
Solution:
For first 5 papers

48
[CAREER AID FOR BANK AND BCS] Writtten math

Needed time for grade each paper in = = 6 min


Remaining time = 180‒30 = 150 min
Remaining papers = 35‒5 = 30
Last 30 papers,
Needed time for grade each paper in= = 5 min
( )
Required % = ×100% = 20%
Answer: 20%
8) If the price of a commodity decreases 20% and consumption increases 20%, then how
much it will increase or decrease in the total expenditure?
Solution
Initial Price 20% Consumption Initial cost New cost Change
decrease increases 20% (Tk.)
Price Tk 100 = 100×80% =100×100 = =80×120 (1000 - 9600)
= Tk 80 10000 = 9600 10000
Consumption 100 =100×1.2% ×100%
Units = 120 units = 4%
Answer: 4%
9) In an office, 1/3rd of the workers are women, 1/2 of the women are married and 1/3rd
of the married women have children. If 3/4th of the men are married and 2/3rd of the
married men have children, what parts of workers are without children? [GKwU Kvh©vj‡q GK
Z…Zxqvsk Kg©Rxex gwnjv, A‡a©K gwnjv weevwnZ Ges GK Z…Zxqvsk weevwnZ gwnjvi mšÍvb i‡q‡Q| hw` 3/4
Ask Kg©Rxex cyiæl weevwnZ nq Ges 2/3 Ask weevwnZcyiæ‡li mšÍvb _v‡K, KZ Ask Kg©Rxex mšÍvb wenxb
i‡q‡Qb?] [BB Officer – 2022]
Solution
Method 01 (Logical Operation)
Let, total number of workers = x
x
Number of women = part
3
x 2x
Number of men = x ‒ = part
3 3
1 x x
Women married = × = part
2 3 6
1 x x
Women having children = × = part
3 6 18
3 2x x
Married Men = × = part
4 3 2
2 x x
Men having Children = × = part
3 2 3
x x 7x
So, Workers with children = + = part
3 18 18
7x 11x
Workers without children = x ‒ = part
18 18

49
[CAREER AID FOR BANK AND BCS] Writtten math

11x
18 11
So, Part of workers without children = = part
x 18
Method 02 (Assumption Basis)
Let, total workers = 90.
1
Workers women = 90× = 30
3
1
Married women = 30× = 15
2
1
Married women have child =15× = 5
3
So, women don't have child = (30 ‒ 5) = 25
Again, Men = 60,
3
Married men = 60× = 45
4
2
Married men having child = 45× = 30
3
So, men don't have child = (60 ‒ 30) = 30
25 + 30 55 11
So, part of worker don't have child = = =
90 90 18
Method 03 (Cross Tabulation)
1 4
rd gwnjv n‡j Ask n‡jv cyiæl| awi, †gvU worker 90 Rb
3 3
1 4 Total
Women, gwnjv = ( ×90 Men, cyiæl = ( ×90 = 60) (90)
3 3
= 30)
Married (weevwnZ) 1 3 60
×30 = 15 ×60 = 45
2 4
Married with child 1 2 35
×15 =5 × 45 = 30
(weevwnZ I mšÍvb Av‡Q) 3 3
Worker with no 30 – 5 = 25 60 – 30 =30 55
child(mšÍvb wenxb
Kg©Rxex)
55 11
So, part of worker don't have child, mšÍvb wenxb Kg©Rxex = = Ask
90 18
11
Answer:
18
Practice
1) There was a shipment of cars. Out of which half was black in color. Remaining cars
were equally blue, white and red. 70% of black cars, 80% of blue cars, 30% of white
cars, 40% of red cars were sold. What percentage of total cars was sold?
2) From a number of apples a man sells half the number of existing apple plus 1 to the
first customer, sells 1/3 of the remaining apple plus 1 to the second consumer, and

50
[CAREER AID FOR BANK AND BCS] Writtten math

sells 1/5 of the remaining apple plus 1 to the third consumer. He then finds that he has
3 apples left. How many apples did he have originally?
3) A man's salary in 2014 was Tk. 20,000 per month and it increased by 10% each
year. Find how much he earned in the years 2015 to 2017 inclusive
4) Mr. Rahim pays 10% tax on all income earned over 60000Tk but he does not pay
any tax on interest on saving certificate. In 2006 he paid 7500 as tax & he earned
12000 as interest on savings certificates. What is his net income in 2006?
5) A restaurant has a set menu for tk.700. A 15% vat is also imposed on the food
value. The set meal has a discount of 20%. Find the effective discount for the set meal.
Vote/Preference
1) In an election between two candidates, one got 55% of the total valid votes. 20% of the
votes were invalid. If the total number of votes was 7500, what was the number of valid
votes that the other candidate got?
Solution:
One Candidate got = 55% of 80 = 44% votes
Other got = 80% − 44% = 36% votes
Let, total number of votes = 100x
According to the question
100x = 7500
7500
Or, 36x = ×36 = 2700
100
Answer: 2700
2) 10% of the voters did not cast their vote in an election between two candidates.10% of
the votes polled were found invalid. The successful candidate got 54% of the valid votes
and won by a majority of 1620 votes. Find the number of voters enrolled on the voters
list. [Bangladesh Bank IT‒2016]
Solution
Enrolled voter = 100x
Successful candidate got = 54%
Voted = 100x×90% = 90x
Other candidate got = (100‒54) = 46%
Valid vote = 90x×90% = 81x
Successful candidate got majority = (54‒46) % = 8%
Majority = 54% ‒ 46% = 8%
According to the question,
8% majority = 1620 no of voters
8% majority =1620
× 100% = 20250
100% majority = =20250 Valid vote = 20250
Therefore casted valid vote = 20250 Now, 81x = 20250
Casted vote = 100‒10% = 90% Or. X = 250
Now, Enrolled voter = 250×100 = 25000
Casted valid vote = 90×90% = 81%
So,81% castedvalid vote = 20250
100% or total enrolled voters = [(22500×100)/90] =25,000
Answer: 25,000
Practices
1) In an election, 30% of the voters voted for candidate A whereas 60% of the remaining voted
for candidate B. The remaining voters did not vote. If the difference between those who voted

51
[CAREER AID FOR BANK AND BCS] Writtten math

for candidate A and those who did not vote was 1200, how many individuals were eligible for
casting vote in the election?
2) In a country 60% of the male citizen and 70% of the female citizen are eligible to vote.
Seventy percent 70% of the male citizen eligible to vote cast their vote and 60% of the female
citizen eligible to vote cast their vote. What fraction of citizen voted during the election?
3) Of the 50 researcher in a work group 40% will be assigned to team A and the remaining
60% to team B. However 70% of the researcher prefers team A and 30% prefer team B. What
is the least possible number of researchers who will not be assigned to the team they prefer?
Equation
1) When a Student got 38% marks failed by 6 marks, when got 45% marks He got 15
marks more than the Passing marks. What's the Percentage mark of Passing?
Abyev`t GKRb QvÎ cixÿvq 38% b¤^i †c‡j †m 6 b¤^‡ii Rb¨ AK…ZKvh© nq, hLb 45% b¤^i cvq ZLb †m
cvk b¤^‡ii †P‡q 15 b¤^i †ewk cvq, cixÿvq cvk b¤^i KZ kZvsk?
Solution
Let, the total marks is x
Now, x×38% + 6 = x×45% ‒ 15
Or, x×7% = 21
21
Or, x = × 100
7
Or, x = 300
Pass marks = 300×38% + 6 = 120
120
120 is ×100% = 40% of 300
300
Another way
Here, let total mark be 100%
According to the question, 38% + 6 = 45% ‒ 15
So, 7% = 21
So, 100% = 300
So, Required Passing Mark‒38% of 300 + 6 = 120 Mark
120
Required % = ×100% = 40%
300
Answer 40%
2) A salesman's commission is 5% on all sales up to Tk. 10,000 and 4% on all sales
exceeding this. He remits Tk. 31,100 to his parent company after deducting his
commission. Find the total sales.
Solution:
Let his total sales be tk x
His commission would be, 5% of 10000 + 4% of (x‒10000)
= 500 + 4x/100 – 400 = 100 + 4x/100 = 100 + x/25
The amount he pays to company
= x‒ (x/25 + 100) = x ‒ x/25 – 100 = 24x/25 ‒ 100
According to the question
24x/25 ‒ 100 = 31100
Or, 24x/25 = 31200
Or, x = (31200 × 25)/24 = 32500

52
[CAREER AID FOR BANK AND BCS] Writtten math

Total sales = 32500 Answer: 32,500


3) Mr. Akber is a potato seller in a local bazar. When he brings potato from the
village market to his shop in the town, he has to pay a minimum of Tk. 100 toll up to
total safe of Tk. 1000. For any amount of sale above Tk. 1000, he has to pay an additional
toll of 7.5% on the increment amount. If total amount of toll paid was Tk. 257.50 then
what were his total sales proceeds from the potatoes? [Standard Bank MTO‒2016]
Solution:
Let, the increment amount is X Tk.
Toll on safety= Tk 100.
Then toll on increment amount = (257.50‒100) = 157.50 Tk.
Now, X×7.5% =157.50
Or, X×0.075 = 157.50
Or, X=2100
Hence Total sales = 1000+2100 =3100 Tk
Answer: 3100 Tk

Practices
1) A salesman is paid a monthly salary of tk 15000 plus 12.5% commission on all of
his sales .What should be his total annual sales in TK so that his annual
earnings from salary and commission is tk 265000?
Games
1) A basket ball team has won 15 games and lost 9. If these games represent 16.67% of
the games to the played, then how many more games must the team win to average 75%
for the season? [Midland bank ATO‒2015]
Solution:
Total game that have already been played = win + lose = 15+9 = 24
Let, number of total games = x
According to the question
16.67%×x =24
( × )
Or, x = =144
.
To win 75% of the game the team need to win=144×75%=108
So it needs to win (108‒15) =93 more games.
Answer: 93 games
2) Football team has won 10 games and lost 5. If the team wins the remaining games they
will have won 75% of all the games. How many more games will they play? Solution
Total game that have already been played = win + lose = 10+5 = 15
Lost game percentage = 100 ‒ win percentage = 100 – 75 =25%
Let, number of games = x
Now, 25%×x = 5
Or, x = 20
More games they have to play = 20 ‒ 15 = 5
Answer: 5 games

53
[CAREER AID FOR BANK AND BCS] Writtten math

Profit Loss
Basic
1) GKwU c‡b¨i weµq g~j¨ 10% evo‡bv n‡jv Ges weµq 30% K‡g ‡M‡jv| c«fve wK n‡e?
mgvavb
awi weµqg~j¨ 100 UvKv Ges c‡Y¨i cwigvb 100 †KwR
100 †KwR c‡Y¨i c~e©g~j¨ = 100×100 = 10000 UvKv
100 †KwR c‡Y¨i bZyb g~j¨ = 110×70 = 7700 UvKv
n«vmK…Z g~j¨ = 10000 ‒ 7700 = 2300 UvKv
2300
n«v‡mi cwigvb = 10000 ×100% = 23%
DËit c‡Y¨i `vg 23% K‡g hv‡e|
2) GKwU c‡b¨i µqg~j¨ Dci 20% evov‡bv n‡jv| mKj wWmKvD‡›Ui c‡iI 8% jvf _vK‡jv| wWmKvD›U
KZ?
mgvavb
cÖwµqv ‒ 1t‒ cÖwµqv ‒ 2t‒
awi, µqg~j¨ = 100 UvKv awi, µqg~j¨ = 100 UvKv
awi, wWmKvD›U = c% 20% evov‡bvi ci weµqg~j¨ = 120 UvKv
cÖkœg‡Z, mKj wWmKvD‡›Ui c‡iI 8% jvf _vK‡j
100 ‒ c weµqg~j¨ = 108 UvKv
120× 100 = 108 cÖK…Z jvf = 120 ‒ 108 = 12 UvKv
ev, 120× (100 ‒ c) = 10800 12
wWmKvD›U = 120 ×100% = 10 %
ev, 120× (100 ‒ c) = 10800
10800 DËit wWmKvD›U 10 %
ev, (100 ‒ c) = 120 = 90
ev, c = 100 ‒ 90 = 10
3) GKwU `ªe¨ wjwLZ g~‡j¨i 25% Qv‡o µq Kiv n‡jv| hw` µqg~‡j¨i 140% ‡ewk jv‡f weµq Kiv nq Z‡e
wjwLZ g~‡j¨i ‡P‡q 40 UvKv ‡ewk nq| `ªe¨wUi µqg~j¨ KZ? [BB AD 2024]
mgvavb awi, wjwLZ g~j¨ = 100K UvKv
µqg~j¨ = 100K×75% = 75K UvKv
weµqg~j¨ = 140%×75K = 105K UvKv
cªkœg‡Z,
105K − 100K = 40
40
ev, K = 5 = 8
ev, µqg~j¨ = 75×8 = 600 UvKv DËi 600 UvKv
4) A man sold an article for Rs.75 and lost something. had he sold it for Rs. 96, his gain
would have been double the former loss. The C.P. of the article is?
Solution
Let Cost Price is Tk x.
According to the question, 2×loss = profit

54
[CAREER AID FOR BANK AND BCS] Writtten math

2(x‒75) = 96‒x
Or, 2x – 150 = 96‒ x
Or, 3x = 246
Or, x = 82 Answer: Tk 82
5) A man buys a shirt at wholesale price and 80% marked it up to Initial price of 450
takas. How much more taka will be increased to achieve to 100% mark up?
Solution
Let, Initial price be 100x
Marked up price = 180x
According to the question
180x = 450
450
Or, x = = 2.5
180
100% marked up price = Tk 200×2.5 = Tk 500
More taka will be increased to achieve to 100% markup = Tk 500 – Tk 450 = Tk 50
Answer Tk 50
6) When a Product is sold for Tk. 700 the gain is thrice the amount when it sold for Tk.
500 as loss. Cost price of the product is? Abyev`t GKwU cY¨ 500 UvKvq weµx Ki‡j hZ UvKv ÿwZ
nq 700 UvKvq weµx Ki‡j 3 ¸Y jvf nq| cY¨wUi µqg~j¨ KZ wQj?
Solution
Here, Let the Cost Price be CP and Selling Price be SP
According to the question,
Gain = 3×Loss
Or, SP ‒ CP = 3(CP ‒ SP)
Or, 700 ‒ CP = 3(CP ‒ 500)
Or, 700 + 1500 = 3CP + CP
Or, 2200 = 4CP
2200
Or, CP = = 550
4
Answer tk. 550
7) The percentage profit earned by selling an article for Rs. 1920 is equal to the
percentage loss incurred by selling the same article for Rs. 1280. At what price should
the article be sold to make 25% profit?
Solution
Let, Cost Price = Tk. x
Now, 1920 ‒ x = x ‒ 1280
Or, 2x = 3200
Or, x = 1600
125
At 25% profit, the article should be sold= Tk. 1600× = Tk. 2000 Answer: Tk. 2000
100
8) On selling 17 balls at Rs. 720, there is a loss equal to the cost price of 5 balls. The cost
price of a ball is:
Solution
Let the cost price of a ball is Rs.x
Given, on selling 17 balls at Rs. 720, there is a loss equal to the cost price of 5 balls

55
[CAREER AID FOR BANK AND BCS] Writtten math

The equation is :
17x − 720 = 5x
Or, 17x − 5x =720
Or, 12x = 720
Or, x = 60
Therefore, cost price of a ball is Rs. 60. Answer 60
9) A man sells two commodities for Rs. 4000 each, neither losing nor gaining anything
out of the deal. If one of the products was sold at 25% profit, the other one sold at a loss
of what percentage?
Abyev`t GKRb we‡µZv cÖwZwU 4000 UvKv K‡i `ywU cY¨ weµq Ki‡j †Kvb jvf ev ÿwZ nqbv| hw` GKwU cY¨ 25%
jv‡f weµq nq Ab¨cb¨wU KZ kZvsk ÿwZ‡Z weµx n‡qwQj?
Solution
4000
If one product was sold at 25% then Cost price = Tk. = Tk.3200
1.25
Since there is no loss or gain, total Cost Price and Selling Price should be Tk. 4000 + 4000 =
Tk. 8000
So, cost of the second product = Tk.(8000 ‒ 3200) = Tk. 4800
Loss for second product = Tk. (4800 ‒ 4000) = Tk. 800
800
Loss% = ×100 =16.67% Answer: 16.67%
4800
10) A shopkeeper bought 20 kg rice of one type at 26 taka per kg & 36 kg rice of another
type at 30 taka per kg. Then he sold all rice at 30 taka per kg. What is the percentage of
profit or loss of the shopkeeper?
Solution
Cost Price of 56 kg rice = Tk.(20×26 + 36×30) = Tk 1600
1600
⸫Cost Price of 1 kg rice = Tk = Tk 28.58
56
Selling Price of 1 kg rice = Tk 30
Profit = 30 ‒ 28.57= 1.43
1.43
%profit = ×100% = 5%
28.58
Answer 5%
11) A man purchased a cow for Rs. 3000 and sold it the same day for Rs. 3600, allowing
the buyer a credit of 2 years. If the rate of interest be 10% per annum, then the man has
a gain of:
Solution
Here, Principle = Tk. 3000, interest rate = 10%, time = 2years
10
We know that, I = Pnr = 3000×2×10% = 3000×2× = 600
100
So, the future value = 3000 + 600 =3600
Selling price of the cow is Rs. 3600.
3600 − 3600
Therefore, the profit or gain by selling the cow is = ×100 = 0%
3600

56
[CAREER AID FOR BANK AND BCS] Writtten math

The man has a gain of 0% as the cost price and selling price are the same
Alternative way
20% CP: FV = 5: 6 = 3000: 3600
FV = 3600
SP = 3600
No Loss No Gain Answer No Loss No Gain
12) A trader bought some mangoes for Tk. 150 per dozen and an equal number of apples
for Tk.100 per dozen. If he sells all the fruits for Tk.140 per dozen, what be his profit/loss
in percentage?
Solution
Given that,
Selling Price of 1dozen fruits is, SP = Tk. 140
150+100
Cost Price of 1dozenz fruits is, CP = Tk. = Tk. 125
2
Profit of 1dozen fruits is = Tk. (140 ‒ 125) = Tk. 15
15
Now, Profit% = × 100% = 12%
125
Another way
Let, x dozen of mangoes are bought
Therefore buying price = Tk. 150x + Tk. 100x = Tk 250x
Selling price of 2x dozen mangoes = Tk. 140×2x = Tk. 280x
Selling price ‒ Buying price 280x ‒ 250x 30
Now, Profit% = × 100 = × 100 = × 100%=12%
Buying price 250x 250
Answer 12%
13) The price of 20 articles is the same as the selling price of x articles. If the profit is
25%. Then what is the value of x?
Solution
Let, Cost Price of 20 article = Tk Y
Y
Cost Price of 1 article = Tk
20
Again, Selling Price of X article = Tk Y
Y
Selling Price of 1 article =Tk
X
According to the question,
Y Y
= 125% of
X 20
Y Y 125
Or, = ×
X 20 100
1 1
Or, =
X 16
⸫ X = 16 Answer 16

57
[CAREER AID FOR BANK AND BCS] Writtten math

14) Raju purchased an item for Rs. 8200 and sold it at a gain of 25%. From that amount
he purchased another item and sold it at a loss of 20%. What is his overall gain loss?
Solution
Let, Raju’s purchase price = Tk 8200
125
At 25% profit selling price = Tk 8200× = Tk 10250
100
80
At 20% the selling price of new product = Tk 10250× = Tk 8200
100
Answer: no loss, no profit
15) A dishonest merchant makes a 15% profit at the time of buying and a 10% loss at the
time of selling the goods he trades. By doing so if the merchant makes amount of Tk.
3,500 on selling the particular goods, what was the real cost of the goods the merchant
sold? [Agrani Bank‒ (SO) ‒2017(Written)]
Solution:
Let, actual cp of the article Tk 100
But showing price = Tk (100+15) = Tk 115
At 10% losses the sp is Tk115×90% = tk103.5
Profit = Tk(103.5‒100) = tk3.5
When profit tk 3.5 then cp tk 100
When profit tk 1 the cp tk.
.
×
When profit tk 3500 then CP = = Tk 100000
.
Answer: Tk 100,000
16) By selling a table for tk 39 gains is as much percentage as its cost. What is the cost
price? [Sonali Bank Officer (IT)‒2016]
Solution:
Let, Cost price = Tk x
x×x
x% gain selling price = Tk
100
x×x
According to the question, x + = 39
100
=> 100x+x2 =3900
=> x2+130x‒30x‒3900 = 0
=> (x+130)(x‒30) = 0
So, x= ‒130 [Not accepted]
or x‒30 = 0
So x =30
The cost price 30 Tk Answer: 30 Tk
Two products
17) The cost of two watches taken together is Tk. 840. If, by selling one at a profit of 16%
and the other at a loss of 12%, there is no loss or gain in the while transaction, find the
cost of each of the two watches. [Social Islami Bank ‒ (PO)‒2017 ]
Solution:
Let cost price of 1st watch = Tk. x
Then, cost price of 2nd watch = Tk. (840‒x)

58
[CAREER AID FOR BANK AND BCS] Writtten math

According to the question, x×16% = (840‒x)×12%


Or, 0.16x = 0.12(840‒x)
Or, 0.16x = 100.8‒0.12x
Or, 0.28x = 100.8
Or, x = 360
So, cost price of 1st watch = Tk. 360 and
Cost price of 2nd watch = Tk. (840‒360) = Tk. 480
Answer: Tk. 360 and Tk. 480
18) A shopkeeper bought 20 kg rice of one type at 26 taka per kg & 36 kg rice of another
type at 30 taka per kg. Then he sold all rice at 30 taka per kg. What is the percentage of
profit or loss of the shopkeeper?
Solution
Cost Price of 56 kg rice = Tk. (20×26 + 36×30) = Tk 1600
1600
⸫Cost Price of 1 kg rice = Tk = Tk 28.58
56
Selling Price of 1 kg rice = Tk 30
Profit = 30 ‒ 28.57= 1.43
1.43
%profit = ×100% = 5% Answer: 5%
28.58
19) A man has 100 balls, 50 red and 50 black sell 48 each. If black sell 20% loss and red
sell 20% profit. What is the net profit or loss in tk? [DBBL PO‒2017]
Solution:
Selling price of Red balls = Tk50×48 = Tk2400
Selling price of Black balls = Tk50×48 = Tk2400
Total selling price = Tk 4800
×
CP of Black balls = Tk = Tk = Tk3000
%
×
CP of Red balls = Tk % = Tk = Tk 2000
Total cost price = Tk (2000+3000) = Tk 5000
Net Loss = Tk (5000‒4800) = Tk 200 Answer: 200 tk
20) A shopkeeper buys 100 mangoes at tk 12 each. He sells 60 mangoes at tk 17.40 each
and x mangoes at tk 11.31 each. The shopkeeper makes a profit of at least 10%. Find the
least possible value of x [Petroleum AD]
Solution:
Total cost = Tk12×100= Tk1200
Selling price = Tk 1200×110% = Tk1320
According to the question, 60×17.4+11.31x = 1320
Or, 11.31x =1320 ‒1044
Or, 11.31x = 276
276
Or, x =
11.31
Or, x = 25 Answer: 25
19) A book and a pen were sold for Tk. 3040 making a profit of 25% on the book
and 10% on the pen. By selling them for Tk. 3070, the profit realizes would have
been 10% on the book and 25% on the pen. Find the cost of each. [SEBL MTO‒2016]

59
[CAREER AID FOR BANK AND BCS] Writtten math

Solution:
Let, the cost price of pen be X and book be Y
According to the question, 125% of Y + 110% of X = 3040
Or, 125 Y +110 X = 304000
Or, 25 Y + 22 X = 60800…….. (i)
And, 110% of Y B + 125% of X = 3070
Or, 110 Y + 125 X = 307000
Or, 22 Y +25 X = 61400……. (ii)
Now, (i)× 25 – (ii) × 22 =>
625 Y + 550 X = 1520000
484 Y + 550 X = 1350800
----------------------------------
141 Y = 169200
Or, Y = 1200
Putting the value of Y in equation (i) we get
22×1200 + 25X = 60800
Or, 25X = 60800‒30000
Or, X = 30800/25
Or, X = 1400
The cost price of pen Tk. 1400 and book Tk. 1200 respectively
Answer: The cost price of pen Tk. 1400 and book Tk. 1200
20) A video magazine distributor made 3500 copies of the May issue of the magazine at a
cost of Tk. 4, 00,000. He gave 500 cassettes free to some key video libraries. He also
allowed a 25% discount on the market price of the cassette. In this manner, he was able
to sell all the 3500 cassettes that were produced. If the market price of a cassettes was Tk.
160, what is his gain or loss for the May issue of the video magazine? [National Bank
PO‒2015]
Solution:
As the distributor gives 500 copies free,
The remaining number of copy=3500‒500 =3000
Market price of each cassette was Tk. 160.
So, at 25% discount selling price =160 ‒160×25% =Tk. 120.
Now selling price of 3000 cassettes =120×3000 =Tk. 360,000
Cost of 3500 cassettes =Tk. 400000
So, loss = 4, 00,000‒ 3, 60,000 =Tk. 40,000.
Answer: Tk. 40,000.
21) A shopkeeper sells two shirts at the same price. He makes 10% profit on one and
losses 10% on the other. How much percentage does he gain or lose? [Agrani Bank SO
(Auditor) Written‒2018]
Solution:
Let, Selling price of first & Second shirt be = tk 100
First case,
10% profit on CP SP1 = 100 SP2 = 100 T = 200
So,CP + CP Of 10%=100 CP1 = = tk 111.11
%
Or, CP = CP = = tk 90.0 TCP = 202.02
2
%
Loss = CP – SP = 202.02 – 200 = 2.02
.
Loss% = ×100% = 1%
.

60
[CAREER AID FOR BANK AND BCS] Writtten math

Second case,
10% loss on CP
Alternative for MCQ:
CP ‒ CP of 10% =100
By applying effective rate:
Or, CP = = 10‒10‒(10×10/100)
Total Cost price= + =202.02 tk = 1% loss
Total Selling price = (100+100) = 200 tk
Loss = CP‒SP = 202.2‒200 = 2.02
. ×
Loss percentage = =1%
.
Answer: 1%
22) A trader marked the price of the T.V. 30% above the cost price of the T.V. and gave
the purchaser 10% discount on the marked price, thereby gaining Rs.340. Find the cost
price of the T.V ?[Sonali Bank AE(IT)‒2016]
Solution:
Let, cost price of tv is x tk CP = 100x
Marked price = 130x
Then, the market price = x + x×30% = tk
Discounted SP= 130xx90% =
At 10% discount on market price, 117x
Selling price = ‒ ×10% = Tk Gain = 117x – 110x = 17x
Gain = ‒ x = tk
ATQ
According to the question, = 340 17x = 340
Or, x = 340×100/17 Or, x = 20
Or, x = 2000
So, the cost price of the TV is Tk 2000 CP = Tk 100×20 = Tk 2000
Answer: Tk 2000
23) A person bought an article and sold it at a loss of 10%.If he had bought it for 20%
less and sold it for tk.55 more, he would have made a profit of 40%.what was the cost of
the article in taka?
Solution: Let original CP be x
Let, CP = x First SP =
SP = 0.9x New CP = 0.8x =
New CP = 0.8x
New SP = 140% × =
New SP = 0.9x+55
ATQ
According to the question,
‒ = 55
40% = × ×
Or, = 55
Or, 0.9x+55 ‒0.8x = 0.8x×0.4
Or, 55 = 0.32x+0.8x‒0.9 Or, x = 250 (Answer)

Or, x = = 250 Answer: tk 250


.
24) A computer company offered to sell a city agency new machines at a 15% discount
from the price list, and to allow the agency Tk. 8500 for each of two old computers being

61
[CAREER AID FOR BANK AND BCS] Writtten math

traded in the list price of the new machine is Tk. 62500. If the agency accepts the offer,
how much money will it have to pay for the 4 new machines?
Solution:
Total list price= Tk 62500×4 = Tk 250000
Price after discount = Tk 250000×0.85= Tk 212500
Price have to pay= Tk {212500-(8500×2)} =195500
Alternative:
Listed price = Tk 62500
15% discount, Selling price = Tk 85% of 62500 = Tk53125
So, SP of 4 computers= Tk 53125 × 4 = 212500
Exchanging amount of 2 computer = Tk 8500 × 2 = Tk 17000
So, actual paying amount = 212500- 17000) = 195500 tk Answer: 195500 tk
25) Karen sold her house at a loss of 25 percent of the price that she originally paid for
the house, and then bought another house at a price of 30 percent less than the price she
originally paid for her first house. If she sold the first house for $225,000, what was her
net gain, in dollars, for the two transactions?
Solution:
Let, CP of 1st house =100%
Now, 75% = $225000
Or, 100% = $300000
CP of 2nd = 300000×70/100 = $210000
Net gain =225000-210000=$15000
Solution: $15000
26) A salesman sells two shirts at BDT 594 each. On one he gains 10% and on the other,
he loses 10%. Find his gain or loss percent on the whole. [Janata Bank Ltd Officer
(Cash)-2020]
Solution:
Total S.P = Tk 594×2 = Tk 1188
Let CP = x
According to the question,
594-x = gain = 10% of x
Or, x =
.
Or, x =540
2nd condition, x-594 = loss = 10%×x
Or, x = 660
So total Cost price = 660+540 =1200
Loss =1200-1188 = 12
%loss = ×100 = 1% Answer: 1%
Pratise
1) When a producer allows 36% commission on the retail price of the product, he earns a
profit of 8.8%. What would be his profit percent if the commission is reduced by 24%?
Answer: 49.60%

62
[CAREER AID FOR BANK AND BCS] Writtten math

2) When a producer allows 36% commission on the retail price of the product, he earns a
profit of 8.8%. What would be his profit percent if the commission is reduced to 24%?
3) A shirt sold at 6% profits. If the purchase price was 4% less and selling price was Tk. 4
more, the profit is 12.5%.What was the purchase price of the shirt. [Madhumoti Bank
‒(PO)‒2017(Written)]
Exponential

1) If m and whole number such that mn = 121; value of (m‒1)(n+1) = ?


Solution
Given that, mn = 121
Both m and n are whole numbers
121=121×1 = 121
121=11×11
Now, If m =11 then n = 2
(m‒1)(n+1) = (11 – 1)(2+1) = 103 =1000
If, m = 121 then n =1
(m‒1)(n+1) = (121‒1)(1+1) = 1202 = 14400 Answer 1000 or 14400
2) (2x+4 ‒ 4.2x+1) ÷ (2x+2 ÷ 2)
Solution
2x+4 - 4.2x+1 2x+4 - 22×2x+1 2x+4 - 2x+3 2x24 - 2x23 2x(24 - 23) 8
= = = = = = 4 (Ans)
2x+2 ÷ 2 2x+2-1 2x+1 2x+1 2x×2 2
p pq −2 q
4 ×16 ×2 −pq
3) q pq −3 q ×4
8 ×4 ×2
Solution
4 p ×16pq×2−2 q −pq = 22 p ×24pq×2−2 p −2pq 22 p ×24pq×2−2 p
×4 = ×2 = ×2−2pq
8 q ×4pq×2−3 q 23 q ×22pq×2−3 q 23 q ×22pq×2−3 q
= 22√p +4pq−2√p ÷ 23√p +2pq−3√p ×2−2pq = 24pq−2pq ×2−2pq = 22pq−2pq =20 = 1[Answer]
9 8 2
4) ( )x×( )(x‒1) = , value of x?
4 27 3
Solution

9 8 2 32 23 2
( )x×( )(x‒1) = Or, ( 2 )x×( 3 )(x‒1) =
4 27 3 2 3 3
32x 23(x -1) 2 2-2x 2(3x -3) 2
Or, ( 2x )×{ 3(x - 1) }= Or, -2x × (3x - 3) =
2 3 3 3 3 3
-2x+3x-3 x-3
2 2 2 2
Or, -2x+3x-3 = Or, x-3 = ( )1
3 3 3 3
2 (x‒3) 2 1
Or, ( ) =( ) Or, x – 3 = 1
3 3
Or, x = 4
Answer: 4

63
[CAREER AID FOR BANK AND BCS] Writtten math

7 5 3
÷ ÷
2 2 2
5) Simplify ÷5.25[Pubali Bank PLC, JO – 2023]
7 5 3
÷ of
2 2 2
Solution
7 5 3 7 2 2 14
Denominator = ÷ ÷ = × × =
2 2 2 2 5 3 15
7 5 3 7 4 14
Numerator = ÷ of = + =
2 2 2 2 15 15
14
15 14 15 525 100 4 4
Now, ÷5.25 = × ÷ = 1× = Answer:
14 15 14 100 525 21 21
15
6) 3.27x = 9x + 4 n‡j x Gi gvb KZ?
mgvavb: 3.27x = 9x + 4
 3. (33) x = (32)x + 4  3.33x = 32x + 8
3x + 1 2x + 8
3 =3
 3x + 1 = 2x + 8  3x  2x = 8  1 x=7
Answer: 7
9 1 9 1
7) If logx = the value of the base is-(logx = n‡j x Gi gvb-) [Rupali Bank Ltd.
16 2 16 2
(Officer-2019)]
9 1
mgvavb: logx   =
16 2
1
9
 x2 =
16
9 2 81 81
x=  = Answer:
16 256 256

7) If + = 320, then find the value of n.


Solution:
Given, 2 +2 = 320
Or, 2 (2 +2 ) = 320 Or, 2 ( +2) = 320
Or, 2 × = 320 Or, 2 = 320× = 128
Or, 2 =2 Or, n = 7 Answer: 7
x
`8) 2 = ⁷√1024, find the value of x?
Solution
2x =⁷√1024 Or, 2x = 1024
×
Or, 2x = 2 Or, x =
Answer:

64
[CAREER AID FOR BANK AND BCS] Writtten math

Equation
Basic
1) b = 2.35 + 0.25x
c = 1.75 + 0.40x
In the equations above, b and c represent the price per pound, in dollars, of beef and
chicken, respectively, x weeks after July 1 during last summer. What was the price per
pound of beef when it was equal to the price per pound of chicken?
Solution:
B =2.35 + 0.25x
C = 1.75 + 0.40x
Therfore, 2.35+.25x =1.75+.40x
Or, x = 4
B = 2.35+0.25×4 = 3.35 Answer: $3.35
2) `ywU msL¨vi AbycvZ 3:5| msL¨v؇qi mv‡_ 10 ‡hvM Ki‡j, AbycvZ 5:7 nq | msL¨vØq KZ?
mgvavb
awi, msL¨vØq 3K Ges 5K
3K+10 5
cÖkœg‡Z, 5K+10 = 7
ev, 21K + 70 = 25K + 50
ev, 4K = 20
20
ev, K = 4
myZivs, msL¨vØq 5×3 = 15 Ges 25
DËit 15 Ges 25
3) Taxi fare is described by the following relationship:
Total taxi Fare = Tk. A upto 2 km + Tk. B per km (exceeding 2 km) + Tk.60 for per hour
Waiting.
A person paid Tk.432 for running 52 km and 2 hours of waiting charge. The same person
paid Tk.732 for running 102 km and 2 hours of waiting charge. Find the value of "A"
and"B".
Solution
Here, Total taxi Fare = Tk. A upto 2 km + Tk. B per km (exceeding 2 km) + Tk.60 for per
hour waiting
Equation Distance Waiting Total Tk.
time
First equation Tk. Tk.50B Tk. 60×2 432
2A
Second Tk. Tk. 100B Tk. 60×2 732
equation 2A

Therefore, 2A + 50B = 432 ‒ 120 = 312..... (i)


And 2A + 100B = 732 ‒ 120 = 612....... (ii)
(ii) ‒ (i),

65
[CAREER AID FOR BANK AND BCS] Writtten math

2A‒2A + 100B ‒ 50B = 612 ‒ 312


Or, 50B = 300
So, B = 6
Putting B = 6 at any of the equations,
2A = 312 − 50×6 = 12
Or, A = 6
Answer A = 6 and B = 6
4) GKwU ‡U«b 1g ‡÷k‡b 1/3 hvÎx bvg‡jv Ges 280 Rb DV‡jv| 2q ‡÷k‡b 1/2 bvg‡jv Ges 12 Rb DV‡jv
| eZ©gv‡b 248 hvÎx _vK‡j c«_‡g KZ wQ‡jv?
mgvavb
awi c«_‡g wQj = K msL¨K Rb
bvg‡jv/K‡g †M‡jv DV‡jv/evo‡jv _vK‡jv
1g ‡÷k 1 1 2
× K Rb 280 Rb K ‒ K + 280 Rb =
3 3 3 K + 280 Rb
2q ‡÷kb 1 2 1 2 1
12 Rb
2 ( 3 K + 280) Rb 2 (3 K + 280) + 12 Rb = 3 K + 152 Rb
cÖkœg‡Z,
1
3 K + 152 Rb = 248
1
ev, 3 K = 248 ‒ 152 = 96
ev, K = 96 × 3 = 288 DËit hvÎx wQj 288 Rb
5) A bag Contains coin of TK 1, 2, 5 in the ration of 2:3:4. If the total Amount is 280 find
the number of coins of Tk. 5 denomination. [GKwU e¨v‡M 1, 2 I 5 UvKvi ‡bv‡Ui msL¨vi AbycvZ
2:3:4| hw` †gvU UvKv 280 nq Z‡e 5 UvKvi KZwU †bvU i‡q‡Q ?]
Solution
Let, no of 1, 2 and 5 tk. notes respectively 2x, 3x and 4x
According to the question,
1×2x +2×3x + 5×4x = 280
Or, 28x = 280
Or, x = 10
200
So, Amount of Money of BDT 5 = 20×10 = 200 which as denomination = = 40
5
Answer 40
Linear equation
6) 10 cows can graze in a field for 15 days and 20 cows can graze in the same field for 10
days. For how many days can 30 cows graze in the field?
Solution
Let, Total number of grass x unit
New grass grows per day y unit
x + 15y = 10×15 = 150 ‒‒‒‒‒‒ (1)
x +10y = 20×10 = 200 ‒‒‒‒‒‒ (2)

66
[CAREER AID FOR BANK AND BCS] Writtten math

On subtracting & solving, 15y – 10y = 150 – 200


Or, 5y = −50
Or, y = −10
Now, x − 150 = 150
Or, x = 300
Now, 300 + (−10) ×Z = 30×Z
300
Or, Z = = 7.5 days
40
Answer 7.5 days
7) Matin visited his cousin Raju during the summer vacation. In the mornings, they both
would go for swimming. In the evenings, they would play tennis. They would engage in at
most one activity per day, i.e. either they went swimming or played tennis each day.
There were days when they took rest and stayed home all day long. There were 32
mornings when they did nothing, 18 evenings when they stayed at home, and a total of 28
days when they swam or played tennis. What duration of the summer vacation did Matin
stay with Raju?
Solution
Let, total days = d
Matin, went for swimming = (d – 32) days
Matin, played tennis for (d – 18) days
According to the question,
(d – 32) + (d −18) = 28
Or, 2d =28+50
78
Or, d =
2
⸫ d = 39
Matin stayed with raju for 39 days. [Answer]
8) Abir and Chaity buy chocolates and pens at a grocery store that sales each of its
chocolates for a certain price and each of its pan for a certain price. Abir spends twice as
much as Chaity spends buying three times as much chocolate and the same number of
pens. If Chaity spends TK.500 on four chocolates and five pens. How much does one pen
cost? [DBBL, AO – 2024]
Solution
Price of per piece pen and chocolate is receptively Tk. P and Tk.C
First Condition, Second Condition,
Chaity spends , 4C + 5P = 500 ‒‒‒‒ (1) Abir spends, 4×3C + 5P = 500×2
Or, 12C + 5P = 1000 ‒‒‒‒‒‒ (2)
Subtracting equation (1) from equation (2)
12C ‒ 4C + 5P ‒ 5P= 1000 ‒ 500
Or, 8C = 500
500
Or, C = = 62.5
8
Putting the value of C = 62.5 we get, 5P = 500 ‒ 62.5×4 = 250
250
Or, P = = 50 Answer One pen costs Tk. 50
5

67
[CAREER AID FOR BANK AND BCS] Writtten math

9) Two students appeared at an examination. One of them secured 9 marks more than
the other and his marks were 56% of the sum of their marks. What are the marks
obtained by them?
Solution
Let, One gets x & other One gets (x + 9).
According to the question,
14 14
X + 9 = (x + x + 9)× [ = 0.56]
25 25
Or, 25x + 225 = 28x + 126
Or, 3x = 99
Or, X = 33
Other one gets (x + 9) = 33 + 9 = 42 Answer 42
10) There are in 1 million people in Comilla City. If 10% male increases and 6% female
decreases, the total population will remain unchanged. Find the male and female
numbers.
Solution
Let, number of male = x
Number of Female = y
1st condition,
x + y = 1000000 [1 million = 1000000] ‒‒‒‒‒‒‒‒‒‒‒ (1)
2nd condition
110x 94y
+ = 1000000
100 100
Or, 110x + 94y = 1000000×100
⸫ 110x + 94y = 100000000 ‒‒‒‒‒‒‒‒‒‒‒‒‒‒ (2)
(1) × 110 – (2)
110x + 110y = 11,00,00,000
110x + 94y = 10,00,00,000
‒‒‒‒‒‒‒‒‒‒‒‒‒‒‒‒‒‒‒‒‒‒‒‒‒‒‒‒‒‒‒‒‒‒‒‒‒‒‒‒‒‒
Or, 16y = 10000000
10000000
Or, y =
16
⸫ y = 6, 25, 000
From equation (1)
y = 1000000 – 625000
Or, y = 3, 75, 000 (Answer)
11) There are two examination rooms A & B. If 10 students are sent from A to B,the
number of students in each room becomes the same. If 20 candidates are sent from B to
A,the number of students in A becomes double the number of students in B.What is the
number of students in room A?
Solution
1st condition
A ‒ 10 = A +10
Or, A = B +20

68
[CAREER AID FOR BANK AND BCS] Writtten math

2nd condition
2(B‒20) = A+20
Or, 2y‒40 = A+20
Or, 2B‒40 = B+20+20
Or, B = 80
Now, A = 80+20 = 100
Students in room A = 100
Answer:100 students
12) Babu purchased 6 identical maple wood furniture and 7 identical birch wood
furniture for his office. The regular price of each maple wood furniture was thrice the
regular price of each birch wood furniture. However, Babu obtained a 20% discount off
the regular price of each maple wood furniture. If the regular price of each birch wood
furniture was Tk. 500, what was the total price of all 13 furniture? [DBBL, TO – 2024]
Solution
Here, the price of maple wood furniture = Tk. 3×500 = Tk. 1500
After 20% discount the price of maple wood furniture = Tk. 1500×80% = Tk. 1200
Now, the price of 13 piece of furniture = 6×maple wood furniture + 7×birch wood furniture =
Tk. (6×1200 + 7×500) = Tk. (7200 + 3500) = Tk. 10700
Answer Tk. 10700
13) The cost of 3 horses is same as the cost of 5 cows. If total cost of 4 horses and 6 cows
is Tk 1900, find the cost of one horse.
Solution Let, the cost of one horse and one cow are Tk x and Tk y respectively.
5
Therefore, according to the question, 3x = 5y ⇒ x = y ... (i)
3
Again, 4x+6y =1900
5
⇒ 4 × y + 6y = 1900
3
⇒ 20y + 18y = 5700
⇒ 38y = 5700
5700
⇒ y= = 150
38
On putting y = 150 in (i), we get
5
x = ×150 = 250
3
∴cost of one horse = Tk 250
Answer Tk 250
Investement
14) Ram’s father invested some money and for every Tk. 12 he invested he got a total of
Tk.15 back. if Ram’s father invested Tk. 300, how much in total did he get back?
Solution
For Tk 12 he gets Tk 15
15
For Tk 300 he gets = ×300 = Tk 375
12
Answer Tk 375

69
[CAREER AID FOR BANK AND BCS] Writtten math

15) Bilkis invests Tk. 2,400 in the XYZ Bank at 5%. How much additional money must
she invest at 8% so that the total annual income will be equal to 6% of her entire
investment?
Solution
Let X is the additional amount of money invested at 8%.
So, the entire investment will be (2400+X)
According to the question,
Or, 5% of 2400 + 8% of X = 6% of (2400+X)
Or, 120 + 0.08X = 144+0.06X
Or, 0.02X = 24
Or, 2X = 2400
Or, X = 1200
Answer: Tk 12
16) A man invests BDT 50000 in two bonds Beximco and DBBL at annual interest rate
6.5% and 9% respectively. He earns total BDT 4000 as interest. Find the investment of
each bonds..
Solution
Let, the investment at 6.5% interest rate bond = Tk x
And the investment at 9% interest rate bond = Tk (50000 – x)
According to the question
6.5%of x+ 9%of (50000‒x) = 4000
Or, 6.5x+450000‒9x = 400000
50000
Or, x =
2.5
⸫ x = 20000
Therefore, the investment at 6.5% interest rate bond = Tk 20,000
The investment at 6.5% interest rate bond = Tk (50,000 – 20,000) =Tk 30,000 (Answer)
18) Three friends invest equal shares in a business. After careful calculation, they
realized that each of them would have invested Tk.3,000 less if they had found 2 more
friends with equal shares. What is the amount of total investment in the business?
Solution
Let, each of the three friends invested Tk. x and total investment of three friends = Tk. 3x
According to the question,
3x = 5(x ‒ 3000)
Or, 3x = 5x ‒ 15000
Or, 2x = 15000
So, x = 7500
Therefore total investment, 3x = Tk. 3×7500 = Tk. 22500
Answer Tk. 22500
Distribution
20) A sum of money is to be distributed equally among a group of children. If there were
25 children less than each would get TK. 1.50 more, and if there 50 children more, each
would get TK. 1.50 less. Find the number of children and the amount of money
distributed. [National Bank PO‒2015]
Solution

70
[CAREER AID FOR BANK AND BCS] Writtten math

Let, total number of children be ‘x’ and From equation (i) and (ii)
money distributed to child ‘y’ 50y‒75=25y+37.5
So, total amount of money =xy Or, y = 4.5
According to the question, From equation (2) =>>
xy =(x‒25)(y+1.50) 1.5x = 50×4.5‒75
Or, xy = xy‒25y‒37.5+1.5x Or, x =100
Or, 25y+37.5=1.5x…….. (i) So, the number of children =100
Again, xy = (x+50)(y‒1.50) And the amount of money = Tk. (4.5×100) = Tk.
Or, xy = xy‒75+50y‒1.5x 450
Or, 1.5x = 50y‒75……. (ii) Answer: children100 and Tk. 450.

21) A man won a lottery of Tk.20,00,000. He gave 2/5 of the amount to his wife and 1/4 of
the balance to his son. He donated 2/9 of the balance amount. How much money was
donated?
Solution Here, total amount = Tk 2000000
2
Amount received by wife = ×2000000 = Tk. 800000
5
Remaining amount = 2000000 – 800000 = Tk.1200000
1
Amount received by son = × 1200000 = Tk.300000
4
Remaining amount = 1200000 – 300000 = Tk.900000
2
Amount donated = × 900000 = Tk.200000
9
AnswerTk.200000
22) A widow took 1/3rd of the estate. Each son of the twos took the remaining estate. If
the widow and one son took 360000 Tk, then what is the value of the estate?
Solution
1
Widow gets = rd of the estate
3
1 2
Two sons get = (1 ‒ ) = rd of the estate
3 3
2 1 2
One son gets = × = th of the estate
3 2 6
1 2 4
Widow + 1 son = + = th of the estate
3 6 6
Let, value of the estate = Tk x
4
According to the question, x = 360000
6
6
Or, x = 360000 × = Tk 5,40,000
4
Answer Tk 5,40, 000
23) 200 children came to park with bicycle and tricycle All the older children
used bicycles and all the younger children used tricycles for coming to the park.

71
[CAREER AID FOR BANK AND BCS] Writtten math

As such there were 480 wheels altogether that rode into the park. Find the
number of elder children and younger children?
Solution
Let, number of elder children = x and number of younger children = 200 ‒ x
According to the question
2x + 3(200 ‒ x) = 480
Or, 2x + 600 ‒ 3x = 480
Or, x = 600 ‒ 480 = 120
Therefore, number of elder children = 120 and number of younger children = 200 ‒
120 = 80
Answer: 120 and 80
24) A labor is paid Tk.100 per hour for working 8 hours in a day as regular pay, His
payment is 1.5 times than the rate of regular pay in excess of 8 hours in a single day as
overtime. If the labor received a total of Tk.1000 for a single day, how long did he work
on that day?
Solution
Let. A labor does “x” hour extra duty
According to the question
100×8 + x×100×1.5 = 1000
Or, 100(8 + 1.5x) = 1000
1000
Or, 8 + 1.5x = = 10
100
Or, 1.5x = 10 ‒ 8 = 2
2
Or. x = = 1.33
1.5
Total hours he works = 8 + 1.33 hours = 9.33 hours = 9 hours 20 mins
Another way
For 8 hours duty a labor gets = Tk 100×8 = Tk 800
Extra payment he gets = Tk (1000 ‒ 800) =Tk 200
Extra per hour payment = Tk 100×1.5 = Tk = 150
200
Extra time he works = hours = 1.33 hours
150
Total hours he works = 8 + 1.33 hours = 9.33 hours = 9 hours 20 mins
Answer: 9 hours 20 mins
Comsmption and price
25) A family has 480 kg of rice for X number of weeks. If they need to use the same
amount for 4 more weeks, they need to cut down their weekly assumption of rice by 4 kg.
What is the value of X? [Rupali Bank Officer Cash-2018]
Solution:
Family’s previous Consumption by week =

72
[CAREER AID FOR BANK AND BCS] Writtten math

Latest Consumption by week =


According to the question,
Or, - =4
Or, 4x (x+4) = 480×4
Or, x (x+4) = 480
Or, x² + 4x - 480 = 0
Or, (x-20) (x+24) = 0
Or, x-20=0
Or, x=20
Either, x = ‒24
Answer: 20 weeks
26) A reduction of 20% in price of sugar enables a man to purchase 4 kg more at Tk.
280. What is the original rate and reduction rate?
Solution
Approach‒01
Let, the original price of per kg sugar is Tk. x
80 4x
Reduced of per kg sugar is Tk. x ‒ x×20% = x×80% = x = = Tk. 0.8x
100 5
According to the question, [mZ©KZv, mgxKiY †Zix I K¨vjKz‡jkb hZœmnKv‡i bv Ki‡j g¨v_wU wgj‡e bv]
280 280 10×280 280 280(10 ‒ 8)
‒ =4 Or, ‒ =4 Or, =4
0.8x x 8x x 8x
280×2
Or, x = = 17.5
4×8
Now, the original price = Tk. 17.5 and reduced price = Tk. 17.5×0.8 = Tk. 14
Approach‒02
Here, 20% of 280 = 56 Tk = Reduction Price of 4kg Sugar
So, Reduction Price of l kg Sugar14 Tk
So, Original Price of 1 kg Sugar = 14 × 1.25 = 17.5 Tk
Answer Original price = Tk. 17.5 and reduced price = Tk.14
27) If the price of onion falls by 2.5% a person can buy 9 kg more of onion for
1260 than before. If the price had raised by 12.5% how much onion would have
bought for same sum?
Solution
Let, original price = Tk x
25x 1000x - 25x 975x 39x
Reduced price = Tk(x–x × 2.5%) = Tk( x ‒ ) = Tk = Tk = Tk
1000 1000 1000 40
According to the question,
New quantity of onion – Previous quantity onion = 9
1260 1260
Or, – =9
reduced price original price

73
[CAREER AID FOR BANK AND BCS] Writtten math

1260 1260
Or, – =9
39x x
40
40 1260
Or, 1260 × – =9
39x x
1260×40 1260
Or, – =9
39x x
1260×40 - 1260 ×39
Or, =9
39x
1260×(40 - 39)
Or, =9
39x
1260×1
Or, =9
39x
1260 140
Or, x = =
39×9 39
140 140 9 1260
New price = Tk × 112.5 = Tk × = Tk
39 39 8 312

1260 1260×312
New quantity of onion = 1260 = 1260
= 312 kg
312
Answer: 312 kg
28) Amin can buy oranges at a certain rate. When the price falls to 60% he can
buy 4 oranges more at tk 12. How many oranges can he buy at original price at
tk. 24?
Solution
Let, initial orange be 'n' and Origin price of orange is x tk
Here nx =12
According to the question, 60 % x×(n+4) =12
Or, 3xn+12x = 60
Or, 3×12+12x = 60
Or, 12x = 24
⸫x = 2
He can buy 24/2=12 at original price
Price Ratio = 10:6
Quantitty Ratio = 6:10
Previous price=12/6 Or, TK. 2
Present price=12/10 Or, TK. 1.2
Oranges can be purchased at = 24/2 = 12
Answer Tk. 12

74
[CAREER AID FOR BANK AND BCS] Writtten math

29) Two students appeared at an examination. One of them secured 9 marks


more than the other and his marks were 56% of the sum of their marks. What
are the marks obtained by them?
Solution
Let, One gets X & other One gets (X+9)
According to the question,
X+9 = 14/25 (X+X+9)
Or, 25X + 225 = 28X + 126
Or, 3X = 99
Or, X = 33
Other one gets Or, (X+9) = 33+9 = 42
Answer 42
30) A man engaged a worker on the condition that he would pay him Tk.30, 000
and one uniform after one year of service. The worker served only 9 months and
got Tk.22,000 and a uniform. What is the price of the uniform?
Solution Let the price of the uniform be Tk. x
Then, total income for 12 months = Tk. (30000+x)
30000+x 90000+3x
⇒ Income for 9 months = ×9 =
12 4
90000+3x
Given, = 22000 x
4
⇒ 90000 + 3x = 88000 + 4x ⇒x = 2000
Answer price of the uniform is Tk. 2000

31) When a Student got 38% marks failed by 6 marks, when got 45% marks He
got 15 marks more than the Passing marks. What's the Percentage mark of
Passing? Abyev`t GKRb QvÎ cixÿvq 38% b¤^i †c‡j †m 6 b¤^‡ii Rb¨ AK…ZKvh© nq, hLb 45% b¤^i
cvq ZLb †m cvk b¤^‡ii †P‡q 15 b¤^i †ewk cvq, cixÿvq cvk b¤^i KZ kZvsk?
Solution
Let, the total marks is x
Now, x×38% + 6 = x×45% ‒ 15
Or, x×7% = 21
21
Or, x = × 100
7
Or, x = 300
Pass marks = 300×38% + 6 = 120
120
120 is ×100% = 40% of 300
300
Another way
Here, let total mark be 100%
According to the question, 38% + 6 = 45% ‒ 15

75
[CAREER AID FOR BANK AND BCS] Writtten math

So, 7% = 21
So, 100% = 300
So, Required Passing Mark‒38% of 300 + 6 = 120 Mark
120
Required % = ×100% = 40%
300
Answer 40%
32) A company employs 20 workers for every 3 managers, and 5 managers for
every director. If the total number of employees at the company is between 300
and 400, what is the number of managers who work at the company?
Solution:
W = Workers
M = Managers
D = Director

As per the question,


20W = 3M
5M = 1D
100W = 15M = 3D; Total employees = 100 + 15 + 3 = 118
200W = 30M = 6D; Total employees = 200 + 30 + 6 = 236
300W = 45M = 9D; Total employees = 300 + 45 + 9 = 354
400W = 60M = 12D; Total employees = 400 + 60 + 12 = 472
From the question, we are asked to find the number of managers of a company that has
between 300 to 400 employees.
The only option is the highlighted one. Hence the total number of managers would be
45
Answer: 45
33)There are students and benches in a classroom. If 4 students sit in each bench,
3 benches are left vacant. But if 3 students sit in a bench, 3 students are left
standing. What are the total no. of students?
Solution: Another way
Let the number of students = y
let the number of benches = x +3=
From the first statement or, 3x+36 = 4x−12
y = 4x−12 (three empty benches × 4 students) Or, x = 48
From the second statement
y = 3x+3
Substituting equation 2 into equation 1
3x+3 = 4x−12
Or, x =15
Substituting the value for x in equation 2
y =3×15+3=48 Answer: 48

76
[CAREER AID FOR BANK AND BCS] Writtten math

Interest
Simple
Finding rate of interest
1) Reena took loan of Tk. 1200 at simple interest rate for as many years as the rate of
interest. If she paid Tk.432 as interest at the end of the loan period, what is the rate of
interest?
Solution Let, rate = r% and time = r years, I = Tk.432, P = Tk.1200
I 432×100 36
Then, r = = = ⇒ r2 = 36 ⇒r=6
pr 1200×r r
Answer Hence, the rate of interest is 6%
2) Mr. A deposited a certain amount of money for a fixed period of time. On maturity, he
received a total of Tk. 60,000 and the ratio of interest and investment was 3:6. If the
interest rate was 6.25%(simple). Calculate the time period for which the monkey was
invested?
Solution
Ratio method
3
Interest = Tk. 60000× = Tk. 20000
9
6
Investment (Principal) = Tk. 60000× = Tk. 40000
9
If principal is 100 then interest is 6.25 in 1 year
20000×100
If principal is 40000 then interest is 20000 in = = 8 years
6.25×40000
Formula method
Let, Interest, I = 3x, Investment, P = 6x
Here, rate of interest, r = 6.25%
We know,
I = Pnr
Or, 3x = 6x×n×6.25%
3×100
Or, n = = 8%
6.25×6
Answer: 8%
3) If A lent to B 5000tk for 2 years and to C 3000tk for 4 years at the same interest, then
A got 2200 tk interest in total. What is the rate of interest?
Solution
Here, rate of interest = r%
According to the question,
5000×2×r%+3000×r%×4 = 2200 [⸪I = Pnr]
Or, 100r+120r = 2200×100
Or, 220r = 220000
Or, r = 10% Answer: 10%

77
[CAREER AID FOR BANK AND BCS] Writtten math

4) Reena took loan of Tk. 1200 at simple interest rate for as many years as the rate of
interest. If she paid Tk.432 as interest at the end of the loan period, what is the rate of
interest?
Solution Let, rate = r% and time = r years, I = Tk.432, P = Tk.1200
×
Then, r = = =
×
⇒ = 36 ⇒r=6
Answer Hence, the rate of interest is 6%
Finding principal
1) Sakib and Labib individually borrowed different amount of money from a particular
bank on the same day at rate of 20% simple interest. The total money paid by Sakib in 3
years as principal plus interest was the same amount Labib paid in 2 years as principal
plus interest. Find the ratio of their individual loan amount. [Agrani Bank Senior Officer-
2017]
Solution:
Sakib and Labib borrowed amount X & Y Tk respectively
According to the question,
X+ X of 20%×3=Y+Y of 20%×2
Or,X+0.6 X = Y +0.4Y
Or, 1.6 X = 1.4Y
Or, X: Y = 1.4:1
Or, X: Y = 7:8 Answer: 7:8
2) The simple interest rate of a bank was reduced to 5% from 7%. As a
consequence Karim’s income from bank interest was reduced by Tk. 2100 in 5
years. How much is Karim’s initial deposit with the bank? [Standard Bank (TAO) Cash
- 16]
Solution
The rate of interest rate reduced= 7% - 5% = 2%
In 5 years interest reduced=2100 Tk
In 1 year interest reduced = 2100/5=420 Tk
Tk.2 reduce when deposit Tk. 100
Tk. 1” ” ” 100/2
Tk.420” ” ” (100×420)/2
=21,000 Tk
Answer: 21,000 Tk
3) The population of a village is 10000. In one year the male population is increase by 6%
and female population by 4%. If the population end of the year is 10520. What is the
original size of male population in the village? [GKwU MÖv‡g 1000 Rb RbmsL¨v i‡q‡Q| 1 eQ‡i cyiæl‡`i
msL¨v 6% I gwnjv‡`i msL¨v 4% e„w× cvq| 1 eQi ci RbmsL¨v nq 10520| MÖvgwU‡Z cyiæl‡`i cÖK…Z msL¨v KZ?]
Solution
Let, the number of male population was = x and, the number of female population was =
(10000 ‒ x)
Population increase in one year = (10520 ‒ 10000) = 520

78
[CAREER AID FOR BANK AND BCS] Writtten math

According to the Question,


g¨v_wU B›Uv‡i‡÷i bv n‡jI GLv‡b, I = Pnr m~Î cÖ‡qvM Kiv n‡q‡Q|
x×6%×1 + (10000 ‒ x)×4%×1
= 520 Avcwb g¨v_ Kg Kiæb †mUv mgm¨v bv Z‡e mKj m~Î Rvbv _vK‡Z n‡e|
Or, 6x + 40000 ‒ 4x = 520× 100
Or, 2x = 52000 ‒ 40000
Or, x = 6000
So the male population originally there = 6000 Answer6000
4) Alif bought a bike, but he paid 15000 taka directly & remaining money with 8%
interest in 2 years. In total he paid 28920 takas. What was the actual cost of the bike?
Solution
Here, rate of interest, r = 8%, time, n = 2 years
Remaining money, he paid =Tk(28920‒15000) =Tk13920
Principal + Interest =Tk13920
Or, P+ Prn = 13920 [⸪I = Pnr]
Or, P (1+8%×2) = 13920
13920 13920×100
Or, P = = = 12000
1.16 116
Or, P=12000
Actual cost of the bike =Tk(15000+12000) = Tk27000
Answer: Tk 27000
5) Salam used a part of Tk.100,000 to purchase a television. Of the remaining portion, he
invested 1/3 of it at 4% simple annual interest and 2/3 of it at 6% simple annual interest.
If, after a year, the income from two investments totaled tk 320, what was the purchase
price of the television? [ONE Bank SCO‒2017]
Solution:
Let, The purchase price of the TV=Tk x
Given that,
Total amount=100000 Tk
Remaining amount of money =Tk(100,000 – x)
According to the question
{(100,000 – x)× ×4%}+{(100,000 ‒ x)× × 6%} = 320
%
Or, {(100,000 – x) × } + {(100,000 ‒ x)×4% } = 320
, , ×
Or, ( × ‒ + ‒ = 320
Or, + 4000 – ( + ) = 320
,
Or, ‒ 320 =
Or, =
Or,16x = 1504000
Or, x = 94,000
Answer: 94,000 Tk
6) A total of Tk. 1200 is deposited in two saving accounts for one year portion at 5%
simple interest and the rest at 7% simple interest. If Tk. 72 was earned as interest, how
much was deposited at 5%? [Standard Bank MTO‒2016]

79
[CAREER AID FOR BANK AND BCS] Writtten math

Solution:
Let, Tk. x be deposited at 5% simple interest and Tk. (1200‒x) at 7%.
According to the question,
5% of x + 7% of (1200‒x) =72
Or, 5x/100+ 7(1200‒x)/100 =72
Or, 5x+ 8400‒ 7x = 7200
Or, ‒2x = 7200‒8400
Or, ‒2x = ‒1200
Or, x = 600
Hence, Tk. 600 is deposited at 5% simple interest. Answer: Tk. 600
Finding time
1) In how many years, Tk. 150 will produce the same interest @8% Tk. 800 produce in 3
years @4.5%? [PKSF AM 2014]
Solution
Principal, P1= Tk. 150 4.5
Interest = 800×3× = Tk. 108
Interest rate, r1 = 4.5% 100
Interest rate, r2 = 8% Again, I = P1×n×r
Or, 108 = 150 ×n×8%
Principal, P2 = Tk. 800 108×100
Time, n =? Or, n = = 9 years
8×150
Answer9 years
2) What will be the deposited amount at initial stage, if it becomes tk 43,750 the end of 5
years with a simple interest rate of 15% per annum? How many years it will take the
same deposited amount to become tk 55,000 tk? [Bank Asia PO/MTO-2015]
Solution:
Here, Second case,
Principal + Interest, P+I = 43750 Principal, P=25,000 tk
Time, n =5 years Interest, I=55,000-25,000 = 30,000 tk
Interest rate, r = 15% And rate of interest, r =15%
We know, P+I = P + Pnr Time, n =?
Or, 43750 = P (1+nr) We know that, I = Pnr
Or, (1+5×15%) = 43750 Or, 30000 = 25000×15%×n
×
Or, P = = 25000 Or, n = =8
. ×
Answer: 8 Years

Compound
Finding principal
1) ‡Kv‡bv Avmj 3 eQ‡i Pµe„w× g~‡j 6690 UvKv Ges 6 eQ‡i 10035 UvKv n‡j, Avmj KZ? [BB AD
2024]
mgvavb
awi, Avmj = K UvKv Ges my‡`i nvi = L%
3 eQ‡ii Pµe…w× g~j = 6690 UvKv Ges 6 eQ‡i 10035 UvKv n‡j, m~Îg‡Z cvB
K×(1+L%)3 = 6690_______ (1)

80
[CAREER AID FOR BANK AND BCS] Writtten math

Ges K× (1+L%)6 = 10035______ (2)


K × (1+L%)6 10035 3 3
3 = 6690 = 2 ev, (1+L%) = 2
3
mgxKiY (2) ‡K (1) bs `¦viv fvM K‡i cvB,
K×(1+L%)
3
GLb, (1+L)3 = 2 [(1) bs mgxKi‡Y ewm‡q cvB]
3
ev, K ×2 = 6690
2
ev, K = 6690×3 = 4460
myZivs Avmj = 4460 DËi 4460 UvKv
2) Ann invested a certain sum of money in a bank that paid simple interest. The
amount grew to $240 at the end of 2 years. She waited for another 3 years and got
a final amount of $300. What was the principal amount that she invested at the
beginning? [NCC JO 2022, UCB PO – 2017]
Solution
Amount at end of 2 years = $240
Amount at end of 5 years = $300
Difference in amount for time period of 3 years = $60
60
Annual Interest earned = = $20
3
Principal amount invested at the beginning = 240 ‒ 2×20 = $200
া া:
আসল + 5 বছেরর নাফা = 300
আসল + 2 ,, ,, = 240
‒‒‒‒‒‒‒‒‒‒‒‒‒‒‒‒‒‒‒‒‒‒‒‒‒‒‒‒‒‒‒‒‒‒
[(‒) কের ] 3 বছেরর নাফা = 60
60
∴ 2 ,, ,, = 3 ×2 = 40
∴ Principal = 240 ‒ 40 = 200 টাকা
Answer: Tk 200
3) A sum of money lent out at simple interest amounts to Tk. 720 after 2 years and to Tk.
1020 after a further period of 5 years. Find the sum [SBC AM 2019]
Solution:
Here, Sum of money after two years = tk 720 Sum of money for 2 years= P+2I
Sum of money after 7 years = tk 1020 Sum of money for 7 years = P+7I
S.I for 5 years = 1020 - 720 = 300 (-) Interest for 5 years = 7I – 2I
S.I for each year = = 60
Amount after 2 years = 720
S.I for 2 years = 120
Hence Principal = 720 - 120 = tk600
Answer: tk600

81
[CAREER AID FOR BANK AND BCS] Writtten math

Finding time
1) What will be the deposited amount at initial stage, if it becomes to Tk. 33,500 at the
end of 5 years with a simple interest rate 13.5% per annum? How many years it will
take to become Tk. 40,600 of the said deposited.[SEBL TO 2018]
Solution:
Here,
Rate of interest, r =13.5%
Time, n = 5 years
Future value, FV = 33500
Principal, P=?
FV= P+Pnr = P (1+nr) = 33500
Or, P(1+5×13.5%) = 33,500
Or, P = 20000
Now,
20000(1+13.5%×n) = 40600
or, 13.5%×n = 1.03
Or, n = 1.03×100/13.5 = 7.63
Answer: 7.63
Finding rate of interest
1) A sum of taka 725 is lent in the beginning of a year at a certain rate of interest. After 8
months, a sum of tk 362.50 more is lent but at the rate of twice the former. At the end of
the year Tk 33.50 is earned as interest from both the loans. What was the original rate of
interest?
Solution:
Let, Initial Rate = r%
Final rate = 2r%
Time for tk 725, = 1 year
Time for tk 362.5, = = year
According To The Question,
× × + × × = 33.50
Or, 725×1× + (362.5) × × = 33.50
Or, 725r+ 241.67r= 3350
Or, r = 3.46%
Answer: 3.46%
3) A sum has been invested at a certain interest rate. Then amount the person
gets after 5 years is 9800 and after 8 years is 12005. At what percent the person
has invested the money?
Solution
Principal + Interest for 8 years = Tk 12005
Principal + Interest for 5 years = Tk 9800
‒‒‒‒‒‒‒‒‒‒‒‒‒‒‒‒‒‒‒‒‒‒‒‒‒‒‒‒‒‒‒‒‒‒‒‒‒‒‒‒‒‒‒‒‒‒‒‒‒‒‒‒‒
Interest for 3 years = Tk (12005 ‒ 9800) = Tk 2205

82
[CAREER AID FOR BANK AND BCS] Writtten math

5
Or, Interest for 5 years = Tk 2205× = Tk 3675
3
Hence, principal for 5 years = Tk (9800 ‒ 3675) = Tk 6125
Here,
Principal = Tk 6125
Interest, I = 3675
Time, n = 5 years
Rate of interest r% =?
We know that.
I = Pnr
I 3675
Or, r = = ×100%
Pn 6125×5
Or, r % = 12%
Answer: 12%
Compound Interest
Finding future value
1) Amit deposited some money in a bank, which pays 15% interest per annum
compounded yearly. If the bank provides simple interest instead of compound interest,
he receives Tk. 2400 after 2 years, Find the total amount that he received after 2 years.
Solution:
Here, Interest, I = 2400
Rate of interest, r = 15%
Time, n = 2 years
Principal, P =?
We know,
I = Pnr
Or, 2400 = P×15% ×2
Or, P = 8000
Now, for compound interest, FV = P ( 1 + %) = 8000×(1 + 15%) = tk 10,580
Answer: tk10,580
2) If the compound interest on a certain sum of money for 3 years at 10% be Tk. 993,
what would be the simple interest?
Abyev`t wbw`©ó UvKv 3 eQ‡i 10% nv‡i Pµe„w× gybvdvmn 993 UvKv n‡j, mij gybvdvq KZ UvKv n‡e?
Solution
Here, Future value, my`vmj = Tk. 993
Time, n = 3 years
Rate of interest, r = 10%
Principal, p =?
We know for compound interest
Future value = P (1 + r%)n
Or, 993 = P (1 + 10%)3
10 3
Or, 993 = P(1 + )
100
3
Or, 993 = P(1.1)

83
[CAREER AID FOR BANK AND BCS] Writtten math

993
Or, P = = 746
1.331
We know for simple interest
Future value = Interest + Principal
Or, Future value = Pnr + Principal
Or, Future value =P(1 + nr)
Or, Future value =746 × (1 + 3×10%)
Or, Future value = 970
Another way
So, P: CP = 1000:1331
Here, At 10% CI, P: CP=10:11
Atq, 331 unit equivalent to 993 Tk
So, 1000
3000 Tk
So, At 10% SI.3000 ×x3 = 900 Tk
Answer900 Tk
3) Rima invests Tk. 5000 in a bond which gives interest at 4% per annum during the first
year, 5% during the second year and 10% during the third year. How much does he get
at the end of third year?
Solution
Here, rate of interest for first year, r1 = 4%, second year r2 = 5% and third year r3 = 10%
Principal, P = Tk. 5000
After first year After second year After third year
Interest 5000×4% = Tk.200 (5000 + 200)×5% = Tk. 260 (5200 + 260)× 10% = 546
he gets at the end of third year = 5000 + 200 + 260 + 546 = Tk. 6006
Solution Tk. 6006
Mixed
1) The difference between simple & compound interest annually on same amount at 8%
for 2 years is Taka 12.80, what is the principal amount? [NCC Bank MTO-2018]
Solution:
The Principal of simple & Compound be =x
First case: Simple Interest
SI= {(x×8×2)/100} = 4x/25
Second Case:
Compound Interest
={x(1+8/100)2}-x
= x{(1+2/25)2}-x
=x(27/25)2-x
=(729x/625)-x
=(729x-625x)/625
=104x/625

84
[CAREER AID FOR BANK AND BCS] Writtten math

According to the question,


(104x/625)-(4x/25)=12.80
Or,104x-100x=625×12.80
Or, 4x=8000
Or, x=2000
Hence, the principal is both cases 2000 tk Answer: tk 2000
2) Find the interest rate where difference between compound and simple interest on a
principal of tk 20000 for 2 years is tk 392
Solution:
Here, Principal, P = tk 20,000
Time, n = 2 years
Interest rate, r =?
According to the question
Compound interest - simple interest = 392
{P(1 + ) – P} – Pnr = 392
Or, {20000(1 + ) − 2000} - 20000×2×r = 392
Or, 20000(1 + 2r + − 1- 2r) = 392
Or, 20000× = 392
Or, = = 0.0196
Or, r = 14 Answer: 14%
3) There is 60% increase in an amount in 6 years at simple interest. What will be
the compound interest of Rs. 12,000 after 3 years at the same rate?
Solution
Let, Principal amount is tk P and rate of interest is r
Here, time n = 6

According to the question,


P×6×r% =Interest = 60%×P
Or, r = 10%
Now, Compound interest for 3 years at the rate 10% = 1200(1 + 10%)3– 1200
10 3 110 3 11 11 11
= 1200(1 + ) – 1200 = 1200( ) – 1200= 12000× × × ‒ 12000
100 100 10 10 10
= 12(11×11×11 – 1000) = 12(1331 – 1000) = 12×331 = 3972
Answer: Tk 3972

85
[CAREER AID FOR BANK AND BCS] Writtten math

Time and work


Type: Basic
1
1) Jalil hires 10 workers to construct a road within 100 days. After 20 days if th of the
4
work is done and Jalil fires 2 workers in how many more days will work be finished?
[DBBL, AO – 2024]
Solution
Method 1:
Total unit of the work is done = 10×20×4 = 800
1
If th of the work is done then work remain = 10×20×3= 600 units
4
600
Therefore, 150 units of the work have to be finished by (10 ‒ 2) = 8 workers in = 75 days
8
Short cut
M1 = 10 M2 = 10 ‒ 2 = 8
D1 = 20 D2 =?
1 3
W1 = W2 =
4 4

M1× D1 M2× D2 10×20 8×D2 4


Formula, = Or, = Or, 10×20×4 = 8× D2× Or, D2
W1 W2 1 3 3
4 4
3
= 800× = 75
32
Answer 75 days
2) A reads 18 pages per hour, starts at 12.30pm. B reads 30 pages per hour starts at 2.00
pm. B is 3 page behind from A when starting. At what time will they be reading the same
page?
Solution
Time A B
From 12.30 to 2 pm In 1.5 hours A reads = 18×1.5 B reads = 0 page
= 27 pages
At 3 pm A reads = 27+18 = 45 pages B reads = 30
At 4 pm A reads = 45+18 = 63 pages B reads = 30 + 30 = 60 pages
At 4:30 pm A reads = 63 + 9 = 72 pages B reads = 60 + 15 ‒3 = 72 pages
Another way
From 12.30 to 2 pm, in 1.5 hours A reads = 18×1.5 = 27 pages
At 2 pm B is behind of A = 27 + 3 = 30 pages
At 2 pm A ahead’s = 30 ‒ 18 = 12 pages
30
To cover 30 pages B needs = = 2.5 hours
12
They read same page at 2pm + 2.5 hours = 4:30 pm Answer 4:30 pm

86
[CAREER AID FOR BANK AND BCS] Writtten math

Alone
1) Aryan and Rayan together can do a piece of work in 12 days, which Rayan and Arko
can do in 16 days. After Aryan has been working at it for 5 days, and Rayan for 7 days,
Arko finishes it in 13 days. In how many days Arko alone will do the work?
Solution
1
Aryan and Rayan together can do in 1 day = parts of the job
12
1
Rayan and Arko together can do in 1 day = parts of the job
16
5
5 days’ work of Aryan and Rayan = parts
12
2
2 days’work of Rayan and Arko = parts
16
5 2 5 1 20 + 6 26
7 days’ work = + = + = = parts
12 16 12 8 48 48
26 48 - 26 22 11
Rest of the work = 1 - = = = part
48 48 48 24
Arko finished the remaining work in 11 days
24
Hence, Arko needs = ×11 = 24 days
11
Answer: 24 days
2) Sohel and Suman can assemble a car in 24 days. Suman and Sajib can do the same job
in 32 days. After Sohel alone worked on it for 10 days, Suman alone worked for 14 days
and then Sajib finished the rest in 26 days. In how many days, Sajib alone can assemble a
car? [DBBL, MTO – 2024]
Solution
1
Sohel and Suman can do in 1 day = parts
24
1
Suman and Sajib can do in 1 day = parts
32
10
10 days work of Sohel and Suman = parts [10 w`‡bi KvR †mv‡nj I mygb GKmv‡_ Ki‡&e]
24
4
4 days work of Suman and Sajib = parts [4 w`‡bi KvR †mv‡nj I mygb GKmv‡_ Ki‡&e]
32
10 4 40 + 12 52
In 14 days total work done = + = =
24 32 96 96
52 44
Rest of the work = 1 − = parts
96 96
96 96
Rest of work done by Sajib ×(26 – 4) = × 22 = 48 days
44 44
96
Sajib alone can assemble a car = = 48 days
2

87
[CAREER AID FOR BANK AND BCS] Writtten math

Answer 48 days
3) A and B together can complete a piece of work in 3 days. They start together. But,
after 2 days, B left the work, If the work is completed after 2 next days. B alone could do
the work in
Solution
1 1 1
Here, 1 day work of A and B = + = parts
A B 3
2
2 day’s work = parts
3
2 1
Rest of the work = 1 ‒ = parts
3 3
1
parts work done by A in 2 days
3
1‒part work done by A in 2×3 = 6 days
Therefore, A complete a piece of work in 6 days
1 1 1
Now, + =
6 B 3
1 1 1
Or, = ‒
B 3 6
1 (2-1) 1
Or, = =
B 6 6
Hence, B complete a piece of work in 6 days
Answer: 6 days
4) A and B together can complete a piece of work in 3 days. They start together. But,
after 2 days, B left the work, If the work is completed after 2 next days. B alone could do
the work in
Solution
1 1 1
Here, 1 day work of A and B = + = parts
A B 3
2
2 day’s work = parts
3
2 1
Rest of the work = 1 - = parts
3 3
1
parts work done by A in 2 days
3
1-part work done by A in 2×3 = 6 days
Therefore, A complete a piece of work in 6 days
1 1 1
Now, + =
6 B 3
1 1 1
Or, = −
B 3 6
1 (2-1) 1
Or, = =
B 6 6
Hence, B complete a piece of work in 6 days Answer: 6 days

88
[CAREER AID FOR BANK AND BCS] Writtten math

Additional time
1) Machine A takes 2 hours to make a car, machine B takes 3 hours to make the same
car. Working both A and B together for 1 hour, machine B broken. How much
additional time machine A takes to make the car?
Solution
(3 + 2) 5
Working 1 hour A and B do = parts
(3 × 2) 6
5 1
Rest of the work = 1 ‒ = part which is done by A
6 6
1 2
A does part of the work in × 60 = 20 minutes
6 6
Answer 20 minutes
2) A signed a contract for building a road of 1920 meters long within 120 days. He
employed 160 workers for this task. But after 24 days he found that only of the task has
been finished. If A wants to finish the road in time how many additional workers he has
to employee? [UCBL – (PO)-2017 (Written)]
Solution:
Days remaining=120-24=96 × ×
Work done=1920/8=240 meter =
Work remaining =1920× =1680 meter × ×( )
Or, =
240 meter 24 days=160 men
1680 meter 96 days=160×24×1680/ (240×96) =280 Or, M = 280
men Additional man required = 280 –
Additional men need=280-160=120 160
= 120
Answer: 120
3) 20 men can finish a work in 30 days. After how many days should 5 men leave the
work so that the work may be finished in 35 days? [BKB-(SO)-2017(Written)]
Approach -1:
20×30 = x×20 + 15× (35-x)
Or, 600 = 20x+525-15x
Or, x = = 15
Answer: 15 days
Approach -2
Total work = 20×30 = 600 units
15 men finished work = 15×35 = 525 units
Remaining work = 600-525=75 units
So 5 men leave the work finished in = = 15 days
Answer: 15 days
4) A contractor undertook to do a certain piece of work in 40 days. He engages 100 men
at the beginning and 100 more after 35 days and completes the work in stipulated time. If
he had not engaged the additional men, how many days behind schedule would it be
finished? [Janata Bank (EO)-2017(Written)]

89
[CAREER AID FOR BANK AND BCS] Writtten math

Solution
Let, work be done in x days
According to the question,
100×35+ (100+100) ×5= 100×x
Or, 3500+1000 =100x
Or, x = 45
Behind days 45 - 40= 5
Answer: 5 days
Another way
Let, Total work be = 100×35+ (100+100) ×5 = 4500 units
To complete the work 100 men would take = 45 days.
The work would have finished (45-40) or 5 days behind scheduled time.
Answer: 5 days
5) 50 daily workers can complete a dam project in 40 days. If 30 of them work daily and
the rest work in every alternative day, how many more days will be required to complete
the project?
Solution
50 workers do in 40 days = 2000 unit works
30 work daily, the other 20 work on alternate days. This gives you a daily work force of 30 +
= 40 workers daily
40 workers do 2000 unit works in = 50 days
Extra days needed to complete the work = 50–40 =10 days
Answer: 10 extra days
6) Working alone a construct rate Machine A takes 2 hours a built a car while machines
B takes 3 hours to build the same car. If they work together for 1 hour and then
machines B breaks down. How much additional time will take machines A to finish the
Car by itself?
Solution
2A = 3B
Or, A : B = 3 : 2 [Efficiency]
Total work = 6 Unit
According to the question,
(3+2)× 3T = 6
Or, 3T = 6‒5 = 1
Or, T = 1/3 hour = 20 min Answer20 min
7) A, B and C can do a piece of work in 10, 30 and 60 days respectively. A is assisted by B
and C on every alternative day. How long would the work take to finish?
Solution
1 1 3+1 4
In first day A and B’s work = + = = parts
10 30 30 30
1 1 6+1 7
In second day A and C’s work = + = = parts
10 60 60 60

90
[CAREER AID FOR BANK AND BCS] Writtten math

4 7 8+7 15 1
Consecutive 2 day’s work = + = = = parts
30 60 60 60 4
1 3
Rest of the work = (1 - ) = parts
4 4
3
Next 6 day’s work = parts
4
Required time to finish the job = 8 days Answer: 8 days
8) A pipe can fill an empty tank in 15 minutes. Another pipe flows out 20 liters per
minute. It the two pipe open together and fill the tank in 135 minutes, how much tank
contains?
Solution
Let, second pipe fill the tank in y minutes
1 1 1
− =
15 y 135
y - 15 1
Or, =
15y 135
Or, 135(y – 15) = 15y
Or, 135y - 15y = 135×15
Or, 120y = 2025
2025
Or, y =
120
⸫ y = 16.875
Now, The capacity of the tank = 20×16.875= 337.5 liters Answer: 337.5 liters
9) Twenty-four men can complete a work in sixteen days. Thirty-two women can
complete the same work in twenty-four days. Sixteen men and sixteen women started
working for twelve days. How many more men are to be added to complete the work
remaining work in 2 days? [PBL SO/Officer-2017]
Solution:
Here,
24 men can complete in 16 days 1 portion work
1 man can complete in 1 day = × = portion
Again,
32 women can complete in 24 days 1 portion
1 woman can complete in 1 day = = portion
×
Both,1man +1 woman work in 1 day = ( + )
16 men +16 women work 12 days
× ×
=( + )portion = ( + ) portion = portion
Remaining work =1 - = portion
Let,X more men should be added then the remaining work complete in 2 days
Total men =(16+x)

91
[CAREER AID FOR BANK AND BCS] Writtten math

1 man in 1 day done = portion


( )×
(16+x) men in 2 days done = portion
×
Similarly, 16 women in 2 days done = portion
According to the question,
( )× ×
+ =
Or,(32+2x)/384 =1/4-32/768
Or,(32+2x)/384 = (192-32)/768
Or,(32+2x)/384 = 160/768
Or, 32+2x = 160/2
Or,2x = 80 - 32
Or,2x = 48
Or, x = 24 Answer: 24 men

Both
1) Rahim can do a piece of work in 80 days. Rahim works for 10 days and Karim alone
finishes the rest of the work in 42 days. How much time would it take for the two of them
together to complete the whole work? [Bank ASIA MTO -2017]
Solution
Let, the together finish the work in x days
Total portion work =1
Rahim 80 days did =1 portion
Rahim 10 days did = = portion
Remaining work = (1- ) = portion
Karim did portion in 42 days
Karim 1 or full portion work did = 42× = 48 days
×
Both Rahim & Karim did the job together in = ( ) = 30 days Answer: 30 day
(
2) M can do a piece of work in 40 days. He worked at it for 5 days, and then N finishes
the remaining work in 42 days. The number of days that A and B take together to finish
the work are? [PBL TJO 2020]
Solution
In 5 days M does = parts work
Remaining work = 1- = parts
N does parts work in 42 days
" " " full " " = 42× = 48 day
×
The do together = = = = 21( ) days = 22days
( )
Answer: 22 days
3) Two pipe can fill a cistern in 3 hours and 4 hours respectively and a waste pipe can empty
it in 2 hours. If all the three pipes are kept open, then the cistern will be filled in‒

92
[CAREER AID FOR BANK AND BCS] Writtten math

Solution
1 1 1 (4 + 3 - 6) 1
Cistern will be filled in = + ‒ = =
3 4 2 12 12
Hence, the cistern will be filled in 12 hours Answer: 12 hours
Total time
1) Rafiq, Shafiq, and Arif can alone complete a project in 10 days, 20 days, and
10 days respectively. Rafiq started working on the project alone. Shafiq joined
the project after 2 days. After working together 4 days both Rafiq and Shafiq left
the project and Arif got in. How many days it took to complete the entire project?
[BKB- (officer)-2017 (Written)]
Solution:
Rafiq can do in 1 day = of the work
Shafiq can do in 1 day = of the work
Arif can do in 1 day = of the work
Rafiq first 2 days work done = portion = portion
Remaining work = (1- ) portion = portion
Both Rafiq & Shafiq 4 days work = + = portion
Remaining work = - =
Arif full work done in 10 days
Arif portion work done in =10× = 2 days
So, total time taken to complete the whole work = (2+4+2) =8 days
Answer: 8 days
2) Jaya can make 40 pancakes in a minute. Sally can make pancakes at half of
Jaya's rate. What time will it need (in minute) to make 150 cakes, if sally have
already made a start of 30 pancakes alone? [NCC Bank MTO-2018]
Solution:
Jaya can makes 1 minutes 40 pancakes
According to the question,
Sally can makes 1 minutes 20 pancakes
[Since Sally's Speed is half of Jaya]
Sally 20 pancakes made in 1 minutes
So, Sally 30 pancakes made in = =1.5 minutes
Remaining pancakes = (150 - 30) =120
Both Sally & Jaya 1 minutes done = (40+20) = 60 pancakes
So, Both 120 pancakes made in = =2 minutes
Hence, total time taken to make 150 pancakes = (1.5+2) = 3.5 minutes
Answer: 3.5 minutes

93
[CAREER AID FOR BANK AND BCS] Writtten math

3) A, B, C can complete a piece of work in 16, 32, 48 days respectively. They


started working together but C left after working 4 days and B left 2 days before
the completion of the work. How many days require completing the work?[Social
Islamic Bank-PO-207][Lanka Bangla Finance Ltd.MTO-2017]
Solution:
Let, the work finishes in x days
According to the question
+ + =1
Or, 6x+ 3x-6+8 = 96
Or, 9x = 94
Or, x = = 10.44 Answer: 10.44 days
4) Working together, Marjana and Jimi can constructed a house in 12 hours and paint it
in 7.5 hours. If Marjana needs alone 30 hours to construct and Jimi need 12 hours to
paint it alone. In total, will it take for Jimi to construct the house alone and then
Marjana paint it alone?
Solution
For Construction,
(M + J)12 = 30M
Or, (M + J): M = 5: 2 [Efficiency]
So, Jimi’s Efficiency = (5‒2) = 3 Unit / hour
Total work = 5×12 = 60 Unit
Required time to construct the work = 60/3 = 20 hour
Again, For Painting
(M + J)7.5 = 12J
Or, (M + J)15/2 = 12J
Or, 15(M+J) = 24J
Or, (M+J) : J = 8 : 5 [Efficiency]
So, Marjan Efficiency = (8 ‒ 5) = 3 Unit/hour
Total work = 12 × 5 = 60 Unit
60
Required time for Marjan to Paint = = 20h
3
Total Time = 20h + 20h = 40 hour Answer40 hour
Assistance
1) A, B and C can do a piece of work in 10, 30 and 60 days respectively. A is assisted by B
and C on every alternative day. How long would the work take to finish?
Solution
1 1 3+1 4
In first day A and B’s work = + = = parts
10 30 30 30
1 1 6+1 7
In second day A and C’s work = + = = parts
10 60 60 60
4 7 8+7 15 1
Consecutive 2 day’s work = + = = = parts
30 60 60 60 4

94
[CAREER AID FOR BANK AND BCS] Writtten math

1 3
Rest of the work = (1 ‒ ) = parts
4 4
3
Next 6 day’s work = parts
4
Required time to finish the job = 8 days
Answer: 8 days
2) A can do a piece of work in 8 days which B can finish in 12 days. If they work at it on
alternate days with A beginning, In how many days, the work will be finished?
Solution
1 2 3 4 5 6 7 8 9
A B A B A B A B A
In 2 days A and B can do the = + = parts of the job
8th day they together do = ×4= parts works
9th day they together do = + = parts works
Remaining work = 1 - = parts
Parts work done by B in = days
Total days = 9 days
Answer: 9.5 days
3) A & B working separately can do a piece of work in 9 days and 12 days respectively. If
they work for a day alternately, A beginning, in how many days, the work will be
completed?
Solution
In 2 days they did + = part
In 10 days they did = 5× part = part
Rest of the work = 1 - part = part
Part did by A in = day
Therefore total amount of time taken is 10 + = days or 10 days and 6 hours.
Answer: days
4) A can do a piece of work in 30 days, B can do it in 15 days and C can do it in 10 days.
If in every second day B and in every third day C help A in doing the work, how many
days will be required to complete the whole work?
Solution
A
1 2 3 4 5 6 7 8 9 10 11
wor
A AB AC AB A ABC A AB AC AB A ks
1/30
parts in 1 days
AB work 1/30+1/15 = 3/30 parts in 1 days

95
[CAREER AID FOR BANK AND BCS] Writtten math

AC work 1/30+1/10 = 4/30 parts in 1 days


ABC work 1/30+1/15+1/10 = 6/30 parts in 1 days
Day 1: A works alone: 1/30 = 1/30 completed
Day 2: A and B work together, so (1/30) + (1/30 + 1/15) = (1/30) + (1+2)/30 = (1/30) + (1/10)
= 4/30 completed
Day 3: A and C work together, so (4/30) + 4/30 =8/30 completed
Day 4: A and B work together, so (8/30) + 3/30 = 11/30 completed.
Day 5: A works alone: (11/30) + (1/30) = 12/30 completed
Day 6: A, B and C work together, so (12/30) + 6/30 = 18/30 completed
Day 7: A worked alone, so (18/30) + (1/30) = 19/30 completed
Day 8: A and B work together: (19/30) + 3/30 = 22/30 completed
Day 9: A and C work together, so (22/30) + 4/30 = 26/30 completed
Day 10: A and B work together, so (26/30) + 3/30= 29/30 completed
Day 11: A works alone: (29/30) + (1/30) = 30/30 completed
Answer: 11 days
Wage
1) If Arif can do a work in 4.50 hours and Jamal can do this work in 3 hours. If they
got total tk. 450 then what amount Jamal gets? [Avwid GKwU KvR 4.5 N›Uvq I Rvgvj GKwU KvR
3 N›Uvq m¤úbœ K‡i| hw` Zviv me©‡gvU 450 UvKv cvq Z‡e Rvgvj GKv KZ UvKv cvq]

jwRK †W‡fjcg‡›Ut Avwid I Rvgvj GKwU KvR K‡i GK‡Î 450 UvKv cvq| Avwid I Rvgvj Avjv`vfv‡e KZ UvKv
cv‡e Zv wbf©i Ki‡e Zv‡`i Kg©`ÿZvi Dci| Kg©`ÿZv †hfv‡e wbY©q Ki‡eb? Avwid GKwU KvR 4.5 N›Uvq m¤úbœ
1 1
K‡i A_©vr Avwid 1 N›Uvq m¤úbœ K‡i 4.5 Ask KvR| wVK †Zgwbfv‡e Rvgvj 1 N›Uvq m¤úbœ K‡i
3 Ask KvR

Solution
From the condition
1 1 2 1
Arif: Jamal = : = :
4.5 3 9 3
2
Or, Arif: Jamal = = 2: 3
3
Ratio total = 2 + 3 = 5
Jamal gets = Jamal’ partial × Total amount
Jamal's portion
Or, Jamal gets = × Total amount
Ratio total
3
Or, Jamal gets = × 450 = Tk. 270
5
Answer Tk. 270
Tank and machine
1) Of the 3 machines Company X uses for production, Machine A can produce 30
units in 5 hours, Machine B can produce 6 units in 12 hours, and Machine C can
produce 33 units in 12 hours. Of the 30 hours, the company uses the facility, the
committee decided they would operate Machine B and C for 2/5 of the total time

96
[CAREER AID FOR BANK AND BCS] Writtten math

and Machine A and B to produce 39 units. Given the committee decided to


operate Machine A and C for the rest of the time, what is the total units produced
by the company? [DBBL, TO – 2024]
Solution
30 6 33
Efficiency of three machines A, B and C = = 6, = 0.5 and
5 12 12
2
Machine operating time = 30× = 12 hours
5
33
Total production produced by B and C = 12×0.5 + 12× = 6 + 33 = 39 units
12
39
To produce 39 units machine A and B needs = = 6 hours
6+0.5
Rest of time = 30 ‒ (6 + 12) = 12 hours
33
In 12 hours A and C produce = 12×6 + 12× = 72+33 = 105
12
Hence, total units produced by the company 39 + 39 + 105 = 183 units
Answer 183 units
1
2) A ship 77 km from the shore springs a leak which admits 2 tones of water in
4
1
5 minutes. 92 tones of water would sink it. But the pumps can throw out 12 tones
2
of water per hour. Find the averages rate of sailing so that the ship may just
reach the shore as it begins to sink. [Janata Bank PLC, RC – 2024]
Solution
2.25×60 45×6 270
Here, In 1 hour Incoming Water to Ship = = = ton
5.5 11 11
270 138
So, In 1 hour water fills to the ship = ‒ 12 = ton
11 11
92×11 22
Required time to save 92 tones of water = =
138 3
77 3
So, average rate of sailing the ship = = 77× = 10.5 km
22 22
3
Answer10.5 km
Practice
1) Raju can finish a work in 15 hrs & Saju can finish the same work in 9 hrs. They work
together first at 6 am but Saju stopped at 9 am. What the time to finish the work by Raju?
Answer: 4:00pm
2) A and B can separately do a piece of work in 20 and 15 days respecttively. They worked
together for 6 days, after which B was replaced by C. If the work was finished in next 4 days,
then the number of days in which C alone could do the work will be?

97
[CAREER AID FOR BANK AND BCS] Writtten math

Answer: 40 days
3) A alone can do a piece of work in 20 days, while B alone can do it in 30 days and C alone
can do it in 60 days. If in every third day B in every fourth day C help A in doing the work,
how many days will be required to complete the whole work? [34rd & 32 th BCS Written
Exam]
Answer: 15.5 days
4) A can do a work in 20 days and B can do it in 25 days. They started together and after some
times B left. Remaining work is finished in 15 days. Find how when B left the workplace?
Answer: 25/9 days

98
[CAREER AID FOR BANK AND BCS] Writtten math

Partnership
Ratio of profit/investment
1) A, B and C enter into a partnership. A initially invests Tk. 25 lakhs and adds another
Tk. 10 lakhs after one year. B initially invests Tk. 35 lakhs and withdraws Tk. 10 lakhs
after 2 year and C invests Tk. 30 lakhs. In what ratio should the profits be divided at the
end of 3 year? [Meghna Bank-MTO -2016]
Solution:
The profit should be divided according to the investment ratios
A: B: C= (25×1+35×2) :(35×2+25×1): (30×3) = (25+70): (70+25): 90 = 95: 95 :90 = 19:19:18
Answer: 19: 19: 18
2) A, B and C enter into a partnership They invest Rs 40000, Rs 80000 and Rs 120000
respectively At the end of the first year B withdraws Rs 40000 while at the end of the second
year C withdraws Rs 80000 In what ratio will the profit be shared at the end of 3 years?
Abyev`t A, B Ges C G h_vµ‡g 40000 UvKv, 80000 UvKv I 1,20,000 UvKv GKwU g~jabx Kviev‡i wewb‡qvM K‡i|
eQi †k‡l A UvKv B 40000 UvKv Ges C UvKv D‡Ëvjb Ki‡j 3 eQi ci Zv‡`i jf¨vsk KZ Abycv‡Z wef³ n‡e
Solution
Their Profit ratio, A: B: C= (40000×36):(80000×12+40000×24):(120000×24+40000×12)
=1440000:1920000:3360000 =144:192:336 =12:16:28 =3:4:7
Answer3:4:7
3) Three partners shared the profit in a business in the ratio 5:7:8. They had partnership
for 14 months, 8 months and 7 months respectively. What was the ratio of their
investment? [Combined 8 Banks & FI’s (SO) –-2019 (Cancelled) (AUST), Somobay Bank OFF
2015]
Solution:
Let, the ratio of their investment = X: Y: Z
After equating the profits, we left with,
14X: 8Y: 7Z = 5: 7: 8 [Formula]
Now, X = , Y = , Z = Another way
LCM of 14, 8, and 7 is 56 Ratio of Pro it
Ratio of Investment =. Ratio of Time
Now,
Investment of X = × 56 = 20 So, Ratio of Investment = : :
Investment of Y = × 56 = 49 = 20: 49: 64
Investment of Z = × 56 = 64
So, the ratio of their investments X: Y: Z = 20: 49: 64
Answer: 20: 49: 64
1) Abu invested 70000 taka. After 6 months Rabu invested 105000 taka. Sabu
invested 140000 taka after another 6 months. In which ratio they will distribute
profit in 3 years from Abu's investment?
Solution
Amount ratio =70,000: 105000: 140000 = 2:3:4

99
[CAREER AID FOR BANK AND BCS] Writtten math

Annual equivalent investment ratio of Abu: Rabu: Sabu = (2×36): (3×30): (4×24) =
12:15:16
Answer: 12:15:16
Ratio of time
1) Ram, Shyam and Mohan invested some money in partnership in the ratio 6:9:10. If
the ratio of profit earned by them is in the ratio 2:3:5, then find the ratio of time
duration of their investments.
Solution
Ratio of Time =
So, Ratio of Time, R: S: M = : :
= × 90: ×90: ×90 = 30: 30: 45 = 2:2: 3 Answer: 2:2: 3
Individual Investment/Profit/Share
1) If A: B =2: 3 and B: C = 5: 6, Amount is tk. 430 then B’s share?
Abyev`t hw` A: B =2: 3 and B: C = 5: 6 nq Ges UvKvi cwigvY 430 n‡j B Gi fv‡M KZ UvKv c‡owQj?
Solution
Here, A: B =2: 3 and B: C = 5:6
Now, A: B: C = 2×5: 3×5: 6×3 = 10: 15: 18
Ratio total = 10 + 15 +18 = 43
B's partial ratio 15
Hence, B’s share = B’s partial share × total amount = × 430 = ×430 = tk. 150
Ratio total 43
Answer tk. 150
2) A, B and C together start a business. The ratio of the investments of A, B and C is
0.125:0.75:0.25. After 8 months A adds thrice amount of his earlier investment and C
withdraws half of his earlier investment. At the end of the year, they earn a total profit of
Rs.5800. What is B's share in the profit?
Solution
Initial investment ratio, A: B: C = 0.125: 0.75: 0.25 = 125: 750: 250 = 1: 6: 2
New ratio = (1×8+4×4): (6×12): (2×8+1×4) = 24: 72: 20 = 6: 18: 5
Ratio total = 6+ 18 + 5 = 29
B’ share = ×5800 = tk 3600 [Ans]
Answer tk. 150
3) A and B started a business in partnership investing Rs. 20,000 and Rs. 15,000
respectively. After six months, C joined them with Rs. 20,000. What will be B's share in
the total profit of Rs. 25,000 earned at the end of 2 years from the starting of the
business.
Solution
Investment ratio = A:B:C = 20,000×2:15000×2:20000×1.5 = 40:30:30 = 8: 6: 6
Ratio total = 8 + 6 + 6 = 20
Here total profit, T = Rs 25,000
8
B’s share = individual fraction × total profit =Rs ×25,000 = Rs10,000
20
Answer: Rs 10,000

100
[CAREER AID FOR BANK AND BCS] Writtten math

A's share B's share


Tk. 1300 are divided among A, B, C, and D in such a way that = =
B's share C's share
C's share 2
= . What is A's Share?
D's share 3
Solution
Here, A’s share: B’s share = 2:3 = (2:3) ×4 = 8: 12
B’s share: C’s share = 2:3 = (2:3) ×6 = 12: 18 and C’s share: D’s share = 2: 3= (2:3) ×9 = 18:
27
So, A: B: C: D = 8:12:18:27

Ratio total = 8+12+18+27 = 65


8
A’s share = ×1300 = Tk. 160 Answer 160 Tk.
65
4) A, B and C enter into a partnership in the ratio (7/2): (4/3): (6/5). After 4 months, A
increases his share 50%. If the total profit at the end of one year be Tk. 21,600, then B's
share in the profit is:
Solution:
Investment ratio of A, B, C = (7/2) : (4/3) : (6/5) = 105 : 40: 36
Let, investment of A, B, C be = 105x, 40x, 36x
After increasing 50% A's share,
Profit Ratio,
A = 105x×4 + (105x×150 /100)×8 = 1680x
B = 40x×12
C = 36x ×12
A: B: C = 1680x: 40x×12: 36x×12 = 35: 10: 9
Now,
54 parts = 21600 tk
10 parts = (21600×100/54) = 4000 taka
Answer: B's share profit = 4000 taka.
5) Salaries of Ravi and Sumit are in the ratio 2:3. If the salary of each is increased by Rs.
4000, the new ratio becomes 40:57. What is Sumit's salary?
Solution
Let, Salary of Ravi is 2x & Sumit is 3x
2x + 4000 40
According to the question, =
3x + 4000 57
Or, 114x + 228000 = 120x + 160000
Or, 6X = 68000
Or, X = 11,333.33
Sumit's Salary = 3×11333.33 = 34000. Answer 34000
6) A, B and C enter into a partnership in the ratio (7/2): (4/3): (6/5). After 4
months, A increases his share 50%. If the total profit at the end of one year be
Tk. 21,600, then B's share in the profit is:
Solution:
Investment ratio of A, B, C = (7/2) : (4/3) : (6/5) = 105 : 40: 36

101
[CAREER AID FOR BANK AND BCS] Writtten math

Let, investment of A, B, C be = 105x, 40x, 36x


After increasing 50% A's share,
Profit Ratio,
A = 105x×4 + (105x×150 /100)×8 = 1680x
B = 40x×12
C = 36x ×12
A: B: C = 1680x: 40x×12: 36x×12 = 35: 10: 9
Now,
54 parts = 21600 tk
10 parts = (21600×100/54) = 4000 taka
Answer: B's share profit = 4000 taka.
7) A and B started a business in partnership investing Rs. 20,000 and Rs. 15,000
respectively. After six months, C joined them with Rs. 20,000. What will be B's share in
the total profit of Rs. 25,000 earned at the end of 2 years from the starting of the
business.
Solution
Investment ratio = A:B:C = 20,000×2:15000×2:20000×1.5 = 40:30:30 = 8: 6: 6
Ratio total = 8 + 6 + 6 = 20
Here total profit, T = Rs 25,000
8
B’s share = individual fraction × total profit =Rs ×25,000 = Rs10,000
20
Answer: Rs 10,000
8)A, B and C started a business by investing Tk 1,20,000, Tk. 1,35,000 and Tk. 1,50,000
respectively. Find the share of each out of an annual profit of Tk. 56,700[PBL CASH-
2016]
Solution:
Ratio of their investment: A: B: C = 120000:135000:150000 = 120: 135: 150 = 8: 9: 10
Sum of their Ratio = 8+9+10 =27
Share of profit;
A= (56700 × ) = Tk 16800
B = (56700 × ) = Tk 18900
C = (56700 × ) = Tk 21000
Answer: Tk 16800, Tk 18900 & Tk 21000
9) Two partner A and B have 70% and 30% shares respectively in a business. After
sometimes, a third partner C joined the business by investing tk. 10 lakh and thus having
20% share in the business. What percentage is the A's share now in the business?
Solution
Given that, Share of A: Share of B = 70%: 30% = 7:3
Sum of the ratio = 7+3 = 10
Here, 20% share is to be 10 lakh
10
1% “ “ “ "
20

102
[CAREER AID FOR BANK AND BCS] Writtten math

10 × 100
100% “ “ “ = 50 lakh
20
Share of A and B = (50 ‒ 10) lakh = 40 lakh
7
A's share = Total share × individual ratio = 40 × = 28 lakh
10
28 × 100
So % of A's share = = 56%
50
Answer 56%
10) A, B, C enter into partnership. A invests 3 times as much as B invests two-third of
what C invests. At the end of the year, the profit earned is Tk 6600.What is the share of
B? [PBL TAT -2017]
Solution:
Let, The amount of investment of C be Tk. x
So, the amount of investment of B be Tk
The amount of investment of A be = (3× ) =Tk 2x
The ratio of investment of A, B & C = 2x: : x = 6:2:3
Sum of the ratio = 6+2+3 =11
So, the amount of share = (66000× ) = Tk.1200 Answer: 1200 Tk
11) A alone can do a job in 12 days, while B alone can do it in 15 days, with the help of C
they can finish it in 5 days. If they are paid Tk. 960 for the whole work, what is the share
of A? [Janata Bank & Rupali Bank Officer, 2020]
Solution:
×
In 5 days (A+B) together did = 5× ( + ) = = part
Remain = part by C
A: B: C = : : = 25:20:15 = 5:4:3
Share of A= 960× = 400 Answer: tk 400
12) Three friends Alice, Bond and Charlie divide tk1105 among them. When tk 10, 20
and 15 are removed from the sums that Alice, Bond and Charlie received, the share of
the sums that they receive is in the ratio of 11: 18: 24. What did Charlie receive?
[Southeast Bank PO 2020]
Solution:
According to the question,
11x + 18x + 24x = 1105- 45
Or, 53x = 1060
Or, x =
Or. X = 20
So Charlie received = 24×20 + 15 = 495 tk. Answer: tk 48000
13) Mr Zahir received Eid bonus of tk 7875. He distributed the money among his three
daughters named Lipi, Rimi & Shahaz. Lipi got 3/5 of what Shahnaz got. Shahnaz got
double of what Rimi got. Determine the amount of each sisters got. [Bank Asia Trainee
officer 2020]

103
[CAREER AID FOR BANK AND BCS] Writtten math

Solution:
Here, Total tk = 7875.
Let, Rimi got = tk x
Shahnaz got = tk2x
Lipi got = tk 2x× =
According to the question,
x+ 2x+ = 7875
Or, = 7875
Or, x = 1875
So, Rimi = 1875tk
Shahnaz = 2×1875= 3750tk
×
And Lipi got = = 2250 tk. (Answer)
14)Two friends P and Q started a business investing in the ratio of 5: 6 R joined them
after six months investing an amount equal to that of Q's. At the end of the year, 20%
profit was earned which was equal to tk 98000.What was the amount invested by
R?[Bangladesh Bank IT-2016]
Solution:
Suppose,
The total profit be T Tk
Then, 20% of T = 98000
×
Or, T =
Or, T = 4,90,000
Total profit after one year = 4, 90,000 Tk
Let, the capitals of P, Q and R be 5x, Tk. 6x and Tk. 6x respectively.
Then, 5x×1+6x×1+6x × ½ = 490000
Or, = 490000
Or, 28x = 490000×2
Or, x = 35000
So, R's investment = (6×35000) = Tk 210000 Answer: Tk 2, 10,000
14) Shakil started a business investing Tk. 25,000 in 2009. In 2010, he invested an
additional amount of Tk. 10,000 and Raihan joined him with an amount of Tk. 35,000. In
2011 Shakil invested another additional amount of Tk. 10,000 and Jafor joined them
with an amount of Tk. 35,000. What will be Raihan’s share in the profit of Tk. 1, 50,000
earned at the end of 3-year from the start of the business in 2009?
Solution:
Investment ratio for 3-year tk. of Shakil: Raihan: Jafor = (25000×3+10000×2+10000):
(35000×2): 35000 = 3:2:1
So, the profit share of Raihan = Tk. 150000 × = Tk. 50,000.
Answer: Tk. 50,000
15) Dawood invested certain amount in three different schemes A, B and C with the rate
of interest 10% p.a., 12% p.a and 15% p.a. respectively. If total interest accrued in one

104
[CAREER AID FOR BANK AND BCS] Writtten math

year was Tk. 3200 and the amount invested in Scheme C was 150% of the amount
invested in Scheme A and 240% of the amount invested in Scheme B, what was the
amount invested in Scheme B?
Solution:
According to the question, investment ratio:
A: C = 100: 150 = 2: 3
B: C = 100: 240 = 5: 12
So, A: B: C = 8:5: 12
Suppose, Dawood invested Tk. 8x, Tk. 5x and Tk. 12x in the schemes A, B and C
respectively.
So, (8x×10%) + (5x×12%) + (12x×15%) = 3200
Or, 3.2x = 3200
Or, x = 1000
So, the amount invested in Scheme B was = Tk. 5x = Tk. 5×1000 = Tk. 5000.
Answer: Tk. 5000
16) Three partners A, B, and C start a business. Twice the capital of A is equal to thrice
the capital of B, and the capital of B is 4 times the capital of C. They share the profit in
the ratio of their capital. In a particular year, the gross profit is Tk. 250000 and the
administrative expenses are 20% of the gross profit. Find the share of profit of each
partner.
Solution:
According to the question, 2(Capital of A) = 3(Capital of B)
Or, (Capital of A): (Capital of B) = 3: 2
Again, (Capital of B) = 4(Capital of C)
Or, (Capital of B): (Capital of C) = 4: 1
So, (Capital of A): (Capital of B): (Capital of C) = (3×4): (4×2): (2×1) = 12: 8: 2 = 6: 4: 1.
Net profit of the business = (Gross Profit) - (Administrative Expenses)
= Tk. 250000 − (20% of Tk. 250000)
= Tk. 250000 − Tk. 50000 = Tk. 200000
So, the profit share of A = Tk. 200000 × = Tk. 109090.91
The profit share of B = Tk. 200000 × = Tk. 72727.27
The profit share of C = Tk. 200000 × = Tk. 18181.82
Answer: Profit share of A is Tk. 109090.91, B is Tk. 72727.27 and C is Tk. 18181.82.
17) A. B and C started a business jointly with a total amount of Tk.28000. A paid Tk.
4500 more than B and B Paid Tk. 7000 less than C. If the company made a profit of Tk.
5600, how much profit should C receive? [Al-Arafah Islami Bank (MTO)-2017]
Solution:
Let, in a business C paid = tk x
In a business B paid = tk (x-7000)
In a business A paid = B+4500 = tk(x-7000+4500) = tk(x-2500)
According to the question,
Their started amount = 28000
Or, x + x-7000 + x-2500 =28000
Or, 3x – 9500 = 28000

105
[CAREER AID FOR BANK AND BCS] Writtten math

Or, 3x = 28000+9500
Or, 3x = 37500
Or, x = = 12,500
Therefore, in a business C paid = tk 12,500
In a business B paid = tk (12,500-7000) = tk5, 500
In a business A paid = tk(x-2500) = tk. (12,500-2,500) = tk 10,000
Now, ratio of their investment = A: B: C = 10000:5500: 12500 = 100:55: 125 = 20:11:25
Ratio Total =12
Now, Profit receives by C = ×5600 = tk 2500 Answer: tk 2500
18) A, B and C inter into partnership. A invests 3 times as much as B invests and B
invests two-third of what C invests. At the end of the year, the profit earned is Tk. 6600.
What is the share of B? [UttaraBank (PO)-2017(Preliminary) + [Pubali Bank (Cash)-
2017(Written), Sonali Bank Officer-2018]
Solution
Let, the investment of C = tk X
The investment of B = C = X
The investment of A = 3B = 3× = 2X
Ratio of their investment = 2X: X: X = 6:2:3
Sum of the ratio = 6+2+3 = 11
Now the share of B = × 6600 = tk 1200 Answer: tk 1200
19) A, B and C enter into a partnership. A contributes one-third of the capital while B
contributes as much as A and C together contribute. If the profit at the end of the year
amounts to Tk.900, what would C receive?
Solution:
Let total capital = Tk.x
Then, A's capital = Tk.
B's capital = (A + C)'s capital
According to the question
(A + B + C)'s capital = x
Or, + (A + C)'s capital + C = x
Or, + +2C =x
Or, C =
Therefore, B = + =
Ratio of their capital = : : = 2: 3:1
Ratio total = 6
C will receive = × 900 = tk 150 Answer: tk 150
18) Riyad, Sami and Abir started a business jointly with a total amount of Tk.280. Riyad
paid Tk.45 more than Sami and Sami paid Tk.70 less than Abir. If the company made a
profit of Tk.56, how much profit should Sami receive?

106
[CAREER AID FOR BANK AND BCS] Writtten math

Solution:
Let, Abir capital = x
Sami capital = x-70
Riyad capital = x-25
According to the question
x+x-70+x-25=280
=> x = 125
Riyad: Sami: Abir =100:55:125 =20:11:25
×
So, Sami receive = =11 Answer: 11 taka
( )
19) A and B started a business with capitals of Tk.3000 and 4000 respectively. After 8
months, A invested Tk.2500 more in the business and 7 months after this, total profit
becomes Tk.980. Find the share of profit for each. [BKB: Officer-Cash – 2018]
Solution:
Ratio of capitals initially, A: B = 3000: 4000
After 15 month the ratio of capital of A and B = {3000×8+ (3000+2500) ×7}: 4000×15
= (24000+5500×7): 60,000 = (24000+38500): 60000 = 62500: 60000 = 25: 24
Sum of the ratio = 25+24 = 49
So, A’s share of profit = 980× = Tk 500
And, B’s share of profit = 980-500 = Tk 480
Answer: A’s share of profit = Tk 500 and B’s share of profit = Tk 480
20) Sumon and Shimul two friends started a business with Tk. 5,000 and Tk.4,000
respectively. After 3 months Sumon added Tk.1000 and simultaneously Dilip joined with
them with Tk.7000. What is the share of profit among them after one year if profit is
Tk.36000? [Janata Bank: IT Officer-2016]
Solution:
Equivalent amount of Sumon = 5000×3+ (5000+1000) × 9 = Tk. 69,000
Equivalent amount of Shimul = 4000×12 = Tk. 48, 000
Equivalent amount of Dilip = 7000×9 = Tk. 63,000
Sumon: Simul:Dilip = 69,000: 48,000: 63,000 = 69: 48: 63 = 23: 16: 21
Sum of the ratios = 23+ 16 + 21= 60
Profit share of,
Sumon = Tk (36000 × ) = Tk 13800
Shimul = Tk (36000 × ) = Tk 9600
Dilip = Tk (36000 × ) = Tk 12600
Answer: 13800 Tk, 9600 Tk and 12600 Tk.
21) A, B and C started a business each investing Tk.20000. After 5 months A withdrew
Tk.6000, B withdrew Tk.4000 and C invested Tk.6000 more. At the end of the year, a
total profit of Tk.69200 was recorded. Find the share of each. [Janata Bank – 2015]
Solution:
Share of A: B: C= {(20000 × 5) +(14000 ×7)}: {(20000×5) +(16000× 7): {(20000 ×5)+(26000
× 7)} =198: 212: 82
Ratio total = 198+212+282 = 692

107
[CAREER AID FOR BANK AND BCS] Writtten math

So, Profit share of “A” = Tk. x 69200 =Tk. 19,800


Profit share of “B” = Tk. x 69200 = Tk.212, 00
Profit share of “C” = Tk. x 69200 = Tk. 28,200 (Answer)
TIME
1) Ravi started a business by investing Tk.30, 000. Prateek joined the business after some
time and invested Tk.20, 000. At the end of the year, profit was divided in the ratio of
2:1. After how many months did Prateek joined the business?
Solution:
Let, Prateek joined for x months in the business.
We know that, Ratio of Investment =
Now, 30000×12:20000x = 2:1
180000
Then, x = = 9 months Answer: 9 months
20000
TOTAL PROFIT/INVESTMENT
1) A, B and C enter into partnership with investment in the ratio 5:7:8. If at the end of
the year A's share of profit is tk 42360. How much is the total investment? [Sonali Bank AE
(IT)-2016]
Solution:
Let, A' investment is 5x tk
B investment is 7x tk
C investment is 8x tk
Total investment = 5x + 7x + 8x = tk 20x
So, 5x = 42360
Or, x = 8472
Now, total investment = 20×8472= tk 169440 Answer: Tk 1, 69,440
2) Four milkmen rented a pasture. A grazed 24 cows for 3 months; B, 10 cows for 5
months; C, 35 cows for 4 months and D, 21 cows for 3 months. If A's share of rent is
Tk.720, find the total rent of the field. [Pubali Bank: SO–2016]
Solution:
Ratio of shares of A, B, C, D = (24 x 3): (10 x 5): (35 x 4): (21 x 3) = 72: 50: 140: 63.
Ratio total = 72+50+140+63 = 325
Let, Total rent of the field = T
Here, A’s share = Tk.720
Or, x T = 720
Or, T = 720 x = Tk. 3250
Hence, total rent of the field is Tk. 3250
Answer: Tk. 3250
3) A, B and C started a business by investing Tk.24000, Tk.32000 and Tk.18000
respectively. A and B are active partners and get 15% and 12% of total profit and
remaining profit is to be distributed among them in the ratio of their investment. If C got
total Tk.65700 as profit, what was the total amount of profit? [Combined 4 Banks – 2019]
Solution:

108
[CAREER AID FOR BANK AND BCS] Writtten math

A : B : C
Capital → 24000 : 32000 : 18000
24 : 32 : 18
12 : 16 : 9
Let the total profit =100x
Extra share of A = 100x×15% = 15x
Extra share of B =100x×12% =12x
Remaining profit = [100x − (15x+12x)] =73x
Let the total profit = 100x
According to the question,
∴ Share of C = ×9 =Tk. 65700
( )
Or, = Tk. 65700
Or, x = Tk. 3700
Hence, required profit = 100x = 100×3700 = Tk. 370000
Answer: Tk. 370000
4) A and B started a business with initial investments in the respective ratio of 18:7. After
four months from the start of the business, A invested Tk.2000 more and B invested
Tk.7000 more [each month]. At the end of one year, if the profit was distributed among
them in the ratio of 2:1 respectively, then what was the total initial investment with
which A and B started the business? [Pubali Bank – JO – 2019; Sonali Bank - Officer (FF) -
2019 (AUST)]
Solution
A's initial investment = TK.18x
B's initial investment = Tk.7x
According to the question
{ × ( )× }
=
{ × ( ) }
×{ ( )× }
Or, =
×{ ( ) }
Or, 18x + 36x + 4000 = 14x + 28x + 28000
Or, 54x - 42x = 28000 - 4000
Or, 12x = 24000
Or, x = = 2000
Total initial investment of A and B = 25x = 25 x 2000 = Tk.50000
Answer: Tk.50000
5) A, B and C are three partners of a business. A receives 2/3rd of the profit and the
remaining profit is equally shared by B and C. If A's income increases by Tk.400 when
the profit of the business rises from 5% to 7%, find the profit received by B.
[PKB (EO -Cash) -2018 (AUST)]
Solution:
Let, profit be x
so for A given profit is it receives
Remaining profit = 1 - =

109
[CAREER AID FOR BANK AND BCS] Writtten math

B and C each receive the profit = and respectively


According to the question
1.07x× - 1.05x× = 400
Or, 2x×.02 = 1200
Or, x= 30,000
Now, we need to determine profit of B at 7% 0.07×30,000 × = TK 350
Answer: Tk 350
6) In a business A invests Tk.600/- more than B. The capital of B remained invested for
7 months, while the capital of A remained invested for 2 more months. If the total profit
be Tk.620/- and B gets Tk.140 less than what A gets, then A’s capital is [Janata Bank - EO -
2018]
Solution:
Let, B's investment be x
A's investment be (x+600)
A: B = [(x+600) × ]: (x × ) = (19x +11400): 15x
Ratio total = 34x+11400
∗( ) ∗
A's profit = tk and B's profit = TK
( ) ( )
According to the question,
×( ) ×
- = 140
( ) ( )
Or, 62 (4x +11400) = 14(34x +11400)
Or, 248x +706800 = 476x +159600
Or, 228x = 547200
Or, x = 2400
A's capital = Tk.(2400+600) = Tk 3000 Answer: Tk 3000

110
[CAREER AID BY YOUSUF CHOTON] Written

Ratio
1) The ratio of the salary of A, B & C is 7:5:3.if B gets tk 222 more than
what C gets, then what is the salary of A?
Solution
Salary of A, B, C is = 7x, 5x, 3x [x is the common term]
Now, 5x −3x = 222
Or, x = 111
The salary of A is = 7×111= 777
Answer: Tk 777
2) 12 Mªvg Mnbvq ‡mvbv Ges Zvgvi AbycvZ 2:1 ‡K, 18 Mªvg Mnbvq I ‡mvbv Ges Zvgvi AbycvZ 5:1
n‡j, `yBUv Mnbv‡K Mwj‡q GKwU AjsKvi ‰Zwi Kiv n‡j ‡mLv‡b ‡mvbv Ges Zvgvi bZyb AbycvZ KZ
n‡e? [BB AD 2024]
mgvavb
2 1
12 Mªvg Mnbvq ‡mvbvi cwigvY = 12×3 = 8 MÖvg Ges 12 Mªvg Mnbvq Zvgvi cwigvY = 12×3 = 4 MÖvg|
5 1
18 Mªvg Mnbvq ‡mvbvi cwigvY = 18×6 = 15 MÖvg Ges 18 Mªvg Mnbvq Zvgvi cwigvY = 18×6 = 3 MÖvg
bZzb AjsKv‡i ‡mvbv Ges Zvgvi bZyb AbycvZ = 8+15 t 4+3 = 23t 7
DËi 23:7
3) In a mixture of milk and water the proportion of milk is 60 % by
weight. If from 80 gm of mixture, 20gm mixture taken out and 6 gm of
water added to the mixture. Then find the ratio of milk and water in the
new mixture. [Janata Bank PLC, RC – 2024]
Solution
Here, Quantity of Milk in 80gm Mixture = 60 % of 80 = 48 gm
Quantity of Water in 80gm Mixture = 80 ‒ 48 = 32 gm
After taken out 20 gm mixture and adding 6 gm pure water
48
Quantity of Milk = 48 ‒ 20× = 36 gm
80
32
Quantity of Water = 32 ‒ 20× + 6 = 30 gm
80
Hence, the ratio of milk and water in the new mixture = 36: 30 = 6: 5
Answer6: 5
4) The ratio, by volume, of bleach to detergent to water in a certain
Solution is 2:40: 100. The Solution will be altered so that the ratio of
bleach to detergent is tripled while the ratio of detergent to water is
halved. If the altered Solution will contain 300 liters of water, how many
liters of detergent will it contain?
Solution
Here, Bleach: Detergent :: Detergent: Water =2:40:100 = 2:40::40:100

111
[CAREER AID BY YOUSUF CHOTON] Written

Bleach 2 3
= 3× =
Detergent 40 20
Detergent 1 40 20
= × =
Water 2 100 100
Now, Bleach: Detergent: Water =3: 20: 100
Let, Amount of bleach in altered mixture = 3x liters
Amount of detergent in altered mixture = 20x liters
Amount of water in altered mixture = 100x liters
Here, 100x = 300
Or, x = 3
Now, Amount of detergent in altered mixture = 20x = 20×3 = 60 liters
Answer: 60 liters
5) Sana and Ripa earn in the ratio 2:1. They spend in the ratio 5: 3 and
save in the ratio 4:1. If the total monthly savings of both Sana and Ripa
are taka 5000, calculate the monthly income of Ripa?
Solution
Let, Sana’s income = 2x
Ripa’s income = x
Sana’s expenditure = 5y
Ripa’s expenditure = 3y

Their savings Ripa = x – 3y and Sana = 2x – 5y


According to the question,
2x - 5y 4
=
x - 3y 1
Or, 2x ‒ 5y = 4x ‒ 12y
Or, 2x = 7y
7y
Or, x =
2
Again,
(2x‒5y)+(x‒3y) = 5000
Or, 3x ‒ 8y = 5000
7y
Or, 3× ‒ 8y = 5000
2
(21y - 16y)
Or, = 5000
2
Or, 5y = 10000
10000
Or, y =
5
Or, y = 2000
2000
So, Ripa’s income = 7× = 7000
2

112
[CAREER AID BY YOUSUF CHOTON] Written

Answer: Tk 7000
6) A brother and a sister share out their collection of 5000 stamps in the ratio of
5:3. The brother then share his stamps with his two friends in the ratio of 3:1:1,
keeping most for himself. How many stamps does each his friends receive?
Solution
5
Share of the brother = 5000 × = 3125 stamps
8
1
One friend Receives = 3125 × = 625 stamps [Answer]
5
7) There are three containers A, B and C which contain water, milk and acid
respectively in equal quantities. 10% of the content A is taken out and poured
into B. Then the same amount from B transferred to C, from which again the
same amount is transferred to A. What is the proportion of milk in container A
at the end of the process? [Combined SO – 2021 based]
Solution
Let, initial amount of each container A, B and C = 1000 L
First case
If 10% of content A is taken out and poured into B then B becomes = 1000 +
10%×1000 = 1100 L
1000 10
Milk concentration becomes in B = = liter
1100 11
Second case
10 1000
If 100 liter of content is poured into C then amount of milk in C = 100× = liter
11 11
1000
1000 10 11 1000 1 10
Now, milk concentration becomes = = liter = × = liter
1100 11 1100 11 1100 121
Third case
10
Again, 100 liter of concentration is poured in A, then amount of milk in A = 100×
121
1000
= liter
121

1000
121 1
= Now, milk concentration becomes = = liter
1000 121
1
Answer liter
121

113
[CAREER AID BY YOUSUF CHOTON] Written

Mensuration
Triangle
1) The length of each of the two equal sides of an isosceles triangle is 10cm
and the included angle between those two sides is 45 degree. Find the area
of the triangle? [BKB – (Cash)‒2018 – (Written)] & [Rupali Bank –
(Cash)‒2018‒(Written) ‒ Cancelled]
Solution:
1
Area of a isosceles triangle = a× b sinθ
2
[For isosceles triangle, a = b]
1
Now, area = a2 sinθ 2
2
1
Or, area = ×102×sin450
2

1 2 2
Or, area = 50 × = 50 = 50× = 25 2
2 2× 2 2
Answer: 25 2 sq m

2) The length of two sides of a triangle are p, q and they are attached with
a right angle. The perimeter of a triangle is 8p. Find the ratio of p and
q[DBBL, MTO – 2024]
Solution
Let, hypotenuse of the triangle is r unit
As the triangle is right angle hence, p2 +q2 = r2 and the perimeter of the triangle, p+q+r
= 8p
Or, r = 7p ‒ q
Now, p2 +q2 = (7p ‒ q)2 = 49p2 ‒ 2×7pq + q2 = 49p2 ‒ 14pq + q2
Or, 48 p2 = 14pq Or, 48p = 14q Or, p: q = 14: 48 = 7: 24
Answer 7: 24
3) A square and an equilateral triangle have equal perimeters, If the
diagonal of the square in 12√ cm, then what is the area of the triangle?
Solution Let one side of a square = a cm
Then, the diagonal of the square = a 2 = 12 2 cm
So, a =12
Then, the parameter of the square = 4×2 = 48 = Parameter of the equilateral
triangle
48
One side of equilateral triangle, x = = 16 cm
3

114
[CAREER AID BY YOUSUF CHOTON] Written

3
Hence, Area of equilateral triangle = ×162 = 64 3 sq cm
4
Answer 64 3 sq cm
4) On his way home from shopping, Mike must travel due south for 5
miles and then due east for another 12 miles to reach his house. If Mike
could travel in a straight line from the store to his house, how many fewer
miles would he travel?
Solution
Pythagorean ratio: 5, 12, 13
How many fewer miles = (5+12) ‒13 = 4
Answer: 4

5) If the perimeter of an isosceles right angle triangle is 6+3√2. Find the


area of this triangle. [GKwU mgwØevû mg‡KvYx wÎfz‡Ri cwimxgv 6+3√2 n‡j wÎfzRwUi
†ÿÎdj KZ?}
Solution
Let, the equal sides of the triangle is a and a unit (base and height)
From the Pythagorean Theorem, we know hypotenuse = a2 + a2 = 2 a2 =
a 2 unit
Therefore, three sides of the triangle is a, a and a 2
Now, a + a + a 2 = 3+3+3√2 = 6+3√2
1
So Area = × (3×3) = 4.5 sq unit
2
Answer 4.5 sq unit
Level: Advance
6) Faiyaz and Ehna was 90 yards’ distance in north and south axis. Faiyaz
walks 70 yards in west and Enha walks 50 yards east and stopped. What is
the straight length between Faiyaz and Ehna?
Solution
From the figure, EF is the distance between Faiyaz and Ehna. After walking
towards West and East their distance become QR.

115
[CAREER AID BY YOUSUF CHOTON] Written

From right angle triangle PQR Or, QR2 = 22,500


PR2 + PQ2 = QR2 Or, QR = 22500 Error! Reference
Or, 902 + (70 + 50)2 = QR2 source not found.
Or, 902 + 1202 = QR2 ⸫QR = 150 yards
Or, QR2 = 8100 + 14400 Answer 150 yards
EC
7) In the figure, what is the area of ΔABC if = 3? [Janata Bank PLC, RC –
CD
2024]
Solution

EC
Answer From the above figure given that, = 3, So, EC = 3CD
CD
Again, EC = EB + BC = 3 + 9 = 12 hence, 12 = 3CD Or, CD = 4
EC CD 12 4
Now, as ΔABC and ΔEDC are similar triangle we get, = Or, =
AC BC AC 9
Or, AC = 27
1
Now, area of ΔABC = ×27×9 = 121.5 sq units
2
Answer 121.5 sq units
8) In the given figure ∟BAD = ∟CAD, AB = 4cm, AC = 5.2 cm, BD = 3 cm find
BC? [Combined SO – 2021 based]
Solution

116
[CAREER AID BY YOUSUF CHOTON] Written

From the above figure ΔABD and ΔABC are similar triangle, because ∟BAD =
∟CAD
AB BD 4 3 5.2×3
Now, = Or, = Or, CD = = 3.9
AC CD 5.2 CD 4
Now, BC = BD + CD = 3 + 3.9 = 6.9 cm
Answer6.9 cm
9) In the figure below, AB is perpendicular to BC and BD = DC, If AD =
√10 cm and AC = 4 cm, then what is the value of BC?
Solution

From the figure, AD = √10, AC = 4 cm. Let BD = DC = x cm and AB = h cm


Here, ABD is a right angle triangle, from the Pythagorean Theorem
AD2 = AB2 + BD2
Or, (√10)2 = h2 + x2
Or, h2 = 10 − x2 ‒‒ ‒‒‒‒ (1)
Again, from triangle ABC
AC2 = AB2 + BC2 [BC = BD + DC = x + x = 2x]
Or, 42 = h2 + (2x)2
Or, 16 = 10 − x2 +4x2
Or, 3x2 = 16 – 10 = 6
Or, x2 = 2
Or, x = √2
Hence, BC = x + x = 2√2 cm [Answer]

117
[CAREER AID BY YOUSUF CHOTON] Written

Rectangular
1) A table is three times as long as it is wide. If it were 5 feet shorter and 5
feet wider, it would be a square. How long and how wide is the table?
[Janata Bank (AEO‒RC) ‒2017 (Written)]
Solution:
Let, width = x feet
Length =3x feet
According to the question,
3x‒5 = x+5
Or, 2x =10
Or, x = 5
So, Width 5 feet
Answer: 5 feet
2) The ratio between the length and the breadth of a rectangular park is 3:2. If a
man cycling along the boundary of the park at the speed of 12 km/hr completes
one round in 8 minutes. Then what is the area of the park?
Solution
5
Perimeter of a rectangle = Distance = speed × time = 12× ×8×60 = 1600 m
18
Let, length 3x m and Breadth = 2x m
Now, 2(3x + 2x) = 1600
Or, 5x = 800
Or, x = 160
Hence, Length = 160×3 = 480 m
Breadth = 160×2 = 320 m
Area = 480× 320 = 153600 sq.m [Answer]
3) GKwU AvqZvKvi N‡ii ˆ`N¨© cÖ‡¯’i †`o¸Y| AvqZvKvi N‡ii †¶Îdj 384 eM©wgUvi n‡j cwimxgv I K‡Y©i
ˆ`N¨© KZ?
Solution
Let, Width = 2x m & Length = 3x m
According to the question,
Area = 2x × 3x = 384
6x² = 384
Or, x² = 64
Or, x = 8 m
Perimeter = 2(3x +2x) = 10x = 10×8= 80 m [Answer]
And length of the diagonal (K‡Y©i ˆ`N¨©) = √(9x²+4x²) = √13x² = √13×64 = 8√13 m
[Answer]
4) In a Rectangular field Length is 6/5 of the Breadth, if the Perimeter is 132 then the
area of the field?
Abyev`t GKwU AvqZKvi gv‡Vi ‰`N¨© cÖ‡¯’i 6/5 Ask, Gi cwimxgv 132 n‡j AvqZb KZ?
Solution
Here, Length: Breadth = 6:5

118
[CAREER AID BY YOUSUF CHOTON] Written

Let, length of the rectangular filed = 6x unit and breadth of the rectangular
filed = 5x unit
Now, 2(6x + 5x) = 132 [⸪Perimeter = 2(Length +
Breadth) unit]
132
Or, 11x = = 66
2
Or, x = 6
Hence, the area of the rectangular filed = 6x×5x sq unit = 30x2 = 30×62 = 1080
sq unit
Answer1080 sq unit
5) GKwU AvqZ‡¶Îi ‰`N¨© c«¯’ A‡c¶v 2¸b| c«¯’ 5 wgUvi evov‡j Ges ‰`N¨© 5 wgUvi Kgv‡j
AvqZ‡¶Î 75 e…w× cvq| ‰`N¨© Ges c«¯’ KZ?
mgvavb
awi, c«¯’ = K wgUvi I ‰`N¨© 2K wgUvi Zvn‡j, ‡¶Îdj = 2K2 eM© wgUvi
cÖkœg‡Z, (K + 5)(2K ‒ 5) = 2K2+75 [ m~Ît AvqZ‡¶Îi †ÿÎdj = ‰`N¨© × c«¯’ eM© &GKK]
ev, 2K2 ‒ 5K + 10K ‒ 25= 2K2+75
ev, 5K = 25 + 75
100
ev, K = 5 = 20
myZivs, AvqZ‡¶Îi c«¯’ = 20 wgUvi Ges AvqZ‡¶Îi ‰`N¨© = 20×2 = 40 wgUvi
DËit40 wgUvi Ges 20 wgUvi
6) The base of a rectangle is three times as long as the height. If the
perimeter is 64 m, what is the area of the rectangle? [BSEC, PO 2020]
Solution
Let, base = b height =h Here, b = 3h
Perimeter, 2(b+h) = 64
Or, (3h+h) = 32
Or, 4h = 32
Or, h = 8
So, b = 3 × 8 = 24
Therefore, area of the rectangle = b×h =24×8=192 sq m
Answer: 192 sq m
7) এক বগে ে র পিরসীমা এক আয়তে ে র পিরসীমার সমান। আয়তে ে র দঘ ে র ৩ ণ
এবং ফল ১২০০ বগিমটার। িত ৫০ cm বগকার পাথর িদেয় বগে বাঁধােত মাট কত পাথর
লাগেব ? [Sonali Bank Senior Officer‒2014]
সমাধান:
মেন কির,
আয়তে ে র x িম
আয়তে ে র দঘ ৩x িম
শতমেত,

119
[CAREER AID BY YOUSUF CHOTON] Written

৩x ২ = ১২০০
বা, x ২ = ৪০০
বা,x = ২০
তরাং, আয়তে ে র = ২০ িম
আয়তে ে র দঘ = ৬০ িম
আয়তে ে র পিরসীমা = ২(৬০+২০) =১৬০ িম
যেহ এক বগে ে র পিরসীমা এক আয়তে ে র পিরসীমার সমান
তরাং বগে ে র পিরসীমা = ১৬০ িম
এবং বগে ে র একবা = ৪০ িম
তাই বগে ে র ফল = (৪০×৪০) =১৬০০ বগিম
িত পাথেরর দঘ ৫০ স.িম = ০.৫ িম.
িত পাথেরর ফল = (০.৫×০.৫)বগিম = ০.২৫ বগিম.
১৬০০
তরাং িত ৫০cm বগকার পাথর িদেয় বগে বাঁধােত মাট পাথর লাগেব = ০.২৫ = ৬৪০০
উওর: ৬৪০০
8) A tailor trims 4 feet from opposite edges of a square piece of cloth, and 5 feet
from the other two edges. If 120 square feet of cloth remain, what was the length
of a side of the original piece of cloth? [DBBL, AO – 2024]
Solution
Let, the length of a side of a square piece cloth = x unit
[jwRKt Kvc‡oi eM©vKvi UzKivi `ycvk †_‡K 4 dzU I Ab¨ `ycvk †_‡K 5 dzU KZ©b Kiv n‡j eM©vKvi Kvc‡oi
UzKivwU GKwU AvqZKvi Kvc‡oi UzKivq cwiYZ n‡e| ]
kZ©g‡Z,
(x ‒ 4×2)×(x ‒ 5×2) = 120 [⸪AvqZKvi‡ÿ‡Îi ‡ÿÎdj = ˆ`N©¨ × cÖ¯’ eM© GKK]
Or, (x ‒ 8)×(x ‒ 10) = 120
Or, x2 ‒ 8x ‒ 10x + 80 ‒ 120 = 0
Or, x2 ‒ 18x ‒ 40 = 0
Or, x2 ‒ 20x + 2x ‒ 40 = 0
Or, x(x ‒ 20) + 2(x ‒ 20) = 0
Or, x = 20, ‒2
Hence, the length of a side of a square piece cloth is 20 feet
Answer 20 feet
9) An amount of Tk. 7200 is spent to cover the floor of a room by carpet.
An amount of Tk. 576 would be saved if the breadth were 3 meters less.
What is the breadth of the room? [BDBL SENIOR OFFICER 2018]
সমাধান:
মেন কির, দঘ x িম. , y িম
িত বগ িম খরচ হেব zz টাকা,
মেত,
xyz = ৭২০০‒‒‒‒‒‒‒‒(১)
xz (y ‒৩) = ৭২০০‒৫৭৬= ৬৬২৪‒‒‒‒‒(২)

120
[CAREER AID BY YOUSUF CHOTON] Written

সমীকরন(১) ক (২) নং িদেয় ভাগ কের=>


xyz /xz (y ‒৩) = ৭২০০/৬৬২৪
বা, y =৩৭.৫
উ: ৩৭.৫ িম
10) M purchased a 30‒inch TV whose height was 18 inches. If the size of a TV is
expressed as the lengths of its diagonal, find the width of the TV M purchased.
[Sonali Bank SO (IT)‒2016]
Solution:
Let, The width of the TV = X inches.
According to the question:
X² + 18² = 30²
Or, X² = 900 ‒ 324
Or, X² = 576
Or, X = 24
So, the width of the TV is 24 inches.
Answer: 24 inches
11) Twice the width of a rectangle is 10 meters more than its length. If the area of
the region enclosed by the rectangle is 600 square meters then find its perimeter.
[BD House Building FC (SO)‒2017 (Written)]
Solution
Let, length = x and width = y
According to the question,
2y = x+10
So, x = 2y‒10 ‒‒‒‒‒‒‒‒‒‒‒ (1)
Again, y(2y‒10) = 600
____________
So, y = 20
Thus, x = 30
Perimeter = 2(30+20) =100 meter Answer: 100 meter
12) A gardener plants two rectangular gardens in separate regions on his
property. The first garden has an area of 600 square feet and a length of 40 feet.
If the second garden has a width twice that of the first garden, but only half of
the area, what is the ratio of the perimeter of the first garden to that of the
second garden? [Sonali Bank (Cash)‒2018‒(Written)]
Solution:
Width of the first garden = 600/40 = 15 feet.
According to the question,
Width of the second garden = 15 × 2 = 30 feet
And, area of the second garden = ½ × 600 = 300 sq. feet
So, length of the second garden = 300/30 = 10 feet
Now, Perimeter of the first garden: Perimeter of the second garden = 2(40+15):
2(10+30) = 110: 80 = 11: 8
Answer: 11: 8

121
[CAREER AID BY YOUSUF CHOTON] Written

13) A rectangular plot has a concrete path running in the middle of the plot
parallel to the breadth of the plot. The rest of the plot is used as a lawn, which
has an area of 240m2. If the width of the path is 3m and the length of the plot is
greater than its breadth by 2m, what is the area of the rectangular plot?
[Combined SO 2019, re exam]
Solution Length = x +2
Let, the width of plot = x meter
So, Length = (x + 2) meter 3m
Area of concrete path = 3x sq. meter
According to the question, x
x(x + 2) ‒ 3x = 240
Or, x2+2x−3x =240
Or, x2−x−240 =0
Or, x2−16x+15x−240 = 0
Or, x(x ‒ 16) + 15(x ‒ 16) = 0
Or, (x ‒ 16) (x + 15) = 0
Or, x = 16 because x ≠ ‒15
So, Length = 16 + 2 = 18 meter
So, Area of the plot = (18 × 16) sq. meter = 288 sq. meter.
Answer 288 sq. meters
14) The length and width of a garden is 60 meter and 20 meter respectively.
Inside the garden, there is a 5 meter‒wide path around it. What is the area of the
path in square meter?
Solution
Area including path = 60 × 20 = 1200 sq. m.
Length without path = 60 ‒ (5×2) = 50 m
Width without path = 20 ‒ (5×2) = 10 m
Area without path = 50 × 10 = 500 sq. m.
Area of the path = 1200 ‒ 500 = 700 sq. m.
Answer700 sq. m
15) A rectangular grassy plot is 112 m long
and 78 m broad. It has a travel path 2.5 m wide all around it on the sides. Find
the area of the path
Solution

Area of the rectangular plot = 112×78 = 8736 sq m


Area of the rectangular plot without path = (112 – 2.5×2) × (78 – 2.5×2)
= 107 × 78 = 7811sq m
Area of the path = (8736 – 7811) sq m = 925 sq m Answer: 925 sq m

122
[CAREER AID BY YOUSUF CHOTON] Written

16) The area of rectangle 24 is divided with three non‒overlapping squares. What
is the longest side of rectangle?
Solution

Let, each side of the square = x unit


According to the question
3×x² = 24
Or, x² = 8
Or, x = √8 = 2√2
Now, longest side of the rectangle
= 3×2√2 = 6√2 unit
Answer 6√2 unit
17) A rectangular solid is changed such that the length and width increase by 1
inch apiece and the height is decreased by 9 inches. Despite these changes the new
rectangular solid has the same volume as the original rectangular solid. If the
width and length of the original rectangular solid are equal and the height of the
new rectangular solid 4 times the width of the original rectangular solid, what is
the volume of the rectangular solid?
Solution
Let, the original length, width and height of the solid rectangular are respectively l, l
and h unit
Old Rectangular Solid New Rectangular Solid
Length L L+1
Width L L+1
Height H H‒9
Volume L×L×H (L+1)×(L+1)×(H‒9)
Condition H‒9 = 4L
Or, H = 4L+9
Now, L×L×H = (L+1) × (L+1) × (H‒9)
Or, L2 × (4L+9) = (L+1)2 × (4L+ 9 ‒ 9)
Or, 4L3 + 9L2 = (L2 + 2L + 1) × 4L
Or, 4L3 + 9L2 = 4L3 + 8L2+ 4L
Or, 9L2 ‒ 8L2 = 4L
Or, L2 ‒ 4L = 0
Or, L(L ‒ 4) = 0
So, L = 0 and L = 4
Hence the height of a new solid rectangle = 4×4 + 9 = 25

123
[CAREER AID BY YOUSUF CHOTON] Written

Therefore, volume of new solid rectangle = 4×4×25 = 400 cu unit


Answer 400 cu unit
18) A farmer wishes to start a 100 sq. m rectangular vegetable garden. Since he
has only 30 m barbed (KvUuv) wire, he fences three sides of the garden letting his
house compound wall act as the fourth side fencing. The dimension of the garden
is‒ [JBL AEO 2020]
Solution

Let, length of the rectangular garden be x m


100
So, width = m
x
If the uncovered side is length,
100
x + 2× = 30
x
Or, x2 ‒ 30x + 200 = 0
Or, (x‒20) (x‒10) = 0
Or, x = 20 Or, 10
100
If we take x = 20, length = 20 m and width = =5m
20
100
If we take x = 10, length = 10 m and width = = 10 m. [10 by 10 rectangle is not
10
possible]
Now, if the uncovered side is width,
100
2x + = 30
x
2
Or, 2x ‒ 30x + 100 = 0
Or, (x‒10) (2x‒10) =0
Or, x =10 Or, 5
100
If we take x = 10, length =10 m and width = = 10 m. [10 by 10 rectangle is not
10
possible]
100
If we take x = 5, length = 5 m and width = = 20 m [As width of rectangular can't
5
be larger than length, x= 5 is rejected]
So, dimension of the rectangular garden is: Length 20 m, Width 5 m [Answer]
Inside road
19) A garden is 60 meter long and 20 meter wide. Inside the garden there is a 5
meter wide 4 sided paths inside the garden. If it needs 20 taka per square meter

124
[CAREER AID BY YOUSUF CHOTON] Written

to cover with grass, how much it will need to cover the path with grass? [Premier
Bank TJO‒General‒2018, Standard Bank (TAO) Cash 16]
Solution:
Given, length is 60 meter and width is 20 meter.
Area is (60×20) sq. m. = 1200 sq m.
Length of garden without path = (60‒ 5×2) = 50 meter.
Width of garden without path = (20‒5×2) = 10 meter.
Area of garden without path is (50×10) = 500 sq. meter.
Area of the path =1200‒500 sq meters. = 700 sq. m.
To cover this 700 sq. m with grass total cost is (700× 20) = 14000 tk.
Answer: 14000 tk
Outside road
20) 21 িমটার দঘ ও 15 িমটার িবিশ এক বাগােনর বাইেরর চারিদেক 2 িমটার চওড়া এক পথ আেছ।
িত বগিমটার 25 টাকা িহেসেব পথ েত ঘাস লাগােত মাট কত টাকা খরচ হেব? [Sonali Bank
Officer‒2014]
Solution:
iv¯Ívmn evMv‡bi ˆ`N¨© =21+2×2 = 25 িমটার
iv¯Ívmn evMv‡bi =15+2×2 = 19 িমটার
iv¯Ívmn evMv‡bi ‡ÿÎdj = 25×19 = 475 eM© িমটার
iv¯Ívev‡` evMv‡bi ‡ÿÎdj = (21×15) = 315 eM© িমটার
iv¯Ívi †ÿÎdj = (475‒315) =160 eM© িমটার
m¤c~Y© iv¯ÍvwU‡K Nvm w`‡q †gvov‡Z †gvU LiP =160×25 = 4000 টাকা
Answer: 4000 টাকা
Cross road
21) There are two crosswise roads just in the middle of a field of length 50 m and
breadth 40 m. The breadth of each road is 1.5 m. Find the area covered by two
roads.
Solution

Area of the road crossing along the length of field= 50×1.5= 75 sq m


Area of the road crossing along the breadth of the field= 40×1.5= 60 sq m
But, two roads overlap each other in the middle point of the field which has the area of
= 1.5×1.5= 2.25 sq m
Net Area of roads = area of road (along with length + along with breadth – common
place) = 75 + 60‒2.25 = 132.75 sq m
Answer: 132.75 sq m
22) A lawn is in the form of a rectangle having its sides in the ratio 2:3. The area
of the lawn is hectares. Find the length and breadth of the lawn.

125
[CAREER AID BY YOUSUF CHOTON] Written

Solution:
Let, Length = 3x meter
And breadth = 2x meter
Given,
Area = hectares = meter2 = meter2
According to the question,
3x × 2x =
Or, 6x2 =
Or, x2 =
Or, x2 =
Or, x = ±√ ( )
Or, x = ±
Or, x = (negative is not acceptable)
So, length = 3x = 3 × ( ) meter = 50 meter
Breadth = 2x = 2 × ( ) meter = meter = 33.33 meter
Answer: 33.33 meter
23) A rectangular box that is 7 inches long by 4 inches wide by 3 inches deep is
carefully packed to hold the maximum number of blocks that are 1 inch by 1 inch
by 2 inches. How many blocks can be packed into the box? [NRBC PO 2021]
Solution
7×4×3 42
Number of blocks = = = 42[Answer]
1×1×2 1
Rhombus
1) A Rhombus has an area of 120 square meters and the length of its one
diagonal is 10 meter. Calculate the perimeter of Rhombus. [BKB OFF 2017]
Solution:
Two diagonal are x and y
ACCORDING TO THE QUESTION,
1
×(xy) = 120
2
Or, xy = 240
Or, x = 24
One side of the Rhombus
= √ [( ) 2 + ( ) 2]
= √ (144 + 25)
= √169 = 13
Perimeter of Rhombus = 4×13 = 52 m
Answer: 52 m

126
[CAREER AID BY YOUSUF CHOTON] Written

Circle
1) 42 ‡mwg e¨vmva© wewkó e„ËvKvi GKwU Zvi ‡K evuwK‡q AvqZKvi (AbycvZ 6t5) ‡¶Î ‰Zwi Ki‡j ‡QvU evûi
‰`N¨© wbY©q Ki? [BB AD 2024]
mgvavb& 42 ‡mwg e¨vmva© wewkó e„ËvKvi GKwU Zvi‡K evuwK‡q AvqZKvi ‡¶Î ‰Zwi Ki‡j e„‡Ëi cwiwa n‡e
AvqZ‡ÿ‡Îi cwimxgvi mgvb|
e¨vmva©, 2πr = 42 ‡mwg
22
e„‡Ëi cwiwa = 2rπ = 2× 7 ×42 = 264 ‡mwg
AvqZKvi‡¶‡Îi ‰`N¨© 6K ‡mwg I cÖ¯’ 6K ‡mwg Zvn‡j, cwimxgv = 2(6K+5K) = 22K ‡mwg
cªkœg‡Z, 22K = 264
264
ev, K = 22 = 12
‡QvU evûi ‰`N¨© = 5×12 = 60 ‡mwg.
DËi 60 ‡mwg.
2) What is the circumference of the circle if a 3 by 4 rectangle is inscribed in the
circle?
Solution

Diagonal = 32+42 = 16+9 = 25 = 5


5
Radius = = 2.5
2
Circumference = 2π × 2.5 = 5π unit

3) In a square field of side 30 meters, 4 cows are grazing the field as they are tied
at each of the four corners with a 14 meters long rope for each cow. What is the
ungrazed area in the field?
Solution
Total area of square= 30×30 = 900 sq m
14 meters long rope makes a circle of having radius 14
1
cm. Each circle covers parts of the square.
4
1 22
Four cows are grazing = 4× × ×142 = 616 sq m
4 7
Then ungrazed area = 900–616= 284 sq m
Answer: 284 sq m

4) The figure shows a circular flowerbed, with its center at O having radius of 8
feet. The flowerbed is surrounded by a circular path, which is 3 feet wide. What
is the area of the path in square feet?
Solution

127
[CAREER AID BY YOUSUF CHOTON] Written

The radius of the bigger circle is 8 + 3 = 11 feet, thus its area is = πr2 = π×112 = 121π
sq feet
The radius of smaller circle is 8 feet.
The area of the smaller circle is = πr2= π×82 = 64π sq feet
The difference is = (121‒64)π =57π sq feet
Answer 57π sq feet
5) A square is inscribed in a circle with radius 4 inches. Find the area of the
region inside the circle and outside the square.
Solution:
Given that, radius of a circle, r = 4, so Diameter of a circle =2×4=8
Area of a circle = πr2 = π×42 =16π
Let, a = side of a square
Diagonal of a square = √2a

We have,
Diagonal of a square = Diameter of a circle
√2a = 8 Diagonal of the square = diameter of the circle
Or, a =4√2
Area of square = (4√2)2 = 32
Required area = 16π – 32
Answer: 16π – 32
6) A circular garden is surrounded by a road. If the outer circumference of the
road is greater than the inner circumference by 88 meters, what is the width of
the road? [Rupali SO 2019]
Solution:
Let, the radius of the garden including the road is = x meters, and the radius of the
garden excluding the road is = y meters.
So, the width of the road is = (x‒y) meters
According to the question,
2πx ‒ 2πy = 88
Or, 2× ×(x‒y) = 88
Or, (x‒y) = 4×
Or, (x‒y) = 14
So, the width of the road is 14 meters
Answer: 14 meters
7)The length of a tangent from a point Q at distance 5 cm from the centre of the
circle is 4 cm. Find the radius of the circle [Agrani Bank (SO‒Auditor)‒
2018‒(Written)]

128
[CAREER AID BY YOUSUF CHOTON] Written

Solution:
PQ is a tangent drawn on this circle from
point A.
∴ OP ⊥ PQ
OQ = 5cm and PQ = 4 cm (Given)
In ΔOPQ,
By Pythagoras theorem in ΔOPQ,
OQ2 = PQ2 + OP2
⇒ 52 = 42 + OP2
⇒ OP2 = 25 ‒ 16
⇒ OP2 = 9
⇒ OP = 3
∴ The radius of the circle is 3 cm
Answer: 3 cm
8)The length of a tangent drawn from a point P is 12 cm which is 13cm away
from the centre of the circle. Find the diameter of the circle? [Sonali Bank (SO) –
2018‒(Written)]
Solution:

Let, PT =12 cm be the tangent to the circle whose center is O, TO = r cm and OP =13 cm.
In right angled triangle OPT
OT2 +PT2 = OP2
Or, r2+ 122 =132
Or, r2 =169–144 =25
r = 5 cm
Diameter = 2 × r = 2× 5 cm = 10 cm. Answer: 10 cm
9) A maximum sized rectangular playground is made within a circular space of
radius 2√41 meter. Calculate the area of the playground if its length and width
are distributed in the proportion of 4:5.
Solution:
Here, radius= 2√41
And let, length = 4x m, wide = 5x m
So, diameter = 2× (2√41) = 4√41 m
According to the question,
(4x) 2 + (5x) 2 = (4√41)2
Or, 41 × x2 = 16×41
Or, x2 = 16
Or, x= 4
So, area = (4×4) × (5×4) = 320 m2 Answer: 320 m2

129
[CAREER AID BY YOUSUF CHOTON] Written

Parallelogram
1) ABCD is a parallelogram shaped park. It’s side AB is parallel to the side DC
and DA is parallel to CB. The length of side AB is 20 m .Mrs. Halima was
walking along the side AB. She entered the park through a gate which is 3 m
away from point A and got of the park through a gate at point F on side CD of
the park. The path connecting the two gates divides the park in two parts of
equal area. What is the distance between D and F?
Solution:
By given, we know that
AB = 20cm and AE = 3 cm
By sum of segments, we have

Additionally, we know that segment EF divides the parallelogram in two equal areas,
that is, in two equal trapezoids that mean their bases are congruent.
So,EB = DF
⸫DF = 17 cm
Answer: 17 cm
Cube
1) A solid cube of side 6 is first painted pink and then cut into smaller
cubes of side 2. How many of the smaller cubes have paint on exactly 2
sides?
Solution
Each cube has 27 smaller cubes.
(A cube has 6 faces, 8 vertices and 12 edges.)
Now, 2 side colored will be:
(n‒2) ×12= (3‒2) × 12= 12
Answer12
Trapezoid
If the length of the two sides of a trapezoid are 4 cm and 7 cm. and height
trapezoid is 7 cm. calculate the area of the trapezoid?
Solution
Here, one side of a trapezoid, a = 4 cm
Another side of a trapezoid, b = 7 cm
Height of the trapezoid, h = 7 cm
1 1 77
We know, Area = × (a + b) × h = × (4 + 7) × 7 = = 38.5 sq. cm. [Answer]
2 2 2
Square
1) A man walked diagonally across a square lot. Approximately, what was the
percent saved by not walking along the edges?
Solution

130
[CAREER AID BY YOUSUF CHOTON] Written

Let, Side of the square =1 unit


And Perimeter = 4 unit
Half‒perimeter = 2 unit
Accordingly, The man walks diagonally, diagonal =√(1+1) =√2 =1.414
Save walking = 2 ‒ 1.414 = 0.586
0.586×100
Required% = =29.3%
2
Answer29.3%
2) ABCD is a square and one of its sides AB is also a chord of the circle as
shown in the figure. What is the area of the square?
Solution:
Solution: In the figure AOB is a right angle triangle with adjacent sides AO =
BO = 3 units.
So, the length of the hypotenuse
AB2 = AO² + BO² = 3² + 3² = 18 units.
Also, AB is a side of the square ABCD.
Therefore, the area of the square, AB² = 18 sq units.
Answer: 18 square units

3) square is inscribed in a circle with radius 4 inches. Find the area of the region
inside the circle and outside the square.
Solution:

Given that, radius of a circle, r = 4, so Diameter of a circle =2×4=8


Area of a circle = πr2 = π×42 =16π
Let, a = side of a square
Diagonal of a square = √2a

We have,
Diagonal of a square = Diameter of a circle
√2a = 8 Diagonal of the square = diameter of the circle
Or, a =4√2
Area of square = (4√2)2 = 32
Required area = 16π – 32
Answer: 16π – 32
Cone
1) Movenpick's ice‒cream cones are of 12 cm in diameter and 18 cm
height. To cope with the price hike of the ingredients, the management
planned to reduce the quality of ice‒cream per cone by adjusting the cone
diameter and/or height. If they reduce diameter by 2 cm while

131
[CAREER AID BY YOUSUF CHOTON] Written

maintaining the height, how much ice‒cream can they save per cone
[FSIBL PO‒2016 MCQ, GMAT]
Solution:
Give that,
Initial diameter of ice‒cream = 12 cm
So, Radius of ice‒cream, R = = 6 cm
After 2 cm reduce diameter then new diameter = (12 ‒ 2) = 10 cm
New Radius r = = 5 cm
Total Ice‒cream saved
= Initial Volume ‒ Final Volume
= π(R2)h ‒ πr2h = 6π × (36 ‒ 25) = 66 π
Answer: 66π
Trigometry
1) S‡o GKwU MvQ †n‡j co‡jv| Mv‡Qi †Mvov †_‡K 7 wgUvi D”PZvq GKwU LywU †Xm w`‡q MvQwU †mvRv Kiv
n‡jv| gvwU‡Z LyuwUi ¯ck© we›`yi AebwZ †Kvb 30⁰ n‡j LyuwUi ˆ`N¨© KZ? [IDRA Officer 2022]
Solution

AB = 7 wgUvi| ‡h D”PZvq GKwU LuywU †Xm w`‡q GKwU MvQ


†mvRv Kiv n‡jv|
gvwU‡Z LyuwUi ¯ck© we›`yi AebwZ †Kvb 30⁰ A_©vr = ∟ACB
= 30⁰
LyuwUi ˆ`N¨©, AC = ?
j¤^ 7
AvgivRvwb, Sin300 = AwZf~R = LyuwUi ˆ`N¨©
1 7
Or, 2 = LyuwUi ˆ`N¨©
Or, LyuwUi ˆ`N¨© = 7×2 = 14 wgUvi [Answer]

132
[CAREER AID BY YOUSUF CHOTON] Written

Boat and Stream


1) A boat running upstream takes 10 hours 48 minutes to cover a certain
distance, while it takes 4 hours to cover the same distance running
downstream. What is the ratio between the speed of the boat and speed of
the water current respectively?
Solution
Let, Speed of the boat is x According to the question,
Speed of the current is y 648×(x‒y)}= 240×(x+y)}
Speed of the downstream = (x+y) Or, 648x‒ 648y = 240x+ 240y
Speed of the upstream = (x‒y) Or, 648x – 240x = 648y + 240y
Or, 408x = 888y
Here, 4 hr = 4×60 min = 240 min Or, 34x = 74y
Or, 17x = 37y
10 hr 48 min =10×60+48 min =
Or, x: y = 37: 17
648min Answer: 37: 17
2) A motorboat, whose speed in 15 km/hr in still water goes 30 km downstream
and comes back in a total of 4 hours 30 minutes. The speed of the stream (in
km/hr) is
Solution
Say stream speed = x km/h
According to the question,
distance distance
+ = total time
down stream speed up strem speed
30 30 9
Or, + = 4 hr 3 mins =
(15+x) (15-x) 2
30(15-x) + 30(15+x) 9
Or, =
(15 - x2) 2
Or, 2×10(15 – x) + 2×10(15 + x) = 3× (225 ‒ x2)
Or, 675 ‒ 3x2 = 300 – 20x + 300 + 20x
Or, 3x2= 675 ‒ 600
Or, 3x2 = 75
Or, x2 = 25
⸫x=5
Answer: 5 km/hr

133
[CAREER AID BY YOUSUF CHOTON] Written

Train
1) GKwU ‡Uªb GKwU e¨w³‡K 2 ‡m‡K‡Û AwZµg K‡i| Avevi H ‡U«b wU 50 wgUvi GKwU cø¨vUdg©‡K
4.5 ‡m‡K‡Û AwZµg K‡i| ‡UªbwUi ‰`N¨© I MwZ‡eM wbY©q Ki? [BB AD 2024]
mgvavb †`qvAv‡Q, cø¨vUd‡g©i ˆ`N©¨ 50 wgUvi|
‡Uª‡bi ‰`N¨©
†Uªb GKwU e¨w³‡K 2 ‡m‡K‡Û AwZµg K‡i, mgq = MwZ‡eM ev, 2× MwZ‡eM = ‡Uª‡bi ‰`N¨©
‡Uª‡bi ‰`N¨© + cø¨vUd‡g©i ˆ`N¨©
‡U«b hLb 50 wgUvi GKwU cø¨vUdg©‡K 4.5 ‡m‡K‡Û AwZµg K‡i, mgq = †Uª‡bi MwZ‡eM
2× ‡Uª‡bi MwZ‡eM + 50
ev, 4.5 = ‡Uª‡bi MwZ‡eM ev, 4.5× ‡Uª‡bi MwZ‡eM = 2× †Uª‡bi MwZ‡eM + 50
ev, (4.5 – 2)× †Uª‡bi MwZ‡eM = 50
50
ev, †Uª‡bi MwZ‡eM = 2.5 = 20 wg/‡m
GLb †Uª‡bi ‰`N¨© = 2× MwZ‡eM = 2× 20 = 40 wg
DËi ‡UªbwUi ‰`N¨© = 40 wg I MwZ‡eM = 20 wg/‡m
2) A train passes a station platform in 36 seconds and a man standing on the
platform in 20 seconds. If the speed of the train is 54 km/hr, what is the length of
the platform?
Solution
5
Speed of train = 54× =15m/s
18
Length of train = speed × time =20×15 = 300m.
Say length of platform = x
(300 + x)
Now, = 36
15
Or, x = 540‒300
⸫ x = 240
Answer: 240m

134
[CAREER AID BY YOUSUF CHOTON] Written

Speed and Distance


Type: Basic
1) Saba and Joynal are running at a circular track. Saba runs at a constant rate
of 800 feet/minutes and Joynal is currently rate of 1200 feet/minutes. If Joynal is
currently 80 feet behind Saba, how many second will take Joynal catch up Saba
and 40 feet ahead of her?
Solution
Relative Speed = 1200 – 800 = 400 feet/ minute
Relative Distance = 80 + 40 = 120 feet
Req. Time = (120/400)×60 = 18s
Answer 16 seconds
2) Joyonol runs 1200 feet per minutes; another one runs 800 feet per mint.
Joynul is 80 feet far behind. How many times to Joynul takes overtake and
ahead 40 feet?
Solution
distance (40 + 80) 120
Overtaking time = = = = 0.3 minutes = 0.3×60
relative speed (1200 - 800) 400
= 18 sec
Answer 18 sec
3) †ijc‡_ XvKv I wm‡j‡Ui `~iZ¡ 198 gvBj| GKwU †Uªb wm‡jU †_‡K XvKvi D‡Ï‡k¨ 25 gvBj /N›Uv MwZ‡Z
hvÎv ïiæ K‡i| H GKB mg‡q XvKv †_‡K wm‡jU Gi D‡Ï‡k¨ 20 gvBj/N›Uv MwZ‡Z hvÎv ïiæ K‡i| KLb Ges
XvKv †_‡K KZ `~‡i †Uªb `ywU wgwjZ n‡e? [IDRA Officer 2022]
Solution
MT = 198/(20+25) = 4.4 hr or 4 hr 24 min
They will meet after 4 hr 24 min
Distance from Dhaka = 20×4.4 = 88 km
Another way
`~iZ¡ 198
wgwjZ nevi mgq = Av‡cwÿK †eM = (20+25) = 4.4 N›Uv
XvKv †_‡K `~i‡Z¡ wgwjZ n‡e = 20 × 4.4 = 88 gvBj [Answer]
Type: Advance level
4) Mary passed a certain gas station on a highway while traveling west at a
constant speed of 50 miles per hour. Then, 15 minutes later, Paul passed the same
gas station while traveling west at a constant speed of 60 miles per hour. If both
drivers maintained their speeds and both remained on the highway for at least 2
hours, how long after he passed the gas station did Paul catch up with Mary?
Solution
Let V1= speed of Mary and V2 = speed of Paul
When Paul reaches the gas station, marry would have passed d = V1×t miles = 50 mph
15
× hrs = 12.5 miles
60
So basically, distance between Marry and Paul is 12.5 miles
Relative speed = 60 – 50 = 10 mph [Same direction]

135
[CAREER AID BY YOUSUF CHOTON] Written

12.5
Time = = 1.25 hours = 1 hr 15 minutes
10
Answer: 1 hr 15 minutes
e¨vL¨v:
15 wgwbU †`ix‡Z hvÎv ïiæi Rb¨| cj †h `yiZ¡ wcwQ‡q _vK‡e wVK GKB Kvi‡Y †gwi mgvb `yiZ¡ GwM‡q
_vK‡e| GB `yiZ¡wUB n‡jv cj I †gwii g‡a¨Kvi Av‡cw¶K `yiZ¡| GB Av‡cw¶K `yiZ¡‡K, Av‡cw¶K †eM w`‡q
fvM Ki‡j mgq cvIqv hv‡e|
4) Dhaka Express and Khulna Express trains left from two stations
towards Khulna station and Dhaka station respectively. 3 hours after they
met the trains were 675 miles apart. The Dhaka Express arrived at its
destination 16 hours after their meeting and the Khulna Express arrived
at its destination 25 hours after their meeting. How many hours Dhaka
Express did it take for the whole trip? [DBBL, TO – 2024]
Solution
As the distance is equal the meeting time of two trains = 16 × 25 = 400 =
20 hours
Hence, time taken by Dhaka express = 20 + 16 = 36 hours
Answer 36 hours
5) Train X starts from New York at 10:00am travelling at x mile per hour.
Train Y starts at 11:30 from same station at same direction at 4x/3 miles
per hour. At what time Train Y will meet train X?
Solution
X train
Speed = x mph
Meeting time = t hr
Y train
4x
Speed = mph
3
3
Travelling time to catch train = x (t ‒ ) hr
2
We know that,
4x 3
Distance = speed × time = xt = ×(t ‒ )
3 2
Or, 3xt = 4xt – 6x
Or, 6x = xt
Or, t = 6 hr [ x ≠ 0]
Hence, train Y catches train X at 10:00+ 6 = 4 P.M
Answer: 4 P.M

136
[CAREER AID BY YOUSUF CHOTON] Written

6) A boy goes to school with a speed. of 3 km an hour and return with a


speed of 2 km/hr If he takes 5 hours in all the distance. Find the distance
between the village and the school?
Solution
Let, the distance between the village and the school is ‘d’ km
According to the question,
d d
+ =5
3 2
(2d + 3d)
Or, =5
6
Or, 5d = 30
30
Or, d =
5
⸫d = 6
Answer: 6
1
7) A ship 77 km from the shore springs a leak which admits 2 tones of
4
1
water in 5 minutes. 92 tones of water would sink it. But the pumps can
2
throw out 12 tones of water per hour. Find the averages rate of sailing so
that the ship may just reach the shore as it begins to sink. [Janata Bank PLC,
RC – 2024]
Solution
2.25×60 45×6 270
Here, In 1 hour Incoming Water to Ship = = = ton
5.5 11 11
270 138
So, In 1 hour water fills to the ship = ‒ 12 = ton
11 11
92×11 22
Required time to save 92 tones of water = =
138 3
77 3
So, average rate of sailing the ship = = 77× = 10.5 km
22 22
3
Answer10.5 km
8) Half an hour after car A started traveling from Newtown to Oldtown, a
distance of 62 miles, car B started traveling along the same road from
Oldtown to Newtown. The cars met each other on the road 15 minutes
after car B started it's trip. If car A traveled at a constant rate that was 8
miles per hour greater than car B's constant rate, how many miles had car
B driven when they met? [FSIBL, PO ‒ 2023]
Solution

137
[CAREER AID BY YOUSUF CHOTON] Written

3
Total traveling time for car A = 45 min or hour.
4
Let the speed of car B be x miles/hour.
So, car's A speed = (8+x) miles/hour
Total distance covered by car A and car B is 62 miles.

3 1
Therefore, the equation is ×(8+x) + ×x = 62
4 4
3x x
Or, 6 + + = 62 Or, x = 56
4 4
1 1
Distance traveled by car B = ×x = ×56 = 14
4 4

Answer 14 miles

138
[CAREER AID BY YOUSUF CHOTON] Written

Series
Basic
1) Two fractions are inserted between 1/4 and 1/2 so that the difference between
any two successive fractions is the same. Find the sum of the four fractions.
[DBBL, MTO – 2024]
Solution
As the difference of any two successive fractions is the same then we have evenly
spaced set (arithmetic progression). Sum of the terms of AP
1 1
+
First term + Last term 2 4 6 3
Now, The sum of the four fractions = ×4 = = ×4 = ×2 =
2 2 4 2
3
Answer
2
2) What is the average (arithmetic mean) of all multiples of 10 from 10 to 400
inclusive?
Solution:
Series = 10 + 20 + 30 + ....... + 400
Term = +1= +1 = 40
Sum = × (Last term + first term) = × (400+10) = 8200
Average of all multiples of 10 from 10 to 400 = = 205
Answer: 205
Arithmatic progression
3) Find the sum of all the two digit numbers that are divisible by 3. (`yB AsKwewkó
†hmKj msL¨v 3 Øviv wefvR¨ Zv‡`i †hvMdj wbY©q Kiyb) [Bangladesh Bank Officer General-2020]
Solution:
The smallest and the greatest two digit numbers that are divisible by 3 are 12 and 99
respectively. So, we have to find the sum of the following series: 12 + 15 + 18 + … + 99
It is an arithmetic progression where,
The first term, a = 12,
Common difference, d = (15-12) = 3,
Last term = 99, number of terms n =?
We know, 99 = a + (n-1) d
Or, 99 = 12 + (n-1) ×3
Or, 99-12 = 3n -3
Or, 3n = 90
Or, n = 30
We know,
Sum of first n terms = {2a+ (n-1) d} = {2×12+ (30-1)3} = 15× (24+87) = 1665.
So, the sum of all the two digit numbers that are divisible by 3 is 1665.
Answer: 1665
4) Three numbers are in A.P. and their sum is 30. Also the sum of their squares is
308. Find the numbers.

139
[CAREER AID BY YOUSUF CHOTON] Written

Solution:
Suppose, the numbers are (a - d), a, and (a + d)
Again, (a-d) ² + a² + (a + d) ² = 308
According to the question, Or, a² - 2ad + d² + a² + a² + 2ad + d² = 308
(a - d) + a + (a + d) = 30 Or, 3a² + 2d² = 308
Or, 3a = 30 Or, 3×10² + 2d² = 308
Or, a = 10 Or, 2d² = 308 - 300 = 8
Or, d² = 4
Or, d = 2
So, the numbers are: (a ‒ d) = 10 - 2 = 8, a = 10 and (a+ d) = 10 + 2 = 12
Answer: 8, 10 and 12
5) A series has three numbers a, ar and ar². In the series, the first term is twice
the second term. What is the ratio of the sum of the first two terms to the sum of
last two terms in the series?
Solution:
A Standard geometries series is a + ar + ar2 + . . . + arn-1
According to the question, a = 2ar
Or, r =
The sum of the first two terms in the series = a + ar = a (1+r) = a (1 + ) =
The sum of the last two terms in the series = ar + ar² = ar (1+r) = × a(1+r) = × =
So, the required ratio = : = : = 2: 1 Answer: 2: 1
6) Prove that the sum of the odd numbers from 1 to 125 inclusive is equal to the
sum of the odd numbers from 169 to 209 inclusive.[Agrani Bank Officer (Cash)-
2018]
Solution:
First series = 1 + 3 + 5 + 7 + . . . + 125
Here, First term, a = 1
Common difference, d = 2
n-th term = 125
Or, a + (n-1) ×d = 125 Or, 1 + (n-1) ×2 = 125
Or, n = 63
Sum of the series = {2a+ (n-1) d} = {2 ×1+ (63-1) × 2} = ×126 = 63 ×63 = 3969
Second series = 169 + 171 + 173 + . . . + 209
Here, First term, a = 169
Common difference, d = 2
n-th term = 209
Or, a + (n-1) ×d = 125
Or, 169 + (n-1) ×2 = 209
Or, n = 21
Sum of the series = {2a+ (n-1) d} = {2 ×169+ (21-1) × 2} = ×378 = 21 ×189 = 3969
Hence, the sum of the odd numbers from 1 to 125 inclusive is equal to the sum of the
odd numbers from 169 to 209 inclusive [Proven]

140
[CAREER AID BY YOUSUF CHOTON] Written

7) Mr. Rashid has saved Taka 1200 from his first month's salary. He plans to
increase his monthly savings by Taka 100 in every following month. How much
time would it take to save Taka 106200? [BKB- (officer)-2017 (Written)]
Solution:
First month savings = tk 1200
Second months savings = tk (1200+100) = tk 1300
The series becomes = 1200 +1300+1400 +. ..
Here, a = 1200
Common Difference = 1300-1200 = 100
Sum of the series = {2a+ (n-1) d} = 106200
Or, {2 ×1200+ (n-1) × 100} = 106200
Or, n (2400+ 100n- 100) = 106200×2
Or, 100n2 +2300n = 212400
Or, n2 + 23n – 2124 = 0
Or, n2 +59n – 36n -2124 = 0
Or, (n+59)(n-36) = 0
Therefore, n = 36 months Answer: 36 months
8) Two number x & y are in GP and their sum 30 and sum of their square 468.
Find the numbers. [RBL Officer General -2019]
Solution:
Sum, x + y = 30 Substituting we get,
Sum of their square, x2 + y2 = 468 x+ = 30
Now, (x + y)2 = 900 2
Or, x -30x + 216 = 0
Or, x2 + y2 + 2xy = 900
Or, (x - 12)(x - 18) = 0
Or, 2xy = 900 - 468 = 432
So, x = 12 or 18
Or, xy = 216
y = 18 or 12 (Answer)
Or, y =

9) Two men X and Y started working for a certain company at similar jobs on
January 1, 1950. X asked for an initial salary of Rs. 300 with an annual
increment of Rs. 30. Y asked for an initial salary of Rs. 200 with a rise of Rs. 15
every six months. Assume that the arrangements remained unaltered till
December 31, 1959. Salary is paid on the last day of the month. What is the total
amount paid to them as salary during the period? [GMAT, Janata Cash 2021]
Solution:
Initial salary of X in first year, a = Rs 300 × 12 = Rs 3600
Annual increment = Rs 30 × 12 = Rs 360
Initial salary of X in second year = 3600 + 360 = 3960
Difference, d = 3960 – 3600 = 360
Time, n = 10 years (From Jan 1, 1950 to Dec 31, 1959)
Arithmetic Progression for X’s salary = 3600 + 3960 + 4320 + up to 10th term
Total salary of X = [2a+ (n-1) ×d] = {2×3600 + (10-1)360}
= 5(7200+9×360) = 5×10440 = Rs 52200

141
[CAREER AID BY YOUSUF CHOTON] Written

Again, Semiannual salary of Y, b =200×6 = Rs 1200


Semi Annual increment = Rs 15 × 6 = Rs 90
Initial salary of X after six months = Rs (1200 +90) = Rs 1290
Difference, p = 1290 – 1200 = 90
Time, m = 20 years (From Jan 1, 1950 to Dec 31, 1959 in semiannual)
Arithmetic Progression for Y’s salary = 1200 + 1290 + 1380 + 1470 + up to 20th term
Salary of Y = [2b+ (m-1) ×p] = [2×1200 + (20 - 1) ×90] = 10 × 4110 = 41100
Total Salary = Rs (52200 + 41100) = Rs 93300
Answer: Rs 93,300
Probability
What is the probability of getting a sum of 9 when two dice are thrown
simultaneously?
Solution
Total outcome = 62=36
Favorable outcomes=(3,6), (4,5), (5,4), (6,3)
favorable outcome 4 1
So required probability = = =
total outcome 36 9
1
Answer:
9

142

You might also like